You are on page 1of 359

Civil Law Q&As (2007-2013)

hectorchristopher@yahoo.com

dbaratbateladot@gmail.com

A Compilation of the
Questions and Suggested Answers
In the
PHILIPPINE BAR EXAMINATIONS 2007-2013
In

CIVIL LAW
Compiled and Arranged By:
Baratbate-Ladot, Delight
Salise, Hector Christopher Jay-Arh Jr. M.
(University of San Jose-Recoletos School of Law)

ANSWERS TO BAR EXAMINATION QUESTIONS by


the UP LAW COMPLEX (2007-2013)
&
PHILIPPINE ASSOCIATION OF LAW SCHOOLS (2008)

Never Let The Odds Keep You From Pursuing What You Know In Your Heart You Were Meant To Do.Leroy Satchel Paige

Page 1 of 180

Civil Law Q&As (2007-2013)

hectorchristopher@yahoo.com

dbaratbateladot@gmail.com

FOREWORD
This work is a compilation of the ANSWERS TO BAR
EXAMINATION QUESTIONS by the UP LAW COMPLEX ,
Philippine Association of Law Schools from 2007-2010 and
local law students and lawyers forum sites from 2011-2013
and not an original creation or formulation of the author.
The authors were inspired by the work of Silliman Universitys
College of Law and its students of producing a very good
material to everyone involved in the legal field particularly the
students and the reviewees for free. Hence, this work is a
freeware.
Everyone is free to distribute and mass produce copies of this
work, however, the author accepts no liability for the content of
this reviewer, or for the consequences of the usage, abuse, or
any actions taken by the user on the basis of the information
given.
The answers (views or opinions) presented in this reviewer are
solely those of the authors in the given references and do not
necessarily represent those of the authors of this work.
The Authors.

Never Let The Odds Keep You From Pursuing What You Know In Your Heart You Were Meant To Do.Leroy Satchel Paige

Page 2 of 180

Civil Law Q&As (2007-2013)

hectorchristopher@yahoo.com

dbaratbateladot@gmail.com

TABLE OF CONTENTS
(Titles are based on Sillimans Compilation [Arranged by Topic])

Persons
Capacity: Juridical Capacity (2008).........................................................................................12
Capacity; Juridical Capacity of Donee; Requisites for Acceptance (2012)......................... 12
Capacity: Legal Capacity; Lex Rei Sitae (2007).......................................................................13
Correction of Entries; Clerical Error Act (2008).................................................................... 14
Nationality Principle (2009)...................................................................................................... 14
Nationality Principle; Change of Name not Covered (2009).................................................. 15

Conflict of Laws
Processual Presumption (2009).................................................................................................16
Jurisdiction; Courts may Assume Jurisdiction over Conflict of Laws Cases (2010).........17

Adoption
Adoption; Termination; Death of Adopter (2009)................................................................... 17
Adoption; Illegitimate Child (2010)..........................................................................................18
Adoption; Illegitimate Child; Use of Mothers Surname as Middle Name (2012)...............19
Consent; Consent of the Adopters Heirs (2008).................................................................... 19
Qualifications of Adopter (2010).............................................................................................. 20

Never Let The Odds Keep You From Pursuing What You Know In Your Heart You Were Meant To Do.Leroy Satchel Paige

Page 3 of 180

Civil Law Q&As (2007-2013)

hectorchristopher@yahoo.com

dbaratbateladot@gmail.com

Family Code
Marriage; Annulment; Grounds (2009).................................................................................... 20
Marriage; Annulment; Grounds (2007).................................................................................... 21
Marriage; Annulment; Parties (2012)........................................................................................22
Marriage; Annulment; Support Pendente Lite (2010)............................................................ 22
Marriage; Divorce Decrees; Filipino Spouse Becoming Alien (2009)................................... 23
Marriage; Divorce Decrees; Foreign Spouse Divorces Filipino Spouse (2012)....................24
Marriage; Divorce Decrees; Foreign Spouse Divorces Filipino Spouse (2010)....................25
Marriage; Legal Separation; Prescription (2012).................................................................... 25
Marriage; Legal Separation; Prescription (2007).................................................................... 26
Marriage; Psychological Incapacity (2013)............................................................................. 26
Marriage; Psychological Incapacity (2012)............................................................................. 28
Marriage; Requisites (2008)...................................................................................................... 28
Marriage; Subsequent Marriage (2008).................................................................................... 29
Marriage; Void Marriages; By Reason of Public Policy (2008)...............................................30
Marriage; Void Marriages; By Reason of Public Policy (2007)...............................................30
Marriage; Void Marriages; Property Relations (2009)............................................................ 30
Marriage; Void Marriages; Status of Children (2009)............................................................. 31
Parental Authority; Illegitimate Minor Child (2009)..............................................................32
Parental Authority; In Vitro Fertilization (2010)................................................................... 32

Never Let The Odds Keep You From Pursuing What You Know In Your Heart You Were Meant To Do.Leroy Satchel Paige

Page 4 of 180

Civil Law Q&As (2007-2013)

hectorchristopher@yahoo.com

dbaratbateladot@gmail.com

Paternity & Filiation; Child Born Under a Void Marriage (2010)........................................ 33


Paternity & Filiation; Impugning Legitimacy (2010)............................................................. 34
Paternity & Filiation; In Vitro Fertilization;
Surrogate Mothers Remedy to Regain Custody (2010).....................34
Paternity & Filiation; Legitimacy; Presumption (2008)........................................................ 35
Paternity & Filiation; Legitimation of a Child from a Previous Valid Marriage (2008).....36
Paternity & Filiation; Legitimation of a Dead Child (2009).................................................. 37
Paternity & Filiation; Support: Ascendants & Descendants;
Collateral Blood Relatives (2008)......................................................... 37
Paternity & Filiation; Use of Surname; Illegitimate Child (2009)........................................38
Paternity & Filiation; Who May Impugn Legitimacy (2009)................................................. 39
Property Relations; Adulterous Relationship (2009)............................................................. 39
Property Relations; Accession (2012)...................................................................................... 40
Property Relations; Ante-Nuptial Debt (2007)........................................................................ 40
Property Relations; Unions Without Marriage (2012)............................................................ 41
Property Relations; Unions Without Marriage (2012)............................................................ 42
Property Relations; Void Marriages (2010)..............................................................................43
Property Relations; Void Marriages (2010)..............................................................................45

Never Let The Odds Keep You From Pursuing What You Know In Your Heart You Were Meant To Do.Leroy Satchel Paige

Page 5 of 180

Civil Law Q&As (2007-2013)

hectorchristopher@yahoo.com

dbaratbateladot@gmail.com

Succession
Disposition; Mortis Causa vs. Intervivos; Corpse (2009)...................................................... 46
Heirs; Fideicommissary Substitution (2008)..........................................................................46
Heirs; Intestate Succession; Legitime; Computation (2010)................................................47
Heirs; Representation; Iron-Curtain Rule (2012)................................................................... 49
Heirs; Reserva Troncal (2009).................................................................................................. 49
Intestate Succession (2008)...................................................................................................... 50
Intestate Succession (2008)...................................................................................................... 51
Intestate Succession; Rights of Representation:
Illegitimate, Adopted Child; Iron Curtain Rule (2007)...................... 51
Legitimes; Compulsory Heirs (2012)....................................................................................... 53
Legitime; Compulsory Heirs (2008)..........................................................................................53
Preterition; Disinheritance (2008)...........................................................................................54
Succession; Proof of Death between persons called to succeed each other (2008)..........55
Succession; Rule on Survivorship (2009)............................................................................... 56
Wills; Holographic Wills; Insertions & Cancellations (2012)................................................. 57
Wills; Holographic Wills; Probate (2009)..................................................................................57
Wills; Joint Wills (2008)............................................................................................................. 59
Wills; Joint Wills; Probate (2012)............................................................................................. 59

Never Let The Odds Keep You From Pursuing What You Know In Your Heart You Were Meant To Do.Leroy Satchel Paige

Page 6 of 180

Civil Law Q&As (2007-2013)

hectorchristopher@yahoo.com

dbaratbateladot@gmail.com

Wills; Prohibition to Partition of a Co-Owned Property (2010)............................................ 60


Wills; Notarial Wills; Blind Testator; Requisites (2008)......................................................... 61
Wills; Testamentary Disposition; Period to Prohibit Partition (2008)................................. 61
Wills; Witnesses to a Will, Presence required; Thumbmark as Signature (2007)............... 62

Donation
Donations; Formalities; In Writing (2007)...............................................................................63
Donations; Illegal & Impossible Conditions (2007)................................................................64
Donation; Inter Vivos (2013)..................................................................................................... 64

Property
Accretion; Alluvium (2008)....................................................................................................... 65
Accretion; Rights of the Riparian Owner (2009).................................................................... 67
Builder; Good Faith; Requisites (2013).................................................................................... 68
Easement; Prescription; Acquisitive Prescription (2009)..................................................... 70
Easement; Right of Way (2013).................................................................................................70
Easement; Right of Way (2010).................................................................................................72
Hidden Treasure (2008).............................................................................................................73
Mortgage; Public or Private Instrument (2013)...................................................................... 74
Occupation vs. Possession (2007)............................................................................................ 76
Ownership; Co-Ownership (2009)............................................................................................. 76
Ownership; Co-Ownership (2008)............................................................................................. 77

Never Let The Odds Keep You From Pursuing What You Know In Your Heart You Were Meant To Do.Leroy Satchel Paige

Page 7 of 180

Civil Law Q&As (2007-2013)

hectorchristopher@yahoo.com

dbaratbateladot@gmail.com

Property; Movable or Immovable (2007).................................................................................. 78

Land Titles and Deeds


Acquisition of Lands; Sale of Real Property to an Alien (2009)........................................... 79
Non-Registrable Properties (2007)............................................................................................80
Prescription; Acquisitive Prescription (2008).........................................................................81
Prescription; Judicially Foreclosed Real Property Mortgage (2012)................................... 82
Purchaser in Good Faith; Mortgaged Property (2008)........................................................... 83
Registration; Governing Law (2007).........................................................................................84
Registration; Party Who First took Possession (2013)..........................................................85
Registration; Requisites; Proof (2013)..................................................................................... 86
Remedies; Fraud; Rights of Innocent Purchaser (2009)........................................................89

Contracts
Contract to Sell vs. Conditional Contract of Sale (2012)......................................................90
Rescission of Contract; Fortuitous Event (2008)................................................................... 90
Stipulation; Arbitration Clause (2009)..................................................................................... 91

Obligations
Extinguishment; Compensation (2009)................................................................................... 91
Extinguishment; Compensation (2008)................................................................................... 92
Extinguishment; Novation (2008)............................................................................................ 93
Extinguishment; Payment of Check (2013).............................................................................94

Never Let The Odds Keep You From Pursuing What You Know In Your Heart You Were Meant To Do.Leroy Satchel Paige

Page 8 of 180

Civil Law Q&As (2007-2013)

hectorchristopher@yahoo.com

dbaratbateladot@gmail.com

Extinguishment; Payment of Check; Legal Tender (2008)....................................................95


Liability; Solidary Liability (2008)............................................................................................96
Obligations; Without Agreement (2007)..................................................................................97

Trust
Trust De Son Tort (2007)...........................................................................................................98

Sales
Condominium Act; Partition of a Condominium (2009)........................................................99
Mortgage; Equitable Mortgage (2012)...................................................................................... 99
Option Contract; Liquor & Pulutan as Consideration (2013).......................................... 100
Right of First Refusal; Lessee; Effect (2008).........................................................................101

Lease
Builder; Good Faith; Useful Improvements (2013)...............................................................103
Lease; Caveat Emptor (2009).................................................................................................. 104

Agency
Agency; Sale of a Real Property through an Agent (2010).................................................. 104

Partnership
Liability; Liability of a Partner (2010)................................................................................... 105
Oral Partnership (2009)...........................................................................................................106
Share; Demand during the Existence of Partnership (2012)...............................................107

Never Let The Odds Keep You From Pursuing What You Know In Your Heart You Were Meant To Do.Leroy Satchel Paige

Page 9 of 180

Civil Law Q&As (2007-2013)

hectorchristopher@yahoo.com

dbaratbateladot@gmail.com

Commodatum & Mutuum


Mutuum; Interest; Solutio Indebiti (2012)............................................................................ 107

Guaranty
Guaranty (2009)........................................................................................................................ 108

Surety
Surety (2010).............................................................................................................................108

Pledge
Pledge; Pactum Commissorium (2009)..................................................................................109

Torts and Damages


Damages (2012)........................................................................................................................ 109
Damages; Moral & Exemplary (2009)..................................................................................... 110
Damages; Public Officers acting in the Performance of their Duties (2012)...................111
Death Indemnity (2009)........................................................................................................... 112
Doctrine of Discovered Peril (Last Clear Chance) (2007).....................................................112
Liability; Owner of a Pet; Fortuitous Event (2010)..............................................................113
Liability; Special Parental Authority (2010).........................................................................115
Quasi-Delict; Claims; Requisites (2013).................................................................................115
Quasi Tort (2010)...................................................................................................................... 116

Never Let The Odds Keep You From Pursuing What You Know In Your Heart You Were Meant To Do.Leroy Satchel Paige

Page 10 of 180

Civil Law Q&As (2007-2013)

hectorchristopher@yahoo.com

dbaratbateladot@gmail.com

MULTIPLE CHOICE QUESTIONS


2013 Civil Law Exam MCQ (October 13, 2013)..............118
2012 Civil Law Exam MCQ (October 14, 2012).................130
2011 Civil Law Exam MCQ (November 13, 2011)..............149
2010 Civil Law Exam MCQ (September 12, 2010).............176
2007 Civil Law Exam MCQ (September 09, 2007).............179

Never Let The Odds Keep You From Pursuing What You Know In Your Heart You Were Meant To Do.Leroy Satchel Paige

Page 11 of 180

Civil Law Q&As (2007-2013)

hectorchristopher@yahoo.com

be considered born if it is alive at the


time it is completely delivered from the

Persons
Capacity: Juridical Capacity (2008)
No. II. At age 18, Marian found out that she
was pregnant. She insured her own life and
named her unborn child

as her sole

beneficiary. When she was already due to


give birth, she and her boyfriend Pietro, the
father

of

her

unboarn

child,

were

kidnapped in a resort in Bataan where they


were vacationing. The military gave chase
and after one week, they were found in an
abandoned hut in Cavite. Marian and Pietro
were hacked with bolos. Marian and the
baby delivered were both found dead, with
the baby's umbilical cord already cut. Pietro
survived.
(A). Can Marian's baby be the beneficiary of
the insurance taken on the life of the
mother? (2%)
SUGGESTED ANSWER:
Yes, the baby can be the beneficiary of
the life insurance of Marian. Art. 40 NCC
provides

that

personality;

but

"birth
the

determines

conceived

child

shall be considered born for all purposes


that are favorable to it, provided that it
be

born

later

with

dbaratbateladot@gmail.com

the

conditions

specified in Art. 41. Article 41 states


that "for civil purposes, the fetus shall

child was more than seven months old.


mother's womb. However, if the fetus
had an intra-uterine life of less than
seven months, it is not deemed born if it
dies within twenty-four (24) hours after
its complete delivery from the maternal
womb. The act of naming the unborn

Capacity; Juridical Capacity of Donee;


Requisites for Acceptance (2012)
No.I. b) Ricky donated P 1 Million to the
unborn child of his pregnant girlfriend,

child as sole beneficiary in the insurance

which she accepted. After six (6) months of

is favorable to the conceived child and

pregnancy, the fetus was born and baptized

therefore the fetus acquires presumptive

as Angela. However, Angela died 20 hours

or provisional personality. However, said

after birth. Ricky sought to recover the P 1

presumptive personality only becomes

Million.

conclusive if the child is born alive. The


child need not survive for twenty-four

Is

Ricky

entitled

to

recover?

Explain. (5%)
SUGGESTED ANSWER:

(24) hours as required under Art. 41 of


the Code because "Marian was already
due to give birth," indicating that the

Yes, Ricky is entitled to recover the


P1,000,000.00. The NCC considers a

Never Let The Odds Keep You From Pursuing What You Know In Your Heart You Were Meant To Do.Leroy Satchel Paige

Page 12 of 180

Civil Law Q&As (2007-2013)

hectorchristopher@yahoo.com

fetus a person for purposes favorable to


it provided it is born later in accordance
with the provision of the NCC. While the
donation is favorable to the fetus, the
donation did not take effect because the
fetus was not born in accordance with
the NCC.
To be considered born, the fetus that
had an intrauterine life of less than
seven (7) months should live for 24
hours from its complete delivery from
the mothers womb. Since Angela had an
intrauterine life of less than seven (7)
months but did not live for 24 hours, she
was not considered born and, therefore,
did not become a person. Not being a
person, she has no juridical capacity to
be a donee, hence, the donation to her
did not take effect. The donation not
being effective, the amount donated may
be recovered. To retain it will be unjust
enrichment.

Capacity: Legal Capacity; Lex Rei Sitae


(2007)
No.VII. Write "TRUE" if the statement is
true or "FALSE" if the statement is false. If
the statement is FALSE, state the reason.
(2% each).

dbaratbateladot@gmail.com

(1). Roberta, a Filipino, 17 years of age,


without the knowledge of his parents, can

contract in Australia is governed by


acquire

house

in

Australia

because

Philippine

Law,

property from the age of 16.

until the contract is annulled.

SUGGESTED ANSWER:

ALTERNATIVE ANSWER:

TRUE. Since Australian Law allows alien

FALSE. Laws relating to family rights

to acquire property from the age of 16,

and duties, or to the status, condition or

Roberta may validly own a house in

legal capacity of persons are binding

Australia, following the principle of lex

upon the citizens of the Philippines,

rei sitae enshrined in Art. 16, NCC,

even though living abroad (Art. 15, NCC).

which states "Real property as well as

The age of majority under Philippine law

personal property is subject to the law of

is 18 years (R.A. No. 6809); hence,

the

Roberta, being only 17 years old, has no

Moreover,

even

it

is

assuming

situated."
that

legal

property

acquire

ownership

where

the

will

Australian Laws allow aliens to acquire

country

over

she

bought

legal capacity to acquire and own land.

capacity of Roberta in entering the

Never Let The Odds Keep You From Pursuing What You Know In Your Heart You Were Meant To Do.Leroy Satchel Paige

Page 13 of 180

Civil Law Q&As (2007-2013)

hectorchristopher@yahoo.com

surname in accordance with Rule 108 of

Correction of Entries; Clerical Error Act


(2008)
No. IV. Gianna was born to Andy and
Aimee, who at the time Gianna's birth were
not married to each other. While Andy was
single at the time, Aimee was still in the
process of securing a judicial declaration of
nullity on her marriage to her ex-husband.
Gianna's birth certificate, which was signed
by both Andy and Aimee, registered the
status

of

Gianna

as

"legitimate",

her

surname carrying that of Andy's and that


her parents were married to each other.
(A). Can a judicial action for correction of
entries in Gianna's birth certificate be
successfully maintained to:
a). Change her status from "legitimate" to
"illegitimate" (1%);
and
b). Change her surname from that of Andy's
to Aimee's maiden surname? (1%)
SUGGESTED ANSWER:
Yes, a judicial action for correction of
entries in Gianna's birth certificate can
be successfully maintained to change (a)
her

status

from

"legitimate"

to

"illegitimate," and (b) her surname from


that

of

Andy's

to

dbaratbateladot@gmail.com

Aimee's

maiden

entry and/or change the first name or


the Rules of Court because said changes

nickname in the civil register without


need of a judicial order. Errors that

are substantive corrections.

involve the change of nationality, age,


(B).

can

status, surname or sex of petitioner are

administrative proceedings be brought for

not included from the coverage of the

the

Instead
purpose

of

a
of

judicial
making

action,
the

above

corrections? (2%)

said Act (Silverio v. Republic, G.R. No.


174689, 22 Oct., 2007).

SUGGESTED ANSWER:
No. An administrative proceeding cannot
be brought for the purpose of making the
above corrections. R.A. 9048, otherwise
known as the Clerical Error Act, which

Nationality Principle (2009)


No.XII. Emmanuel and Margarita, American
citizens and employees of the U.S. State
Department, got married in the African

authorizes the city or municipal civil

state of Kenya where sterility is a ground for

registrar or the consul general to correct

annulment of marriage. Thereafter, the

a clerical or typographical error in an

spouses were assigned to the U.S. Embassy


in Manila. On the first year of the spouses

Never Let The Odds Keep You From Pursuing What You Know In Your Heart You Were Meant To Do.Leroy Satchel Paige

Page 14 of 180

Civil Law Q&As (2007-2013)

hectorchristopher@yahoo.com

Court of Appeals 345 SCRA 92 [2000],

tour of duty in the Philippines, Margarita


filed an annulment case against Emmanuel
before a Philippine court on the ground of
her husbands sterility at the time of the
celebration of the marriage.
(A). Will the suit prosper? Explain your
answer. (3%)
SUGGESTED ANSWER:
No, the suits will not prosper. As applied
to foreign nationals with the respect to
family relations and status of persons,
the nationality principle set forth in
Article 15 of the Civil Code will govern
the

relations

Margarita.

of

Since

Emmanuel

they

are

and

American

citizens, the governing law as to the


ground for annulment is not Kenyan Law
which Magarita invokes in support of
sterility as such ground; but should be
U.S. Law, which is the national Law of
both

Emmanuel

and

Margarita

as

recognized under Philippine Law. Hence,


the Philippine court will not give due
course to the case based on Kenyan Law.
The nationality principle as expressed in
the application of national law of foreign
nationals

by

dbaratbateladot@gmail.com

Philippine

courts

is

established by precedents (Pilapil v. IbaySomera, 174 SCRA 653[1989], Garcia v.


Recio, 366 SCRA 437 [2001], Llorente v.

and Bayot v. Court of Appeals 570 SCRA


472 [2008]).

not Covered (2009)


No.XX. (A). If Ligaya, a Filipino citizen

ALTERNATIVE ANSWER:
The

forum

has

jurisdiction

over

an

action for the annulment of marriage


solemnized elsewhere but only when the

residing in the United States, files a petition


for change of name before the District Court
of New York, what law shall apply? Explain.
(2%)

party bringing the actions is domiciled


in the forum. In this case, none of the

SUGGESTED ANSWER:

parties to the marriage is domiciled in

New York law shall apply. The petition of

the

as

change of name file din New York does

officials of the US Embassy whose stay in

not concern the legal capacity or status

the country is merely temporary, lasting

of the petitioner. Moreover, it does nto

only during their fixed tour of duty.

affect the registry of any other country

Hence, the Philippine courts have no

including the country of birth of the

jurisdiction over the action.

petitioner.

Philippines.

They

are

here

Whatever

judgment

is

rendered in that petition will have effect


only in New York. The New York court
Nationality Principle; Change of Name

Never Let The Odds Keep You From Pursuing What You Know In Your Heart You Were Meant To Do.Leroy Satchel Paige

Page 15 of 180

Civil Law Q&As (2007-2013)

hectorchristopher@yahoo.com

Philippine law will apply. The petition for

cannot, for instance, order the Civil


Registrar in the Philippines to change its
records. The judgment of the New York
court allowing a change in the name of
the petitioner will be limited to the
records of the petitioner in New York and
the

use

of

her

new

name

in

all

transactions in New York. Since the


records and processes in New York are
the only ones affected, the New York
court

will

apply

New

YorK

law

in

resolving the petition.


ALTERNATIVE ANSWER:
Philippine law shall apply (Art 15, NCC).
Status, conditions, family rights and
duties are governed by Philippine laws as
to

Filipinos

even

though

sojourning

abroad.
ALTENATIVE ANSWER:
If Ligaya, a Filipino, files a petition for
change of name with the District Court
of New YoRk, the laws of New York will
govern since change of name is not one
of those covered by the principles of
nationality.
(B). If Henry, an American citizen residing
in the Philippines, files a petition for change
of name before a Philippine court, what law
shall apply? Explain. (2%)
SUGGESTED ANSWER:

dbaratbateladot@gmail.com

change of name in the Philippines will


affect only the records of the

of
petitioner and his transactions in the
Philippines. The Philippine court can
never

acquire

jurisdiction

over

the

custodian in the US of the records of the


petitioner. Moreover, change of name has
nothing to do with the legal capacity or
status of the alien. Since Philippine
records and transactions are the only
ones affected, the Philippine court may

lex

the

nationality

governed by national law, the matter of


change of name being included in the
legal status. The Supreme Court has
reiterate in several cases, that the lex
patriae as provided in Article 15 of the
Civil

Code

nationals

is

in

applicable

determining

to

foreign

their

legal

status (supra).

Conflict of Laws

with the laws governing those records

Philippine law.

or

principle, by which his legal status is

effect the change only in accordance

and transactions that law cannot be but

patriae

Processual Presumption (2009)

ALTERNATIVE ANSWER:

No.I. TRUE or FALSE. Answer TRUE if the

U.S. law shall apply as it is his national

statement

law. This is pursuant to the application

statement is false. Explain your answer in

is

true,

or

FALSE

if

the

not more than two (2) sentences.


Never Let The Odds Keep You From Pursuing What You Know In Your Heart You Were Meant To Do.Leroy Satchel Paige

Page 16 of 180

Civil Law Q&As (2007-2013)

(A).

The

doctrine

hectorchristopher@yahoo.com

ALTERNATIVE ANSWER:

of

"processual

presumption" allows the court of the forum


to presume that the foreign law applicable
to the case is the same as the local or
domestic law. (1%)
SUGGESTED ANSWER:
TRUE. If the foreign law necessary to the
resolve an issue is not proven as a fact,
the court of the forum may presume that
the foreign law is the same as the law of
the forum.

Jurisdiction;

Courts

may

Assume

Jurisdiction over Conflict of Laws Cases


(2010)
No.III. Define, Enumerate or Explain. (2%
each)
(C) Give at least two reasons why a court
may assume jurisdiction over a conflict of
laws case.
SUGGESTED ANSWER:
(1) Statute theory. There is a domestic law
authorizing the local court to assume
jurisdiction.
(2) Comity theory. The local court assumes
jurisdiction based on the principle of
comity or courtesy.

dbaratbateladot@gmail.com

(2009)
(1) Public Order. To maintain peace and
order, disputes that disturb the peace of

No.XIII. Rafael, a wealthy bachelor, filed a


petition for the adoption of Dolly, a one-year

the forum should be settled by the court

old foundling who had a severe heart ailment.

of the forum even though the application

During

of the foreign law is necessary for the

proceedings, Rafael died of natural causes.

purpose.

The Office of the Solicitor General files a

(2) Humanitarian Principle. An aggrieved


party should not be left without remedy

the

pendency

of

the

adoption

motion to dismiss the petition on the ground


that the case can no longer proceed because
of the petitioners death.

in a forum even though the application


of the foreign law by the courts of the

(A). Should the case be dismissed? Explain.

forum is unavoidable in order to extend

(2%)

relief.

SUGGESTED ANSWER:

Adoption
Adoption; Termination; Death of Adopter

It

depends

on

the

stage

of

the

proceedings when Rafael died. If he died


after all the requirements under the law

Never Let The Odds Keep You From Pursuing What You Know In Your Heart You Were Meant To Do.Leroy Satchel Paige

Page 17 of 180

Civil Law Q&As (2007-2013)

hectorchristopher@yahoo.com

adoptee who dies because adoption is

have been complied with and the case is


already submitted for resolution, the
court may grant the petition and issue a
decree of adoption despite the death of
the

adopter

(Section

13,

RA

8552).

Otherwise, the death of the petitioner


shall have the effect terminating the
proceedings.
(B). Will your answer be the same if it was
Dolly who died during the pendency of the
adoption proceedings? Explain. (2%)
SUGGESTED ANSWER:
No, if it was Dolly who died, the case
should

be

dismissed.

Her

death

terminates the proceedings (Art. 13,


Domestic Adoption Law).
ALTERNATIVE ANSWER:
It depends. If all the requirements under
the law have already been complied with
and the case is already submitted for
resolution, the death of the adoptee
should not abate the proceedings. The
court

should

issue

dbaratbateladot@gmail.com

the

decree

of

adoption if will be for the best interest of


the adoptee. While RA8552 provides only
for the case where it is the petitioner
who dies before the decree is issued, it is
with more compelling reason that the
decree should be allowed in case it is the

primarily for his benefit.

SUGGESTED ANSWER:
Adoption; Illegitimate Child (2010)
No.VIII. Spouses Rex and Lea bore two
children now aged 14 and 8. During the

The

consent

of

the

14-year-old

legitimate child, of the 10- year -old


illegitimate child and of the biological

subsistence of their marriage, Rex begot a

mother of the illegitimate child are

child by another woman. He is now 10

needed for the adoption (Section 7 and

years of age.

9, RA 8552). The consent of Lea is no


longer

On Leas discovery of Rexs fathering a child


by another woman, she filed a petition for
legal separation which was granted.
Rex now wants to adopt his illegitimate
child.
(A) Whose consent is needed for Rexs
adoption of his illegitimate child? (2.5%)

required

because

there

was

already a final decree of legal separation.


(B) If there was no legal separation, can Rex
still adopt his illegitimate child? Explain.
(2.5%)
SUGGESTED ANSWER:
Yes, he can still adopt his illegitimate
child but with the consent of his spouse,

Never Let The Odds Keep You From Pursuing What You Know In Your Heart You Were Meant To Do.Leroy Satchel Paige

Page 18 of 180

Civil Law Q&As (2007-2013)

hectorchristopher@yahoo.com

law is silent as to what middle name an

of his 14-year-old legitimate child, of the


illegitimate child, and of the biological
mother of the illegitimate child (Section
7 and 9, RA 8552).

Adoption;
Mothers

Illegitimate Child; Use of


Surname

as

Middle

Name

(2012)
No.IV.b) Honorato filed a petition to adopt
his

minor

illegitimate

child

Stephanie,

alleging that Stephanies mother is Gemma


Astorga Garcia; that Stephanie has been
using

her

mothers

middle

name

and

surname; and that he is now a widower and


qualified to be her adopting parent. He
prayed that Stephanies middle name be
changed from "Astorga" to "Garcia," which is
her

mothers

surname

dbaratbateladot@gmail.com

and

that

her

surname "Garcia" be changed to "Catindig,"


which is his surname. This the trial court
denied. Was the trial court correct in
denying Hororatos request for Stephanies
use of her mothers surname as her middle
name? Explain. (5%)
SUGGESTED ANSWER:
No, the trial court was not correct. There
is no law prohibiting an illegitimate child
adopted by his natural father to use as
middle name his mothers surname. The

adoptee may use. In case of In re:

Adoption of Stephanie Nathy Astorga

Consent; Consent of the Adopters Heirs

Garcia, G.R. No. 148311, March 31,

(2008)

2005, the Supreme Court ruled that the


adopted child may use the surname of
the natural mother as his middle name
because there is no prohibition in the

No.V. Despite several relationships with


different
unmarried.

women,
His

Andrew

first

remained

relationship

with

Brenda produced a daughter, Amy, now 30

law against it. Moreover, it will also be

years old. His second, with Carla, produced

for the benefit of the adopted child who

two sons: Jon and Ryan. His third, with

shall

his

Donna, bore him no children although

mothers side and reinforce his right to

Elena has a daughter Jane, from a previous

inherit from his mother and her family.

relationship. His last, with Fe, produced no

Lastly, it will make the adopted child

biological

conform with the time-honored Filipino

adopted without court proceedings, Sandy's

preserve

tradition

of

his

lineage

carrying

the

on

mothers

surname as the persons middle name.

children

but

they

informally

now 13 years old, whom they consider as


their own. Sandy was orphaned as a baby
and was entrusted to them by the midwife
who attended to Sandy's birth. All the

Never Let The Odds Keep You From Pursuing What You Know In Your Heart You Were Meant To Do.Leroy Satchel Paige

Page 19 of 180

Civil Law Q&As (2007-2013)

hectorchristopher@yahoo.com

including Amy and Elena obtains the

children, including Amy, now live with

(A). Is there any legal obstacle to the legal


adoption of Amy by Andrew? To the legal
adoption of Sandy by Andrew and Elena?
(2%)
SUGGESTED ANSWER:
Yes, there is a legal obstacle to the legal
adoption of Amy by Andrew. Under Sec.
9(d) of RA 8552, the New Domestic
Act

consent

of

of

1998,
the

the

written

illegitimate

sons/daughters, ten (10) years of age or


over, of the adopter, if living with said
adopter and the latter's spouse, if any, is
necessary

to

the

adoption.

All

the

children of Andrew are living with him.


Andrew needs to get the written consent
of Jon, Ryan, Vina and Wilma, who are
all ten (10) years old or more. Sandy's
consent

to

Amy's

written consent of Jane, if she is over


ten years old (Sec. 9(d), RA 8552).

andrew in his house.

Adoption

dbaratbateladot@gmail.com

adoption

is

not

necessary because she was not legally


adopted by Andrew. Jane's consent is
likewise not necessary because she is
not a child of Andrew. Sandy, an orphan
since birth, is eligible for adoption under
Sec. 8(f) of RA 8552, provided that
Andrew obtains the written consent of
the other children mentioned above,

husband and wife to adopt jointly except


on certain situations enumerated in the

Qualifications of Adopter (2010)

law. The case of John does not fall in any


No.IX. Eighteen-year old Filipina Patrice

of the exceptions (R.A. 8552).

had a daughter out of wedlock whom she

Family Code

named Laurie. At 26, Patrice married


American citizen John who brought her to
live with him in the United States of
America.

John

at

once

signified

his

willingness to adopt Laurie.

Marriage; Annulment; Grounds (2009)


No.XII. Emmanuel and Margarita, American
citizens and employees of the U.S. State

Can John file the petition for adoption? If


yes, what are the requirements? If no, why?
(5%)

Department, got married in the African


state of Kenya where sterility is a ground for
annulment of marriage. Thereafter, the
spouses were assigned to the U.S. Embassy
in Manila. On the first year of the spouses

SUGGESTED ANSWER:

tour of duty in the Philippines, Margarita


No, John cannot file the petition to

filed an annulment case against Emmanuel

adopt

before a Philippine court on the ground of

alone.

Philippine

law

requires

Never Let The Odds Keep You From Pursuing What You Know In Your Heart You Were Meant To Do.Leroy Satchel Paige

Page 20 of 180

Civil Law Q&As (2007-2013)

hectorchristopher@yahoo.com

governed by Kenyan law and any issue as

her husbands sterility at the time of the


celebration of the marriage.
(B). Assume Emmanuel and Margarita are
both

Filipinos.

Kenya,

they

After

come

their

back

wedding

and

take

in
up

residence in the Philippines. Can their


marriage be annulled on the ground of
Emmanuels sterility? Explain. (3%)
SUGGESTED ANSWER:
No, the marriage cannot be annulled
under the Philippine law. Sterility is not
a ground for annulment of marriage
under Article 45 of the Family Code.
ALTERNATIVE ANSWER:
No, the marriage cannot be annulled in
the Philippines.

The

Philippine

court

dbaratbateladot@gmail.com

shall

have

jurisdiction over the action to annul the


marriage not only because the parties
are residents of the Philippines but
because they are Filipino citizens. The
Philippine court, however, shall apply
the law of the place where the marriage
was celebrated in determining its formal
validity (Article 26, FC; Article 17, NCC).

Since the marriage was celebrated in


Kenya in accordance with Kenyan law,
the formal validity of such marriage is

to the formal validity of that marriage

Since
shall be determined by applying Kenyan

sterility

does

not

constitute

absence or defect in the legal capacity of


the parties under Philippine law, there is

law and not Philippine law.

no

ground

to

avoid

or

annul

the

However, while Kenyan law governs the

marriage. Hence, the Philippine court

formal validity of the marriage, the legal

has to deny the petition.

capacity of the Filipino parties to the


marriage is governed not by Kenyan law
but by Philippine law (Article 15, NCC).
Sterility of a party as a ground for the

Marriage; Annulment; Grounds (2007)

annulment of the marriage is not a

No. VII. Write "TRUE" if the statement is

matter of form but a matter of legal

true or "FALSE" if the statement is false. If

capacity. Hence, the Philippine court

the statement is FALSE, state the reason.

must

apply

Phillippine

law

in

determining the status of the marriage


on the ground of absence or defect in the
legal capacity of the Filipino parties.

(2% each).
(4). The day after John and Marsha got
married,

John

told

her

that

he

was

impotent. Marsha continued to live with

Never Let The Odds Keep You From Pursuing What You Know In Your Heart You Were Meant To Do.Leroy Satchel Paige

Page 21 of 180

Civil Law Q&As (2007-2013)

hectorchristopher@yahoo.com

by any of the spouses.

John for 2 years. Marsha is now estopped


from filing an annulment case against
John.
SUGGESTED ANSWER:
FALSE. Marsha is not estopped from
filing an annulment case against John on
the ground of his impotence, because
she learned of his impotence after the
celebration of the marriage and not
before.

Physical

incapacity

to

consummate is a valid ground for the


annulment of marriage if such incapacity
was existing at the time of the marriage,
continues and appears to be incurable.
The marriage may be annulled on this
ground

within

five

dbaratbateladot@gmail.com

years

from

its

celebration.

Marriage; Annulment; Parties (2012)


No.IX.b) A petition for declaration of nullity
of a void marriage can only be filed by
either the husband or the wife? Do you
agree? Explain your answer. (5%)
SUGGESTED ANSWER:
Yes, I agree. Under the rules promulgated
by the Supreme Court, a direct action for
declaration of nullity may only be filed

ALTERNATIVE SUGGESTED ANSWER:

two business firms to G and their two


No, I do not agree. There are others who
may file a petition for declaration of

children, aged 18 and 19.


B also opened a bank account in the

nullity such as the other spouse in

amount of P3 million in the name of the two

bigamous marriages.

children to answer for their educational


expenses until they finish their college
degrees.

Marriage; Annulment; Support Pendente


Lite (2010)

day-to-day living expenses and upkeep of

No.V. G filed on July 8, 2000 a petition for


declaration of nullity of her marriage to B.
During the pendency of the case, the
couple

entered

agreement

For her part, G undertook to shoulder the

to

into
dissolve

compromise

their

absolute

community of property. B ceded his right to


their house and lot and all his shares in

the

children.

The

Court

approved

the

spouses agreement on September 8, 2000.


(A) Suppose the business firms suffered
reverses, rendering G unable to support
herself and the children. Can G still ask for
support pendente lite from B? Explain. (3%)

Never Let The Odds Keep You From Pursuing What You Know In Your Heart You Were Meant To Do.Leroy Satchel Paige

Page 22 of 180

Civil Law Q&As (2007-2013)

hectorchristopher@yahoo.com

Marriage;

SUGGESTED ANSWER:
If B acquiesces and does not file the

dbaratbateladot@gmail.com

Divorce

Decrees;

Filipino

Spouse Becoming Alien (2009)

action to impugn the legitimacy of the

No.IV. Harry married Wilma, a very wealthy

child within the prescriptive period for

woman. Barely five (5) years into the

doing so in Article 170 of the Family

marriage, Wilma fell in love with Joseph.

Code, G's daughter by another man shall

Thus, Wilma went to a small country in

be

Europe, became a naturalized citizen of

conclusively

presumed

as

the

that country, divorced Harry, and married

legitimate daughter of B by G.

Joseph.

year

thereafter,

Joseph

had squandered the P3 million fund for

permanent residence in the Philippines.

their college degrees, can they ask for more


support from B? Explain. (3%)

and

and

(B) Suppose in late 2004 the two children

their education before they could obtain

returned

Wilma

established

(A). Is the divorce obtained by Wilma from


Harry

recognized

in

the

Philippines?

Explain your answer. (3%)

SUGGESTED ANSWER:

SUGGESTED ANSRWER :

Yes, the two children can still ask for


support for schooling or training for

As to Wilma, the divorced obtained by

some

her

professions,

trade

or

vocation,

is

recognized

as

they shall have finished or completed

foreigner. Philippine personal laws do

their education (Article 194, Paragraph

not

2, Family Code; Javier v. Lucero, 94 Phil.

recognition of the divorce as regards

634 {1954}].Their having squandered the

Harry will depend on the applicability to

money given to them for their education

his case of the second paragraph of

will not deprive them of their right to

Article 26 of the Family Code. If it is

complete an education, or to extinguish

applicable, divorce is recognized as to

the obligation of the parents to ensure

him and, therefore, he can remarry.

the future of their children.

However, if it is not applicable, divorce is


not

recognized

is

the

Philippines

to

she

in

even beyond the age of majority until

apply

because

valid

foreigner.

as

to

now

However,

him

consequently, he cannot remarry.

and,

ALTERNATIVE ANSWER:

Never Let The Odds Keep You From Pursuing What You Know In Your Heart You Were Meant To Do.Leroy Satchel Paige

Page 23 of 180

Civil Law Q&As (2007-2013)

hectorchristopher@yahoo.com

Yes , the divorce obtained by Wilma is


recognized as valid in the Philippines. At
the time she got the divorce, she was
already a foreign national having been
naturalized as a citizen of that small
country in Europe. Based on precedents
established

by

the

Supreme

Court

( Bayot v. CA, 570 SCRA 472 [2008]),


divorce

obtained

by

foreigner

is

recognized in the Philippines if validly


obtained in accordance with his or her
national law .
(B). If Harry hires you as his lawyer, what
legal recourse would you advise him to
take? Why? (2%)
SUGGESTED ANSWER:
I will advice Harry to:

(1) Dissolve

and

liquidate

his

property

relations with Wilma ; and

(2) If he will remarry, file a petition for the


recognition
foreign

and

enforcement

judgment

of

dbaratbateladot@gmail.com

Harry legally marry Elizabeth? Explain.

divorced

of

the
(Rule

39,Rules of Court ).

(C). Harry tells you that he has fallen in


love with another woman, Elizabeth, and
wants to marry her because, after all,
Wilma is already married to Joseph. Can

(2%)

SUGGESTED ANSWER :

In Republic v. Obrecido, however, the


Supreme Court ruled that a Filipino

Yes, he can validly marry Elizabeth,

spouse is given the capacity to remarry

applying the doctrine laid down by the

even though the spouse who obtained

Supreme Court in Republic v. Obrecido

the divorce was a Filipino at the time of

(427 SCRA 114 [2005]). Under the second

the marriage, if the latter was already a

paragraph of Article 26 of the Family

foreigner when the divorce was already

Code, for the Filipino spouse to have

obtained abroad. According to the court,

capacity to remarry, the law expressly

to rule otherwise will violate the equal

requires the spouse who obtained the

protection clause of the Constitution.

divorce to be a foreigner at the time of


the marriage. Applying this requirement
to the case of Harry it would seem that
he is not given the capacity to remarry.
This is because Wilma was a Filipino at
the time of her marriage to Harry.

Marriage;

Divorce

Decrees;

Foreign

Spouse Divorces Filipino Spouse (2012)


(b) Cipriano and Lady Miros married each
other. Lady Miros then left for the US and
there, she obtained American citizenship.

Never Let The Odds Keep You From Pursuing What You Know In Your Heart You Were Meant To Do.Leroy Satchel Paige

Page 24 of 180

Civil Law Q&As (2007-2013)

Cipriano

later

learned

hectorchristopher@yahoo.com

all

about

this

including the fact that Lady Miros has


divorced him in America and that she had
remarried there. He then filed a petition for
authority to remarry, invoking Par. 2, Art.
26

of

the

Family

Code.

Is

Cipriano

capacitated to re-marry by virtue of the


divorce decree obtained by his Filipino
spouse who was later naturalized as an
American citizen? Explain. (5%)
SUGGESTED ANSWER:
Yes, he is capacitated to remarry. While
the second paragraph of Art 26 of the
Family Code is applicable only to a
Filipino who married a foreigner at the
time of marriage, the Supreme Court
ruled in the case of Republic v. Orbecido,
G.R. No. 154380, 5 Oct, 2005, that the
said

provision

equally

applies

to

Filipino who married another Filipino at


the time of the marriage, but who was
already a foreigner when the divorce was
obtained.

Marriage;

Divorce

dbaratbateladot@gmail.com

when a foreign spouse divorces his/her

Decrees;

Foreign

Spouse Divorces Filipino Spouse (2010)


No.I. True or False.
(A). Under Article 26 of the Family Code,

Filipino spouse, the latter may re-marry by

that the decree of divorce obtained


proving only that the foreign spouse has

abroad

by

the

obtained a divorce against her or him

sufficient

to

abroad. (1%)

spouse to remarry.

foreigner

capacitate

spouse

the

is

Filipino

SUGGESTED ANSWER :
FALSE, In Garcia v. Recio , 366 SCRA

Marriage; Legal Separation; Prescription

437 (2001) , the SC held that for a

(2012)

Filipino

spouse

to

have

capacity

to

contract a subsequent marriage, it must


also be proven that the foreign divorced
obtained abroad by the foreigner spouse
give such foreigner spouse capacity to
remarry.
ALTERNATIVE ANSWER:

No.IV.a)

After

they

got

married,

Nikki

discovered that Christian was having an


affair with another woman. But Nikki
decided to give it a try and lived with him
for two (2) years. After two (2) years, Nikki
filed an action for legal separation on the
ground of Christians sexual infidelity. Will
the action prosper? Explain. (5%)

TRUE, Art 26 (2) (FC), clearly provides

Never Let The Odds Keep You From Pursuing What You Know In Your Heart You Were Meant To Do.Leroy Satchel Paige

Page 25 of 180

Civil Law Q&As (2007-2013)

hectorchristopher@yahoo.com

occurrence

Although the action for legal separation


has not yet prescribed, the prescriptive
period being 5 years, if Obecidos affair
with another woman was ended when
Nikki decided to live with him again,
action

account

of

will

sexual

infidelity

committed in 2002 runs from 2002, for

SUGGESTED ANSWER:

Nikkis

of

dbaratbateladot@gmail.com

not

condonation.

prosper

on

However,

if

such affair is still continuing, Nikkis


action would prosper because the action
will surely be within five (5) years from
the commission of the latest act of
sexual infidelity. Every act of sexual
liaison is a ground for legal separation.

Marriage; Legal Separation; Prescription


(2007)
No.VII. Write "TRUE" if the statement is
true or "FALSE" if the statement is false. If
the statement is FALSE, state the reason.
(2% each).
(2). If a man commits several acts of sexual
infidelity, particularly in 2002, 2003, 2004,
2005, the prescriptive period to file for legal
separation runs from 2002.
SUGGESTED ANSWER:
FALSE. The five-year prescriptive period
for filing legal separation runs from the

obligations of marriage because Neil is a


the sexual infidelity committed in 2003,
the prescriptive period runs from 2003
and

so

on.

The

action

for

legal

separation for the last act of sexual

drunkard, a womanizer, a gambler, and a


mama's boy- traits that she never knew or
saw when Neil was courting her. Although
summoned, Neil did not answer Maria's
petition and never appeared in court.

infidelity in 2005 will prescribe in 2010.


To support her petition, Maria presented
three witnesses- herself, Dr. Elsie Chan,
Marriage;

Psychological

Incapacity

(2013)
No.I. You are a Family Court judge and
before you is a Petition for the Declaration
of Nullity of Marriage (under Article 36 of
the Family Code)filed by Maria against Neil.
Maria claims that Neil is psychologically
incapacitated to comply with the essential

and Ambrosia. Dr. Chan testified on the


psychological

report

on

Neil

that

she

prepared. Since Neil never acknowledged


n9r

responded

to

her

invitation

for

interviews, her report is solely based on her


interviews with Maria and the spouses'
minor children. Dr. Chan concluded that
Neil

is

suffering

from

Narcissistic

Personality Disorder, an ailment that she


found to be already present since Neil's

Never Let The Odds Keep You From Pursuing What You Know In Your Heart You Were Meant To Do.Leroy Satchel Paige

Page 26 of 180

Civil Law Q&As (2007-2013)

hectorchristopher@yahoo.com

serious enough to prevent Neil from

early adulthood and one that is grave and


incurable. Maria testified on the specific
instances when she found Neil drunk, with
another

woman,

or

squandering

the

family's resources in a casino. Ambrosia,


the

spouses'

current

household

help,

corroborated Maria's testimony.


On the basis of the evidence presented, will
you grant the petition? (8%)
SUGGESTED ANSWER:
No. The petition should be denied.
The psychological incapacity under Art.
36

of

the

Family

Code

must

be

characterized by (a) gravity, (b) juridical


antecedence, and (c) incurability. It is
not enough to prove that the parties
failed to meet their responsibilities and
duties as married persons; it is essential
that they must be shown to be incapable
of doing so, due to some physiological
(not physical) illness (Republic v. CA and
Molina, G.R. No. 108763, Feb 13, 1997).
In this case, the pieces of evidence
presented are not sufficient to conclude
that

indeed

Neil

psychological
Personality
before

the

is

suffering

incapacity

Disorder]
marriage,

dbaratbateladot@gmail.com

from

[Narcissistic

existing

already

incurable

and

performing

his

essential

marital

obligations. Dr. Chans report contains

mere conclusions. Being a drunkard, a


womanizer, a gambler and a mamas boy,
merely shows Neils failure to perform
his marital obligations. In a number of
cases, the Supreme Court did not find
the existence of psychological incapacity
in cases where the respondent showed
habitual

drunkenness

(Republic

v.

Melgar, G.R. No. 139676, 2006), blatant


display of infidelity and irresponsibility
(Dedel v. CA, 2004) or being hooked to
gambling and drugs (Republic v. TanyagSan Jose, G.R. No. 168328, 2007).
ALTERNATIVE ANSWER:

Yes. The petition should be granted.


The personal medical or psychological
examination

of

requirement

respondent is
for

psychological

not

declaration

incapacity.

It

is

a
of

the

totality of the evidence presented which


shall

determine

psychological

the

incapacity

existence

of

(Marcos

v.

Marcos, G.R. No. 136490, Oct 19, 2000).


Dr. Chans report corroborated by Marias
and Ambrosias testimonies, therefore,
sufficiently prove Neils psychological
incapacity

to

assume

his

marital

obligations.

Never Let The Odds Keep You From Pursuing What You Know In Your Heart You Were Meant To Do.Leroy Satchel Paige

Page 27 of 180

Civil Law Q&As (2007-2013)

Marriage;

hectorchristopher@yahoo.com

cannot be annulled on the ground of

Psychological

Incapacity

(2012)
No.II.b) The petitioner filed a petition for
declaration of nullity of marriage based
allegedly on the psychological incapacity of
the respondent, but the psychologist was
not

able

to

personally

examine

the

respondent and the psychological report


was

based

petitioner.

only

on

Should

the

the

narration

annulment

of
be

granted? Explain. (5%)


SUGGESTED ANSWER:
The annulment cannot be guaranteed
solely on the basis of the psychological
report. For the report to prove the
psychological
respondent,

it

incapacity

of

the

is

that

the

required

psychologist should personally examine


the respondent and the psychological
report

should

be

based

on

the

psychologists independent assessment


of the facts as to whether or not the
respondent

is

psychologically

incapacitated.
Since,

the

dbaratbateladot@gmail.com

psychologist

did

not

personally examine the respondent, and


his report is based solely on the story of
the petitioner who has an interest in the
outcome of the petition, the marriage

respondents psychological incapacity if

When Faye was 25 years old, Brad discovered

the said report is the only evidence of


respondents psychological incapacity.

her continued liason with Roderick and in one


of their heated arguments, Faye shot Brad to
death. She lost no time in marrying her true
love Roderick, without a marriage license,

Marriage; Requisites (2008)

claiming that they have been continuously


cohabiting for more than 5 years.

No. III. Roderick and Faye were high school


sweethearts. When Roderick was 18 and Faye,

(A). Was the marriage of Roderick and Faye

16 years old, they started to live together as

valid? (2%)

husband and wife without the benefit of


marriage. When Faye reached 18 years of age,
her parents forcibly took her back and
arranged for her marriage to Brad. Although
Faye lived with Brad after the marriage,
Roderick continued to regularly visit Faye
while Brad was away at work. During their
marriage, Faye gave birth to a baby girl, Laica.

SUGGESTED ANSWER:
No. The marriage of Roderick and Faye is
not valid. Art. 4, FC provides that the
absence of any of the essential or formal
requisites renders the marriage void ab
initio. However, no license shall be

Never Let The Odds Keep You From Pursuing What You Know In Your Heart You Were Meant To Do.Leroy Satchel Paige

Page 28 of 180

Civil Law Q&As (2007-2013)

hectorchristopher@yahoo.com

necessary for the marriage of a man and


a woman who have lived together as
husband and wife for at least 5 years and
without any legal impediment to marry
each other. In Republic v. Dayot, G.R.
No. 175581, 28 March 2008, reiterating
the doctrine in Nial v. Bayadog, G.R.
No. 133778, 14 March 2000, this fiveyear

period

exclusivity

is

and

characterized
continuity.

In

by
the

present case, the marriage of Roderick


and Faye cannot be considered as a
marriage

of

exceptional

character,

because there were 2 legal impediments


during their cohabitation: minority on
the part of Faye, during the first two
years of cohabitation; and, lack of legal
capacity, since Faye married Brad at the
age of 18. The absence of a marriage
license made the marriage of Faye and
Roderick void ab initio.

Marriage; Subsequent Marriage (2008)


No. I. Ana Rivera had a husband, a Filipino
citizen

like

her,

who

dbaratbateladot@gmail.com

a divorced German national born of a

was

among

the

passengers on board a commercial jet plane


which crashed in the Atlantic Ocean ten
(10) years earlier and had never been heard
of ever since. Believing that her husband
had died, Ana married Adolf Cruz Staedtler,

German

father

and

Filipino

residing in Stuttgart. To avoid being

mother

subsistence of his previous marriage


reqiured to submit the required certificate

provided that: (a) his prior spouse in the

of capacity to marry from the German

first marriage had been absent for four

Embassy in Manila, Adolf stated in the

consecutive years; (b) that the spouse

application for marriage license that he was

present has a well-founded belief that

a Filipino citizen. With the marriage license

the absent spouse was already dead, and

stating that Adolf was a Filipino, the couple

(C) present spouse instituted a summary

got married in a ceremony officiated by the


Parish Priest of Calamba, Laguna in a
beach in Nasugbu, Batangas, as the local
parish

priest

refused

to

solemnize

marriages except in his church. Is the

proceeding for the declaration of the


presumptive death of absent spouse.
Otherwise, the second marriage shall be
null and void. In the instant case, the
husband

marriage valid? Explain fully. (5%)

of

Ana

was

among

the

passengers on board a commercial jet


SUGGESTED ANSWER:

plane which crashed in the Atlantic

No. The marriage is not valid. Art. 41 FC


allows the present spouse to contract a
subsequent

marriage

during

the

Ocean.

The

body

of

the

deceased

husband was not recovered to confirm


his death. Thus, following Art. 41, Ana

Never Let The Odds Keep You From Pursuing What You Know In Your Heart You Were Meant To Do.Leroy Satchel Paige

Page 29 of 180

Civil Law Q&As (2007-2013)

should

have

first

hectorchristopher@yahoo.com

Yes. Jon and Jane can marry each other;

secured

judicial

declaration of his presumptive death


before she married Adolf. The absence of
the

said

judicial

declaration

incapacitated Ana from contracting her


second marriage, making it void ab
initio.

Marriage; Void Marriages; By Reason of


Public Policy (2008)
No.V. Despite several relationships with
different
unmarried.

women,
His

Andrew

first

remained

relationship

with

Brenda produced a daughter, Amy, now 30


years old. His second, with Carla, produced
two sons: Jon and Ryan. His third, with
Donna, bore him no children although
Elena has a daughter Jane, from a previous
relationship. His last, with Fe, produced no
biological

children

but

they

informally

adopted without court proceedings, Sandy's


now 13 years old, whom they consider as
their own. Sandy was orphaned as a baby
and was entrusted to them by the midwife
who attended to Sandy's birth. All the
children, including Amy, now live with
andrew in his house.
(D). Can Jon and Jane legally marry? (1%)
SUGGESTED ANSWER:

dbaratbateladot@gmail.com

Jon is an illegitimate child of Andrew

David and they got married when she was


while Jane is a child of Elena from a
previous

relationship.

Thus,

their

marriage is not one of the prohibited


marriages enumerated under Art. 38 of

20 years old. David had a son, Julian, with


his ex-girlfriend Sandra. Julian and Thelma
can get married.
SUGGESTED ANSWER:

the FC.
TRUE.

Julian

and

Thelma

can

get

married. Marriage between stepbrothers


and
Marriage; Void Marriages; By Reason of
Public Policy (2007)

stepsisters

are

not

among

the

marriages prohibited under the Family


Code.

No. VII. Write "TRUE" if the statement is


true or "FALSE" if the statement is false. If
the statement is FALSE, state the reason.

Marriage;

(2% each).

Relations (2009)

(5). Amor gave birth to Thelma when she


was 15 years old. Thereafter, Amor met

Void

Marriages;

Property

No. III. In December 2000, Michael and


Anna, after obtaining a valid marriage

Never Let The Odds Keep You From Pursuing What You Know In Your Heart You Were Meant To Do.Leroy Satchel Paige

Page 30 of 180

Civil Law Q&As (2007-2013)

hectorchristopher@yahoo.com

assumption that there was no

license, went to the Office of the Mayor of


Urbano, Bulacan, to get married. The
Mayor

was

not

there,

but

the

Mayors

secretary asked Michael and Anna and their


witnesses to fill up and sign the required
marriage contract forms. The secretary then
told them to wait, and went out to look for the
Mayor who was attending a wedding in a
neighboring municipality.

When the secretary caught up with the


Mayor at the wedding reception, she showed
him the marriage contract forms and told
him that the couple and their witnesses
were waiting in his office. The Mayor
forthwith signed all the copies of the
marriage

contract,

dbaratbateladot@gmail.com

gave

them

to

the

secretary who returned to the Mayors


office. She then gave copies of the marriage
contract to the parties, and told Michael
and Anna that they were already married.
Thereafter, the couple lived together as
husband and wife, and had three sons.
(C). What property regime governs the
properties acquired by the couple? Explain.
(2%)
SUGGESTED ANSWER:

The marriage being void, the property


relationship that governed their union is
special co-ownership under Article 147
of the Family Code. This is on the

neighboring municipality.

impediment for them to validity marry


each other.

When the secretary caught up with the


Mayor at the wedding reception, she showed
him the marriage contract forms and told

Marriage;

Void Marriages; Status of

Children (2009)

Anna, after obtaining a valid marriage


license, went to the Office of the Mayor of
Urbano, Bulacan, to get married. The
was

not

were waiting in his office. The Mayor


forthwith signed all the copies of the

No. III. In December 2000, Michael and

Mayor

him that the couple and their witnesses

there,

but

the

Mayors

secretary asked Michael and Anna and their


witnesses to fill up and sign the required
marriage contract forms. The secretary then
told them to wait, and went out to look for the
Mayor who was attending a wedding in a

marriage

contract,

gave

them

to

the

secretary who returned to the Mayors


office. She then gave copies of the marriage
contract to the parties, and told Michael
and Anna that they were already married.
Thereafter, the couple lived together as
husband and wife, and had three sons.
(A). Is the marriage of Michael and Anna
valid,

voidable,

or

void?

Explain

your

answer. (3%)

Never Let The Odds Keep You From Pursuing What You Know In Your Heart You Were Meant To Do.Leroy Satchel Paige

Page 31 of 180

Civil Law Q&As (2007-2013)

hectorchristopher@yahoo.com

lesson to like-minded Lotharios.

SUGGESTED ANSWER :

The marriage is void because the formal


requisite

of

marriage

ceremony

was

absent ( Art.3, F.C. 209, Family Code).


ALTERNATIVE ANSWER:
The

marriage

is

void

because

an

essential requisite was absent: consent


of

the

parties

freely

given

in

the

presence of the solemnizing officer (Art


.2, FC).
(B). What is the status of the three children
of Michael and Anna? Explain your answer.
(2%)
SUGGESTED ANSWER:
The children are illegitimate, having
been born outside a valid marriage.

Parental Authority; Illegitimate Minor


Child (2009)
No.XIV. Rodolfo, married to Sharon, had an
illicit affair with his secretary, Nanette, a
19-year old girl, and begot a baby girl,
Rona. Nanette sued Rodolfo for damages:
actual, for hospital and other medical
expenses

in

delivering

the

child

by

caesarean section; moral, claiming that


Rodolfo

promised

to

dbaratbateladot@gmail.com

fact, he was not; and exemplary, to teach a

marry

her,

representing that he was single when, in

and even though he is giving support for


(C). When Rona reaches seven (7) years old,

the child. To acquire custody over Rona,

she tells Rodolfo that she prefers to live

Rodolfo should first deprive Nanette of

with

off

parental authority if there is ground

financially than Nanette. If Rodolfo files an

under the law, and in a proper court

action for the custody of Rona, alleging that

proceedings. In the same action, the

he is Ronas choice as custodial parent, will

court may award custody of Rona to

the court grant Rodolfos petition? Why or

Rodolfo if it is for her best interest.

him,

because

he

is

better

why not? (2%)


SUGGESTED ANSWER:
No, because Rodolfo has no parental

Parental Authority; In Vitro Fertilization


(2010)

authority over Rona. He who has the


parental

authority

has

the

right

to

No.VI. Gigolo entered into an agreement

custody. Under the Family Code, the

with Majorette for her to carry in her womb

mother alone has parental authority over

his baby via in vitro fertilization. Gigolo

the illegitimate child. This is true even if


illegitimate father recognized the child

undertook to underwrite Majorettes prenatal expenses as well as those attendant

Never Let The Odds Keep You From Pursuing What You Know In Your Heart You Were Meant To Do.Leroy Satchel Paige

Page 32 of 180

Civil Law Q&As (2007-2013)

hectorchristopher@yahoo.com

to her delivery. Gigolo would thereafter pay


Majorette P2 million and, in return, she
would give custody of the baby to him.
After Majorette gives birth and delivers the
baby to Gigolo following her receipt of P2
million, she engages your services as her
lawyer to regain custody of the baby.
(C) Who of the two can exercise parental
authority over the child? Explain. (2.5%)
SUGGESTED ANSWER:
Majorette,

the

mother,

can

exercise

parental authority. Since the child was


born

out

of

wedlock,

dbaratbateladot@gmail.com

Venus. The couple acquired a residential lot

the

child

is

illegitimate and the mother has the


exclusive parental authority and custody
over the child.
ALTERNATIVE ANSWER:
Gigolo can exercise parental authority
over the child. Majorette has no blood
relation to the child. She is just a
carrier of the child.

Paternity & Filiation; Child Born Under a


Void Marriage (2010)
No.X. In 1997, B and G started living
together without the benefit of marriage.
The relationship produced one offspring,

Family
in Paraaque. After four (4) years or in
2001,

having

completed

her

4-year

Code).

While

Venus

was

legitimated by the subsequent marriage


of her parents, such legitimation was

college degree as a fulltime student, she

rendered

and

marriage was later on declared null and

contracted

marriage

without

license.
The marriage of B and G was, two years

void

ineffective

due

to

when

absence

of

the

said

marriage

license.

later, declared null and void due to the

Under Article 178 of the Family Code,

absence of a marriage license.

legitimation

(B). Is Venus legitimate, illegitimate, or


legitimated? Explain briefly. (3%)

subsequent

Venus is illegitimate. She was conceived


and born outside a valid marriage. Thus,
she is considered illegitimate (Art 165,

valid

take

place

marriage

by

between

parents. The annulment of a voidable


marriage

SUGGESTED ANSWER:

shall

shall

legitimation.
underscored

not

The
portion

affect

inclusion
in

the

the

of

the

Article

necessarily implies that the Article's


application

is

limited

to

voidable

marriages. It follows that when the

Never Let The Odds Keep You From Pursuing What You Know In Your Heart You Were Meant To Do.Leroy Satchel Paige

Page 33 of 180

Civil Law Q&As (2007-2013)

hectorchristopher@yahoo.com

presumed as the child of B under Article

subsequent marriage is null or void, the


legitimation must also be null and void.
In the present problem, the marriage
between B and G was not voidable but
void. Hence, Venus has remained an
illegitimate child.

Paternity

&

Filiation;

Impugning

Legitimacy (2010)
No.IV.

Spouses

offsprings.

Albeit

personality

and
they

differences,

begot

had
the

two

serious
spouses

continued to live under one roof. B begot a


son by another woman. G also begot a
daughter by another man.
(A). If G gives the surname of B to her
daughter by another man, what can B do to
protect their legitimate children's interests?
Explain. (5%)
SUGGESTED ANSWER:
B can impugn the status of G's daughter
by

another

daughter

on

man
the

as

dbaratbateladot@gmail.com

his

ground

legitimate
that

for

biological reason he could not have been


the father of the child, a fact that may
be proven by the DNA test. Having been
born during the marriage between B and
G, G's daughter by another man is

164 of the Family Code. In the same

legitimate daughter of B by G.
action to impugn, B can pray for the
correction of the status of the said
Paternity

daughter in her record of birth.

&

Fertilization;
(B). If B acquiesces to the use of his

Filiation;

In

Surrogate

Mothers

Vitro

Remedy to Regain Custody (2010)

surname by Gs daughter by another man,


what is/are the consequence/s? Explain.

No.VI. Gigolo entered into an agreement

(5%)

with Majorette for her to carry in her womb


his baby via in vitro fertilization. Gigolo

SUGGESTED ANSWER:

undertook to underwrite Majorettes pre-

If B acquiesces and does not file the


action to impugn the legitimacy of the
child within the prescriptive period for

natal expenses as well as those attendant


to her delivery. Gigolo would thereafter pay
Majorette P2 million and, in return, she
would give custody of the baby to him.

doing so in Article 170 of the Family


Code, G's daughter by another man shall

After Majorette gives birth and delivers the

be

baby to Gigolo following her receipt of P2

conclusively

presumed

as

the

Never Let The Odds Keep You From Pursuing What You Know In Your Heart You Were Meant To Do.Leroy Satchel Paige

Page 34 of 180

Civil Law Q&As (2007-2013)

hectorchristopher@yahoo.com

million, she engages your services as her


lawyer to regain custody of the baby.
(A) What legal action can you file on behalf
of Majorette? Explain. (2.5%)
SUGGESTED ANSWER:
As her lawyer, I can file a petition for
habeas corpus on behalf Majorette to
recover custody of her child. Since she is
the mother of the child that was born
out

of

wedlock,

she

dbaratbateladot@gmail.com

baby? Explain. (2.5%)

has

exclusive

parental authority and custody over the


child. Gigolo, therefore, has no right to
have custody of the child and his refusal
to give up custody will constitute illegal
detention for which habeas corpus is the
proper remedy.
ALTERNATIVE ANSWER:
The action to regain custody will not
prosper. In the first place Majorette
cannot regain custody of the baby. As
surrogate mother she merely carries the
child in her womb for its development.
The child is the child of the natural
parents- Gigolo and his partner. The
agreement between Gigolo and Majorette
is a valid agreement.
(B) Can Gigolo demand from Majorette the
return of the P2 million if he returns the

SUGGESTED ANSWER:

If Gigolo voluntarily recognized the child


No, he cannot. Both he and Majorette are
guilty of violating the provision of the
Anti-Child Abuse Law (RA7610) on child
trafficking. Being in pari delicto, the
partners shall be left where they are and
Gigolo cannot demand the return of what
he paid.
ALTERNATIVE ANSWER:

as his illegitimate child in accordance


with Article 175 in relation to Article
172 of the Family Code, the child is
entitled to support and inheritance from
Gigolo.
ALTERNATIVE ANSWER:
Yes, because Gigolo is the natural and
biological parent of the baby.

Yes. The agreement between Gigolo and


Majorette is a valid agreement.
Paternity
(D) Is the child entitled to support and
inheritance from Gigolo? Explain. (2.5%)
SUGGESTED ANSWER:

&

Filiation;

Legitimacy;

Presumption (2008)
No. III. Roderick and Faye were high school
sweethearts. When Roderick was 18 and
Faye, 16 years old, they started to live
together as husband and wife without the

Never Let The Odds Keep You From Pursuing What You Know In Your Heart You Were Meant To Do.Leroy Satchel Paige

Page 35 of 180

Civil Law Q&As (2007-2013)

hectorchristopher@yahoo.com

Tanhoti-Liyao, G.R. No. 138961, 07

benefit of marriage. When Faye reached 18


years of age, her parents forcibly took her
back and arranged for her marriage to Brad.
Although Faye lived with Brad after the
marriage, Roderick continued to regularly
visit Faye while Brad was away at work.
During their marriage, Faye gave birth to a
baby girl, Laica. When Faye was 25 years old,
Brad discovered her continued liason with
Roderick

and

in

one

of

their

heated

arguments, Faye shot Brad to death. She lost


no time in marrying her true love Roderick,
without a marriage license, claiming that they
have been continuosly cohabiting for more
than 5 years.

(B). What is the filiation status of Laica?


(2%)
SUGGESTED ANSWER:
Laica

is

legitimate

because

children

conceived or born during the marriage of


the

parents

are

dbaratbateladot@gmail.com

presumed

to

be

legitimate (Art. 164, FC).


(C).Can Laica bring an action to impugn her
own status on the ground that based on
DNA results, Roderick is her biological
father? (2%)
SUGGESTED ANSWER:
No. Laica cannot bring an action to
impugn her own status. In Liyao Jr. v.

No. Laica cannot be legitimated by the


March 2002, the Supreme Court ruled
that impugning the legitimacy of the
child is a strictly personal right of
husband, except: (a) when the husband
died before the expiration of the period
fixed for bringing the action; (b) if he
should

die

complaint,

after

the

without

filing

having

of

the

marriage

of

her

biological

parents

because only children conceived and


born outside of wedlock of parents who
at the time of the conception of the
former were not disqualified by any
impediment to marry each other may be
legitimated (Art. 177, FC).

desisted

therefrom, or (c) if the child was born


after the death of the husband. Laica's
case does not fall under any of the
exceptions.
(D).

Can

Laica

Paternity & Filiation; Legitimation of a


Child from a Previous Valid Marriage
(2008)

be

legitimated

by

marriage of her biological parents? (1%)

the

No. IV. Gianna was born to Andy and


Aimee, who at the time Gianna's birth were
not married to each other. While Andy was

SUGGESTED ANSWER:

single at the time, Aimee was still in the

Never Let The Odds Keep You From Pursuing What You Know In Your Heart You Were Meant To Do.Leroy Satchel Paige

Page 36 of 180

Civil Law Q&As (2007-2013)

hectorchristopher@yahoo.com

dbaratbateladot@gmail.com

No. I. TRUE or FALSE. Answer TRUE if the


process of securing a judicial declaration of
nullity on her marriage to her ex-husband.
Gianna's birth certificate, which was signed
by both Andy and Aimee, registered the
status

of

Gianna

as

"legitimate",

her

surname carrying that of Andy's and that


her parents were married to each other.
(C). Assuming that Aimee is successful in
declaring her former marriage void, and
Andy and Aimee subsequently married each
other, would Gianna be legitimated? (1%)
SUGGESTED ANSWER:
Gianna cannot be legitimated by the
subsequent marriage of Andy and Aimee.
Art. 177 of the FC provides that "only
children conceived and born outside of
wedlock of parents who, at the time of
the conception of the former, were not
disqualified by any impediment to marry
each other may be legitimated." In the
present case, a legal impediment was
existing at the time of the conception of
Gianna. Her mother, Aimee, was still
alive in the process of securing judicial
declaration of nullity on her marriage to
her ex-husband.

Paternity & Filiation; Legitimation of a


Dead Child (2009)

statement is true, or FALSE if the

child's ascendants ,or other relatives .


statement is false. Explain your answer in
not more than two (2) sentences.

Paternity

(E). A dead child can be legitimated. (1%)

&

Filiation;

Support:

Ascendants & Descendants; Collateral


Blood Relatives (2008)

SUGGESTED ANSWER:
No.V. Despite several relationships with
TRUE To be legitimated, the law does not

different

require a child to be alive at the same

unmarried.

time of the marriage of his / her parents

Brenda produced a daughter, Amy, now 30

( Article 177, FC ). Furthermore, Art.

years old. His second, with Carla, produced

181 of the Family Code which states


that [Th]e llegitimation of children who
died before the celebration of marriage
will benefit their descendants, does not
preclude

instances

where

such

legitimation will benefit no one but the

women,
His

Andrew

first

remained

relationship

with

two sons: Jon and Ryan. His third, with


Donna, bore him no children although
Elena has a daughter Jane, from a previous
relationship. His last, with Fe, produced no
biological

children

but

they

informally

adopted without court proceedings, Sandy's


now 13 years old, whom they consider as

Never Let The Odds Keep You From Pursuing What You Know In Your Heart You Were Meant To Do.Leroy Satchel Paige

Page 37 of 180

Civil Law Q&As (2007-2013)

hectorchristopher@yahoo.com

dbaratbateladot@gmail.com

their own. Sandy was orphaned as a baby

Paternity & Filiation; Use of Surname;

and was entrusted to them by the midwife

Illegitimate Child (2009)

who attended to Sandy's birth. All the


children, including Amy, now live with

No.XIV. Rodolfo, married to Sharon, had an

andrew in his house.

illicit affair with his secretary, Nanette, a


19-year old girl, and begot a baby girl,

(B). In his old age, can Andrew be legally

Rona. Nanette sued Rodolfo for damages:

entitled to claim support from Amy, Jon,

actual, for hospital and other medical

Ryan, Vina, Wilma, and Sandy assuming

expenses

that all of them have the means to support

caesarean section; moral, claiming that

him? (1%)

Rodolfo

in

delivering

promised

to

the

child

marry

by

her,

representing that he was single when, in


SUGGESTED ANSWER:
Andrew, in his old age, cannot be legally

fact, he was not; and exemplary, to teach a


lesson to like-minded Lotharios.

entitled to claim support because Art.

(B). Suppose Rodolfo later on acknowledges

195, par 2 of the FC limits the giving of

Rona and gives her regular support, can he

support to "legitimate ascendants and

compel her to use his surname? Why or

descendants."

why not? (2%)

(C). Can Amy, Jon, Ryan, Vina, Wilma, and

SUGGESTED ANSWER:

Sandy legally claim support from each

No. he has no right to compel Rona to

other? (2%)

use his surname. The law does not give

SUGGESTED ANSWER:
Amy, Jon, Ryan, Vina, Wilma and Sandy
cannot legally claim support from each
other because Art. 195, par 5 limits the
giving of support to "legitimate brothers
and sisters, whether full or half blood."

him the right simply because he gave her


support (RA 9255).

Under the Family Code, an illegitimate


child was required to use only the
surname of the mother. Under RA 9255,
otherwise known as the Revilla law,
however, the illegitimate child is given
the option to use the surname of the
illegitimate father when the latter has

recognized the former in accordance

illegitimate child, Rodolfo cannot

with law. Since the choice belongs to the


Never Let The Odds Keep You From Pursuing What You Know In Your Heart You Were Meant To Do.Leroy Satchel Paige

Page 38 of 180

Civil Law Q&As (2007-2013)

hectorchristopher@yahoo.com

No,

compel Rona, if already of age, to use


the surname against her will. If Rona is
still a minor, to use the surname of
Rodolfo

will

require

the

consent

of

Rona's mother who has sole parental


authority over her.

Paternity & Filiation; Who May Impugn


Legitimacy (2009)
No.V.

Four

children,

namely:

Alberto,

Baldomero, Caridad, and Dioscoro, were


born to the spouses Conrado and Clarita de
la Costa. The childrens birth certificates
were duly signed by Conrado, showing them
to be the couples legitimate children.
Later, one Edilberto de la Cruz executed a
notarial document acknowledging Alberto
and Baldomero as his illegitimate children
>with

Clarita.

Edilberto

died

leaving

substantial properties. In the settlement of


his

estate,

Alberto

intervened

claiming

deceaseds

illegitimate

and
shares

Baldomero
as

the

children.

The

legitimate family of Edilberto opposed the


claim.
Are Alberto and Baldomero entitled to share
in the estate of Edilberto? Explain. (4%)
SUGGESTED ANSWER:

Alberto

and

dbaratbateladot@gmail.com

Baldomero

are

entitled to share in Edilbertos estate.

not

illegitimate children. Before they can be


They are not related at all to Edilberto.
They were born during the marriage of
Conrado

and

Clarita,

hence,

are

considered legitimate children of the


said spouses. This status is conferred on
them at birth by law.

conferred
illegitimate
first

the

status

children,

impugn

their

of

Edilbertos

Condrado

must

legitimacy.

Since

Condrado has not initiated any action to


impugn their legitimacy, they continue
to be the legitimate of Condrado. They
cannot be the illegitimate children of

Under Philippine law, a person cannot

Edilberto at the same time. Not being

have more than one natural filiation.

the illegitimate children of Edilberto,

The legitimate filiation of a person can

they have no right to inherit from him.

be changed only if the legitimate father


will successfully impugn such status.

In the problem, therefore, the filiation of


Alberto and Baldomero as legitimate
children of Condrado cannot be changed

Property

Relations;

Adulterous

Relationship (2009)
No. XI. TRUE or FALSE. Answer TRUE if
the statement is true, or FALSE if the

by their recognition by Edilberto as his


Never Let The Odds Keep You From Pursuing What You Know In Your Heart You Were Meant To Do.Leroy Satchel Paige

Page 39 of 180

Civil Law Q&As (2007-2013)

hectorchristopher@yahoo.com

statement is false. Explain your answer in


not more than two (2) sentences.
(B). If there is no marriage settlement, the
salary of a "spouse" in an adulterous
marriage

belongs

to

the

conjugal

partnership of gains. (1%)


SUGGESTED ANSWER:
False. In adulterous relationship, the
salary of a married partner belongs to
the absolute community, or conjugal
partnership, of such married partner
with his or her lawful spouse. Under
Articles 148 of the Family Code, the
property
partner

relations
and

governed

between

his/her

by

ordinary

married

paramour

is

co-ownership

where the partners become co-owners


only

when

they

acquisition
paramour

of
is

contributed
the

deemed

dbaratbateladot@gmail.com

building conjugal or paraphernal? Reasons.

to

property.
to

have

the
The
not

contributed in the earning of the salary


of the married partner.

Property Relations; Accession (2012)


No.III.(a) Maria, wife of Pedro, withdrew P 5
Million from their conjugal funds. With this
money, she constructed a building on a lot
which she inherited from her father. Is the

(5%)

Note: The rule on reverse accession is


SUGGESTED ANSWER:

applicable only to the regime of conjugal

It depends. If the value of the building is


more than the value of the land, the
building

is

conjugal

and

the

land

becomes conjugal property under Art.

partnership of gains in both the Family


Code and the New Civil Code. The foregoing
answer assumes that CPG is the regime of
the property relations of the spouses.

120 of the Family Code. This is a case of


reverse accession, where the building is
considered as the principal and the land,
the accessory. If, on the other hand, the

Property Relations; Ante-Nuptial Debt


(2007)

value of the land is more than the value

No. VII. Write "TRUE" if the statement is

of the building, then the ordinary rule of

true or "FALSE" if the statement is false. If

accession applies where the land is the

the statement is FALSE, state the reason.

principal

and

(2% each).

accessory.

In

the
such

building,
case,

the

the
land

remains paraphernal property and the

(3). An individual, While single, purchases a


house and lot in 1990 and borrows money

building becomes paraphernal propery.


Never Let The Odds Keep You From Pursuing What You Know In Your Heart You Were Meant To Do.Leroy Satchel Paige

Page 40 of 180

Civil Law Q&As (2007-2013)

in

1992

to

repair

it.

hectorchristopher@yahoo.com

In

1995,

such

individual gets married while the debt is

Property

dbaratbateladot@gmail.com

Relations;

Unions

Without

Marriage (2012)

still being paid. After the marriage, the debt


is still the responsibility of such individual.

No.V. a) Spouses Primo and Monina Lim,


childless, were entrusted with the custody

SUGGESTED ANSWER:

of two (2) minor children, the parents of


whom were unknown. Eager of having

FALSE.

The

absolute

Community

of

children of their own, the spouses made it

property is liable for the ante-nuptial

appear

debts of either spouse in so far as the

parents by naming them Michelle P. Lim

same redounded to the benefit of the

and Michael Jude Lim. Subsequently,

family (Art. 94 par.7, FC).

Monina married Angel Olario after Primos

that

they

were

the

childrens

death.
ALTERNATIVE ANSWER:
She decided to adopt the children by
FALSE.

The

the

availing the amnesty given under R.A. 8552

community

to those individuals who simulated the

property, because the property already

birth of a child. She filed separate petitions

constitutes

for the adoption of Michelle, then 25 years

responsibility

debt

is

of

the

absolute

already

community

property under Art. 91 of FC which took


effect in 1988 while the house and lot
here involved was purchased in 1990.

old and Michael, 18. Both Michelle and


Michael gave consent to the adoption.
The trial court dismissed the petition and

There is no indication that the spouse

ruled that Monina should have filed the

who bought the property had legitimate

petition jointly with her new husband.

descendants by a former marriage, which

Monina, in a Motion for Reconsideration

would exclude the house and lot from

argues that mere consent of her husband

the community property, Art. 92 par 3,

would suffice and that joint adoption is not

FC). If the spouses established a conjugal

needed,

partnership, the property belongs to the


individual spouse if full ownership was
vested before marriage (Art. 118, FC).

for

the

adoptees

are

already

emancipated.
Is the trial court correct in dismissing the
petitions for adoption? Explain. (5%)
SUGGESTED ANSWER:

Never Let The Odds Keep You From Pursuing What You Know In Your Heart You Were Meant To Do.Leroy Satchel Paige

Page 41 of 180

Civil Law Q&As (2007-2013)

hectorchristopher@yahoo.com

Yes, the trial court was correct. At the


time the petitions for adoptions were
filed, petitioner had already remarried.
Under the law, husband and wife shall
adopt

jointly,

except

in

the

cases

enumerated in the law. The adoption


cases of Michelle and James do not fall
in any of the exceptions provided in the
law where a spouse is permitted to adopt
alone.

Hence,

Monina

should

adopt

jointly with her husband Angel (Adoption


of Michelle P. Lim, G.R. Nos. 168992-93,
May 21, 2009).

Property

Relations;

Unions

Without

Marriage (2012)
No.V. b) Jambrich, an Austrian, fell in-love
and lived together with Descallar and
bought their houses and lots at Agro-Macro
Subdivision.

In

the

Contracts

to

Sell,

Jambrich and Descallar were referred to as


the buyers. When the Deed of Absolute Sale
was presented for registration before the
Register of Deeds, it was refused because
Jambrich was an alien and could not
acquire
domain.

alienable
After

lands

Jambrich

dbaratbateladot@gmail.com

rights and interests in the Agro-Macro

of

the

and

public

Descallar

separated, Jambrich purchased an engine


and some accessories for his boat from
Borromeo. To pay for his debt, he sold his

properties to Borromeo.

apply to their property relations and the


Borromeo discovered that titles to the three

properties in question are owned by

(3) lots have been transfereed in the name

them in equal shares even though all the

of Descallar. Who is the rightful owner of

funds used in acquiring the properties

the properties? Explain. (5%)

came only from the salaries or wages, or

SUGGESTED ANSWER:

the

income

of

Jambrich

from

his

business or profession. In such case,

It depends. On the assumption that the

while Jambrich is disqualified to own

Family Code is the applicable law, the

any

ownership of the properties depends on

subsequent transfer of all his interest

whether or not, Jambrich and Descallar

therein to Borromeo, a Filipino, was

are capacitated to marry each other

valid as it removed the disqualification.

during their cohabitation, and whether

In such case, the properties are owned

or not both have contributed funds for

by Borromeo and Descallar in equal

the acquisition of the properties.

shares.

If both of them are capacitated to marry

If, on the other hand, Jambrich and

each other, Art 147- co-ownership will

Descallar were not capacitated to marry

part

of

the

properties,

his

Never Let The Odds Keep You From Pursuing What You Know In Your Heart You Were Meant To Do.Leroy Satchel Paige

Page 42 of 180

Civil Law Q&As (2007-2013)

each

other

Art.

hectorchristopher@yahoo.com

148-co-ownership

governs their property relations. Under


this regime, Jambrich and Descallar are
co-owners of the properties but only if
both

of

them

contributed

in

their

acquisition. If all the funds used in


acquiring

the

properties

in

question

came from Jambrich, the entire property


is his even though he is disqualified from
owning it. His subsequent transfer to
Borromeo,

however,

is

valid

as

it

removed the disqualification. In such


case, all the properties are owned by
Borromeo.
Descallar

If,

on

the

other

hand

to

their

contributed

acquisition, the properties are co-owned


by Descallar and Borromeo in proportion
to the respective contributions of the
Descallar and Jambrich.
Note: The facts of the problem are not
exactly the same as in the case of
Borromeo v. Descallar, G.R. NO. 159310,
Feb 24, 2009, hence, the difference in
the resulting answer.

Property

Relations;

dbaratbateladot@gmail.com

ab initio under Article 36 of the Family

Void

Marriages

(2010)
No.VII. G and B were married on July 3,
1989. On March 4, 2001, the marriage,
which bore no offspring, was declared void

Code. At the time of the dissolution of the

SUGGESTED ANSWER:

marriage, the couple possessed the


following properties:

Since the marriage was declared void ab

a house and lot acquired by B on


August 3, 1988, one third (1/3) of
the

purchase

price

(representing

downpayment) of which he paid; one

or

Conjugal

Partnership

was

ever

established between B and G. Their


property relation is governed by a

G on

special co-ownership under Article 147

February 14, 1990 out of a cash gift

of the Family Code because they were

given to her by her parents on her

capacitated to marry each other.

third

(1/3) was paid by

graduation on April 6, 1989; and


the balance was paid out of the
spouses joint income; and

initio in 2001, no Absolute Community

an apartment unit donated to B by


an uncle on June 19, 1987.

Under

that

Article

147,

wages

and

salaries of the former spouses earned


during their cohabitation shall be owned
by them in equal shares while properties
acquired thru their work for industry

(B) Who

owns

Explain. (5%)

the

foregoing

properties?

shall be owned by them in proportion to

Never Let The Odds Keep You From Pursuing What You Know In Your Heart You Were Meant To Do.Leroy Satchel Paige

Page 43 of 180

Civil Law Q&As (2007-2013)

hectorchristopher@yahoo.com

during the cohabitation, hence, it is his

their respective contributions. Care and


maintenance of the family is recognized
as

valuable

contribution.

In

the

absence of proof as to the value of their


respective

contributions,

they

shall

share equally.
If ownership of the house and lot was
acquired by B on August 3, 1988 at the
time he bought it on installment before
he got married, he shall remain owner of
the house and lot but he must reimburse
G for all the amounts she advanced to
pay the purchase price and for one-half
share in the last payment from their
joint income. In such case, the house
and lot were not acquired during their
cohabitation, hence, are not co-owned by
B and G.
But if the ownership of the house and lot
was acquired during the cohabitation,
the house and lot will be owned as
follows:
(1) 1/3 of the house and lot is owned by B.
He is an undivided co-owner to that
extent

for

his

dbaratbateladot@gmail.com

contributions

in

its

acquisition in the form of the down


payment he made before the celebration
of the marriage. The money he used to
pay the down payment was not earned

exclusive property.
(2) 1/3 of the house and lot is owned by

G. She is an undivided co-owner to the


extent

for

her

contribution

in

its

acquisition when she paid 1/3 of the

After

summing

up

their

prospective

shares, B and G are undivided co-owners


of the house and lot in equal shares.

purchase price using the gift from her

As

parents. Although the gift was acquired

exclusive by B because he acquired it

by G during her cohabitation with B, it is

before their cohabitation. Even if he

her exclusive property. It did not consist

acquired it during their cohabitation, it

of wage or salary or fruit of her work or

will

industry.

because it did not come from his wage or

to

the

still

be

apartment,

his

it

exclusive

is

owned

property

salary, or from his work or industry. It


(3) 1/3 of the house is co-owned by B

was acquired gratuitously from his uncle.

and G because the payment came from


their co-owned funds, i.e., their joint

(B) If G and B had married on July 3, 1987

income during their cohabitation which

and their marriage was dissolved in 2007,

is shared by them equally in the absence


of any proof to the contrary.

who owns the properties? Explain. (5%)


SUGGESTED ANSWER:

Never Let The Odds Keep You From Pursuing What You Know In Your Heart You Were Meant To Do.Leroy Satchel Paige

Page 44 of 180

Civil Law Q&As (2007-2013)

hectorchristopher@yahoo.com

The answer is the same as in letter A.


Since the parties to the marriage which
was later declared void ab initio were
capacitated to marry each other, the
applicable law under the New Civil Code
was

Article

144.This

Article

is

substantially the same as Article 147 of


the Family Code.
Hence, the determination of ownership
will remain the same as in question A.
And

even

assuming

that

the

two

provisions are not the same, Article 147


of the Family Code is still the law that
will govern the property relations of B
and G because under Article 256, the
Family

Code

has

retroactive

effect

insofar as it does not prejudice or impair


vested or acquired rights under the new
Civil Code or other laws. Applying Article
147 retroactively to the case of G and B
will not impair any vested right. Until
the declaration of nullity of the marriage
under the Family Code, B and G have not
as yet acquired any vested right over the
properties

acquired

during

their

cohabitation.

Property
(2010)

Relations;

dbaratbateladot@gmail.com

No.X. In 1997, B and G started living

Void

Marriages

together without the benefit of marriage.


The relationship produced one offspring,
Venus. The couple acquired a residential lot

and G. Their properties are governed by


in Paraaque. After four (4) years or in

the special co-ownership provision of

4-year

Article 147 of the Family Code because

college degree as a fulltime student, she

both B and G were capacitated to marry

and

each other. The said Article provides

2001,

having

contracted

completed

marriage

her

without

license.

that when a man and a woman who are

The marriage of B and G was, two years


later, declared null and void due to the

capacitated to marry each other, live


exclusively with each other as husband
and wife without the benefit of marriage,

absence of a marriage license.

or under a void marriage: (1) their wages


(A). If you were the judge who declared the

and salaries shall be owned by them in

nullity of the marriage, to whom would you

equal shares; and (2) property acquired

award the lot? Explain briefly. (3%)

by both of them through their work or


industry shall be governed by the rules

SUGGESTED ANSWER:

on co-ownership. In co-ownership, the


Since the marriage was null and void, no
Absolute

Community

or

Conjugal

parties are co-owners if they contributed


something of value in the acquisition of

Partnership was established between B


Never Let The Odds Keep You From Pursuing What You Know In Your Heart You Were Meant To Do.Leroy Satchel Paige

Page 45 of 180

Civil Law Q&As (2007-2013)

the

property.

proportion

Their

to

contributions.

hectorchristopher@yahoo.com

share

their

In

an

is

respective
ordinary

co-

the family is not recognized as a valuable


contribution for the acquisition of a
property. In the Article 147 special cohowever,

Succession

in

ownership the care and maintenance of

ownership

dbaratbateladot@gmail.com

care

and

Disposition; Mortis Causa vs. Intervivos;


Corpse (2009)
No. XI. TRUE or FALSE. Answer TRUE if
the statement is true, or FALSE if the
statement is false. Explain your answer in
not more than two (2) sentences.

maintenance is recognized as a valuable


contribution
contributor

which
to

half

will
of

entitle
the

the

property

acquired.
Having

(E). A person can dispose of his corpse


through an act intervivos. (1%)
SUGGESTED ANSWER:

been

acquired

during

their

False. A persons cannot dispose of his

is

corpse through an act inter vivos, i.e.,

presumed acquired through their joint

an act to take effect during his lifetime.

work and industr under Article 147,

Before his death there is no corpse to

hence, B and G are co-owners of the said

dispose. But he is allowed to do so

property in equal shares.

through an act mortis causa, i.e., an act

cohabitation,

the

residential

lot

Article 147 also provides that when a

to take effect upon his death.

party to the void marriage was in bad


faith, he forfeits his share in the coownership

in

favor

of

the

common

children or descendants, the default of

Heirs;

Fideicommissary

Substitution

(2008)

children or descendants, the forfeited

No. XIII. Raymond, single, named his sister

share shall belong to the innocent party.

Ruffa in his will as a devisee of a parcel of

In the foregoing problem, there is no

land which he owned. The will imposed

showing that one party was in bad faith.

upon Ruffa the obligation of preseving the

Hence, both shall be presumed in good

land and transferring it, upon her death, to

faith and no forfeiture shall take place.

her illegitimate daughter Scarlet who was


then only one year old. Raymond later died,

leaving behind his widowed mother, Ruffa

and Scarlet.

Never Let The Odds Keep You From Pursuing What You Know In Your Heart You Were Meant To Do.Leroy Satchel Paige

Page 46 of 180

Civil Law Q&As (2007-2013)

hectorchristopher@yahoo.com

illegitimate child, cannot inherit the

(A). Is the condition imposed upon Ruffa, to


preserve the property and to transmit it
upon her death to Scarlet, valid? (1%)
SUGGESTED ANSWER:
Yes, the condition imposed upon Ruffa
to preserve the property and to transmit
it upon her death to Scarlet is valid
because

it

is

tantamount

to

fideicommissary substitution under Art.


863 of the Civil Code.
(B).

If Scarlet

predeceases

Ruffa, who

inherits the property? (2%)


SUGGESTED ANSWER:
Ruffa

will

inherit

the

property

as

Scarlet's heir. Scarlet acquires a right to


the

succession

Raymond's

from

death,

the

even

time

though

of
she

should predecease Ruffa (Art. 866, Civil


Code).
(C). If Ruffa predeceases Raymond, can
Scarlet inherit the property directly from
Raymond? (2%)
SUGGESTED ANSWER:
If

Ruffa

Raymond's

predeceases
widowed

dbaratbateladot@gmail.com

mother

Raymond,
will

be

entitled to the inheritance. Scarlet, an

property by intestate succession from


Raymond who is a legitimate relative of

each
Ruffa (Art. 992, Civil Code). Moreover,
Scarlet is not a compulsory heir of

of

their

children

as

their

presumptive legitimes.
Peter later re-married and had two (2)

Raymond, hence she can inherit only by

children by his second wife Marie. Peter and

testamentary

Marie,

succession.

Since

having

successfully

engaged

in

Raymond executed a will in the case at

business, acquired real properties. Peter

bar, Scarlet may inherit from Raymond.

later died intestate.


(A). Who are Peters legal heirs and how will
his estate be divided among them? (5%)

Heirs; Intestate Succession; Legitime;


Computation (2010)

SUGGESTED ANSWER:

No.XI. The spouses Peter and Paula had

The legal heirs of Peter are his children

three (3) children. Paula later obtained a

by the first and second marriages and

judgment of nullity of marriage. Their

his surviving second wife.

absolute community of property having


been dissolved, they delivered P1 million to

Their shares in the estate of Peter will


depend, however, on the cause of the

Never Let The Odds Keep You From Pursuing What You Know In Your Heart You Were Meant To Do.Leroy Satchel Paige

Page 47 of 180

Civil Law Q&As (2007-2013)

hectorchristopher@yahoo.com

Surviving
nullity of the first marriage. If the
nullity

of

the

first

marriage

was

psychological incapacity of one or both


spouses, the three children of that void
marriage are legitimate and all of the
legal heirs shall share the estate of Peter
in equal shares. If the judgment of
nullity was for other causes, the three
children are illegitimate and the estate
shall

be

distributed

such

that

an

illegitimate child of the first marriage


shall receive half of the share of a
legitimate child of the second marriage,
and the second wife will inherit a share
equal to that of a legitimate child. In no
case may the two legitimate children of
the second marriage receive a share less
than one-half of the estate which is their
legitime.

When

the

estate

is

not

sufficient to pay all the legitimes of the


compulsory heirs, the legitime of the
spouse is preferred and the illegitimate
children suffer the reduction.
Computation:
(A)

If

the

ground

of

nullity

is

psychological incapacity:
3

children

marriage
2

by first 1/6 of the estate


for each

children by second 1/6 of the estate

marriage

for each

spouse

dbaratbateladot@gmail.com

second 1/6 of the estate

the
(B) If the ground of nullity is not
psychological capacity:
2

legitimate of the estate for

children

each

of

second

marriage
Surviving

illegitimate 1/12

children

of

is

left

after

paying

the

legitime of the surviving spouse which is


preferred.
Hence, the remaining of the estate
shall be divided among the 3 illegitimate
children.

of the estate

second spouse
3

estate

estate

for

each of first marriage

(B). What is the effect of the receipt by


Peters 3 children by his first marriage of
their presumptive legitimes on their right to
inherit following Peters death? (5%)

Note: The legitime of an illegitimate


child is supposed to be the legitime of

SUGGESTED ANSWER:

a legitimate child or 1/8 of the estate.

In the distribution of Peters estate, of

But the estate will not be sufficient to

the

pay

children of the first marriage shall be

the

said

legitime

of

the

illegitimate children, because only of

presumptive

received

by

the

collated to Peters estate and shall be

Never Let The Odds Keep You From Pursuing What You Know In Your Heart You Were Meant To Do.Leroy Satchel Paige

Page 48 of 180

Civil Law Q&As (2007-2013)

hectorchristopher@yahoo.com

dbaratbateladot@gmail.com

While Arnel is a legal heir of Franco, he


imputed

as

an

advance

of

their

respective inheritance from Peter. Only


half

of

the

presumptive

legitime

is

collated to the estate of Peter because


the other half shall be collated to the
estate of his first wife.

Heirs; Representation; Iron-Curtain Rule


(2012)
No.VIII.a) Ricky and Arlene are married.
They begot Franco during their marriage.
Franco had an illicit relationship with
Audrey and out of which, they begot Arnel.
Frnaco

predeceased

Ricky,

Arlene

and

Arnel. Before Ricky died, he executed a will


which when submitted to probate was
opposed by Arnel on the ground that he
should be given the share of his father,
Franco. Is the opposition of Arnel correct?
Why? (5%)
SUGGESTED ANSWER:
No, his opposition is not correct. Arnel
cannot

inherit

from

Ricky

in

the

representation of his father Franco. In


representation, the representative must
not only be a legal heir of the person he
is representing, he must also be a legal
heir of the decedent he seeks to inherit
from.

is not a legal heir of Ricky because under

(B).In reservatroncal, all reservatarios (reser


Art 992 of the NCC, an illegitimate child
has no right to inherit ab intestato from

vees) inherit as a class and in equal shares


regardless of their proximity in degree to
the prepositus. (1%)

the legitimate children and relatives of


his

father

disqualified

or
to

mother.
inherit

Arnel
from

is

SUGGESTED ANSWER:

Ricky

FALSE. Not all the relatives within the

because Arnel is an illegitimate child of

third degree will inherit as reservatario ,

Franco and Ricky is a legitimate relative

and not all those who are entitled to

of Franco.

inherit will inherit in the equal shares .


The

applicable

laws

of

intestate

succession will determine who among


Heirs; Reserva Troncal (2009)

the relatives will inherit as reservatarios

No. I. TRUE or FALSE. Answer TRUE if the


statement

is

true,

or

FALSE

if

the

statement is false. Explain your answer in


not more than two (2) sentences.

and what shares they will tak, i.e., the


direct line excludes the collateral, the
descending

direct

line

excludes

the

ascending ,the nearer excludes the more


remote, the nephews and nieces exclude

Never Let The Odds Keep You From Pursuing What You Know In Your Heart You Were Meant To Do.Leroy Satchel Paige

Page 49 of 180

Civil Law Q&As (2007-2013)

hectorchristopher@yahoo.com

half the share of a full-blood sibling.

the uncles and the aunts, and half blood


relatives inherit half the share of fullblooded relatives.

Intestate Succession (2008)


No. VII. Ramon Mayaman died intestate,
leaving a net estate of P10,000,000.00.
Determine how much each heir will receive
from the estate:
(A). If Ramon is survived by his wife, three
full-blood brothers, two half-brothers, and
one nephew (the son of a deceased fullblood brother)? Explain. (3%)
SUGGESTED ANSWER:
Having died intestate, the estate of
Ramon shall be inherited by his wife and
his full and half blood siblings or their
respective representatives. In intestacy,
if the wife concurs with no one but the
siblings of the husband, all of them are
the

intestate

heirs

of

the

deceased

husband. The wife will receive half of the


intestate estate, while the siblings or
their

respective

dbaratbateladot@gmail.com

representatives,

will

inherit the other half to be divided


among them equally. If some siblings are
of the full-blood and the other of the half
blood, a half blood sibling will receive

The wife will receive one half (1/2) of the


(1). The wife of Ramon will, therefore,
receive one half () of the estate or the

estate or P5,000,000.00. The other half


shall be inherited by (1) the full-blood
brother,

amount of P5,000,000.00.
(2). The three (3) full-blood brothers, will,

represented

by

his

three

children, and (2) the half-sister. They will

therefore, receive P1,000,000.00 each.

divide the other half between them such

(3).

that the share of the half-sister is just

The

nephew

P1,000,000.00

will

by

receive

right

of

half the share of the full-blood brother.


The share of the full-blood brother shall

representation.
(4). The two (2) half-brothers will receive

in

turn

be

inherited

by

the

three

nephews in equal shares by right of

P500,000.00 each.

presentation.
(B). If Ramon is survived by his wife, a halfsister,

and three

nephews

(sons

of a

deceased full-blood brother)? Explain. (3%)


SUGGESTED ANSWER:

Therefore, the three (3) nephews will


receive P1,111,111.10 each the halfsister

will

receive

the

sum

of

P1,666,666.60.

Never Let The Odds Keep You From Pursuing What You Know In Your Heart You Were Meant To Do.Leroy Satchel Paige

Page 50 of 180

Civil Law Q&As (2007-2013)

hectorchristopher@yahoo.com

Intestate Succession (2008)

dbaratbateladot@gmail.com

(D). How should the house and lot, and the


cash be distributed? (1%)

No.X. Arthur executed a will which contained


only: (i) a provision disinheriting his daughter

SUGGESTED ANSWER:

Bernica for running off with a married man,


and (ii) a provision disposing of his share in

Since the probate of the will cannot be

the family house and lot in favor of his other

allowed,

children Connie and Dora. He did not make

succession apply. Under Art. 996 of the

any provisions in favor of his wife Erica,

Civil Code, if a widow or widower and

because as the will stated, she would anyway

legitimate children or descendants are

get of the house and lot as her conjugal

left, the surviving spouse has the same

share.

share as of the children. Thus, ownership

The

will

was

very

brief

and

straightforward and both the above provisions


were contained in page 1, which Arthur and
his

instrumental

witness,

signed

at

the

bottom. Page 2 contained the attestation


clause and the signatures, at the bottom
thereof, of the 3 instrumental witnesses

the

rules

on

intestate

over the house and lot will be created


among

wife

Erica

and

her

children

Bernice, Connie and Dora. Similarly, the


amount of P 1 million will be equally
divided among them.

which included Lambert, the driver of Arthur;


Yoly, the family cook, and Attorney Zorba, the
lawyer who prepared the will. There was a 3rd

Intestate

page, but this only contained the notarial

Representation:

acknowledgement.

Child; Iron Curtain Rule (2007)

The

attestation

clause

Succession;

Rights

Illegitimate,

of

Adopted

stated the will was signed on the same


occasion by Arthur and his instrumental

No. X. For purpose of this question, assume

witnesses who all signed in the presence of

all formalities and procedural requirements

each

have been complied with.

other, and the

notary

public

who

notarized the will. There are no marginal


signatures or pagination appearing on any of
the 3 pages. Upon his death, it was discovered
that apart from the house and lot, he had a P
1 million account deposited with ABC bank.

In 1970, Ramon and Dessa got married.


Prior to their marriage, Ramon had a child,
Anna. In 1971 and 1972, Ramon and Dessa
legally

adopted

Cherry

and

Michelle

respectively. In 1973, Dessa died while


giving birth to Larry Anna had a child, Lia.

Anna never married. Cherry, on the other

hand, legally adopted Shelly. Larry had

Never Let The Odds Keep You From Pursuing What You Know In Your Heart You Were Meant To Do.Leroy Satchel Paige

Page 51 of 180

Civil Law Q&As (2007-2013)

hectorchristopher@yahoo.com

requires the representative to be a legal

twins, Hans and Gretel, with his girlfriend,


Fiona. In 2005, Anna, Larry and Cherry
died in a car accident. In 2007, Ramon
died. Who may inherit from Ramon and
who may not? Give your reason briefly.
(10%)
SUGGESTED ANSWER:
The following may inherit from Ramon:
(1). Michelle, as an adopted child of
Ramon, will inherit as a legitimate child
of Ramon. As an adopted child, Michelle
has all the rights of a legitimate child
(Sec 18, Domestic Adoption Law).
(2). Lia will inherit in representation of
Anna. Although Lia is an illegitimate
child, she is not barred by Articles 992,
because

her

mother

Anna

is

an

illegitimate herself. She will represent


Anna as regards Anna's legitime under
Art. 902, NCC and as regards Anna's
intestate share under Art. 990, NCC.
The following may not inherit from
Ramon:
(1). Shelly, being an adopted child, she
cannot represent Cherry. This is because
adoption creates a personal legal relation
only

between

adopted.

The

dbaratbateladot@gmail.com

the

adopter

law

on

and

the

representation

heir of the person he is representing and

time, adoption was governed by the New


also of the person from whom the person
being

represented

was

supposed

to

inherit. While Shelly is a legal heir of


Cherry, Shelly is not a legal heir of
Ramon.

Adoption

personal

legal

created

relation

only

purely
between

Cherry and Shelly.

Civil Code. Under the New Civil Code,


husband and wife were allowed to adopt
separately or not jointly with the other
spouse. And since the problem does not
specifically and categorically state, it is
possible to construe the use of the word
"respectively"

in

the

problem

as

indicative of the situation that Cherry


(2). Hans and Gretel are barred from

was

inheriting from Ramon under Art. 992,

Michelle was adopted by Dessa alone. In

NCC. Being illegitimate children, they

such

cannot inherit ab intestao from Ramon.

alternative answer to the problem will be

ALTERNATIVE ANSWER:
The problem expressly mentioned the
dates of the adoption of Cherry and
Michelle as 1971 and 1972. During that

adopted

case

of

by

Ramon

separate

alone

adoption

and

the

as follows: Only Lia will inherit from


Ramon in representation of Ramon's
illegitimate daughter Anna. Although Lia
is an illegitimate child, she is not barred
from inheriting from Ramon because her

Never Let The Odds Keep You From Pursuing What You Know In Your Heart You Were Meant To Do.Leroy Satchel Paige

Page 52 of 180

Civil Law Q&As (2007-2013)

hectorchristopher@yahoo.com

dbaratbateladot@gmail.com

No.VIII.b) How can RJP distribute his estate


mother is herself illegitimate. Shelly

by will, if his heirs are JCP, his wife; HBR

cannot

and RVC, his parents; and an illegitimate

inherit

in

representation

of

Cherry because Shelly is just an adopted


child of Cherry. In representation, the
representative must not only be a legal
heir of the person he is representing but
also of the decedent from whom the
represented

person

is

supposed

to

inherit. In the case of Shelly, while she


is a legal heir of Cherry by virtue of
adoption, she is not a legal heir of
Ramon.

Adoption

creates

personal

legal relation only between the adopting


parent and the adopted child (Teotico v.
Del Val, 13 SCRA 406, 1965. Michelle
cannot inherit from Ramon, because she
was adopted not by Ramon but by Dessa.
In the eyes of the law, she is not related
to Ramon at all. Hence, she is not a legal
heir of Ramon. Hans and Gretel are not
entitled to inherit from Ramon, because
they are barred by Art. 992 NCC. Being
illegitimate
cannot

children

inherit

from

of

Larry,

the

they

legitimate

relatives of their father Larry. Ramon is


a legitimate relative of Larry who is the
legitimate father.

Legitimes; Compulsory Heirs (2012)

child, SGO?

after working for so many years in the


Middle East. He had saved P100.000 in his

SUGGESTED ANSWER:

saving account in Manila which intended to

A testator may dispose of by will the free

use to start a business in his home

portion of his estate. Since the legitime

country. On his flight home, Ernesto had a

of JCP is 1/8 of the estate, SGO is of

fatal heart attack. He left behind his

the estate and that of HBR and RVC is

widowed mother, his common-law wife and

of the hereditary estate under Art 889 of


the NCC, the remaining 1/8 of the estate
is the free portion which the testator
may dispose of by will.

their twins sons. He left no will, no debts,


no other relatives and no other properties
except the money in his saving account.
Who are the heirs entitled to inherint from
him and how much should each receive?
(3%)

Legitime; Compulsory Heirs (2008)

SUGGESTED ANSWER:

No.

Filipino

The mother and twin sons are entitled to

worker, was coming home to the Philippines

inherit from Ernesto. Art. 991 of the

XII.

Ernesto,

an

overseas

Civil Code, provides that if legitimate


Never Let The Odds Keep You From Pursuing What You Know In Your Heart You Were Meant To Do.Leroy Satchel Paige

Page 53 of 180

Civil Law Q&As (2007-2013)

hectorchristopher@yahoo.com

ascendants are left, the twin sons shall


divide the inheritance with them taking
one-half of the estate. Thus, the widowed
mother gets P50,000.00 while the twin
sons shall receive P25,000.00 each. The
common-law wife cannot inherit from
him because when the law speaks "widow
or widower" as a compulsory heir, the
law refers to a legitimate spouse (Art.
887, par 3, Civil Code).

Preterition; Disinheritance (2008)


No.X. Arthur executed a will which contained
only: (i) a provision disinheriting his daughter
Bernica for running off with a married man,
and (ii) a provision disposing of his share in
the family house and lot in favor of his other
children Connie and Dora. He did not make
any provisions in favor of his wife Erica,
because as the will stated, she would anyway
get of the house and lot as her conjugal
share.

The

will

was

very

brief

and

straightforward and both the above provisions


were contained in page 1, which Arthur and
his

instrumental

dbaratbateladot@gmail.com

page, but this only contained the notarial

witness,

signed

at

the

bottom. Page 2 contained the attestation


clause and the signatures, at the bottom
thereof, of the 3 instrumental witnesses
which included Lambert, the driver of Arthur;
Yoly, the family cook, and Attorney Zorba, the
lawyer who prepared the will. There was a 3rd

acknowledgement.

be preterited.
The attestation clause stated the will was
signed on the same occasion by Arthur and
his instrumental witnesses who all signed
in the presence of each other, and the

(B). What other defects of the will, if any,


can cause denial of probate? (2%)
SUGGESTED ANSWER:

notary public who notarized the will. There


are no marginal signatures or pagination

The other defects of the will that can

appearing on any of the 3 pages. Upon his

cause its denial are as follows: (a) Atty.

death, it was discovered that apart from the

Zorba, the one who prepared the will was

house and lot, he had a P 1 million account

one of the three witnesses, violating the

deposited with ABC bank.

three-witnesses rule; (b) no marginal


signature at the last page; (c ) the

(A). Was Erica preterited? (1%)

attestation did not state the number of


SUGGESTED ANSWER:

pages upon which the will is written;

Erica cannot be preterited. Art. 854 of


the

Civil

Code

provides

that

only

compulsory heirs in the direct line can

and,

(d)

no

pagination

appearing

correlatively in letters on the upper part


of the three pages (Azuela v. C.A., G.R.

Never Let The Odds Keep You From Pursuing What You Know In Your Heart You Were Meant To Do.Leroy Satchel Paige

Page 54 of 180

Civil Law Q&As (2007-2013)

hectorchristopher@yahoo.com

dbaratbateladot@gmail.com

No. 122880, 12 Apr 2006 and cited cases

(B). Between Marian and the baby, who is

therein, Art 805 and 806, Civil Code).

presumed to have died ahead? (1%)

(C). Was the disinheritance valid? (1%)

SUGGESTED ANSWER:

SUGGESTED ANSWER:

Marian is presumed to have died ahead


of the baby. Art. 43 applies to persons

Yes, the disinheritance was valid. Art.


919, par 7, Civil Code provides that
"when a child or descendant leads a
dishonorable

or

disgraceful

life,

like

running off with a married man, there is


sufficient cause for disinheritance."

who are called to succeed each other.


The proof of death must be established
by positive or circumstantial evidence
derived from facts. It can never be
established from mere inference. In the
present case, it is very clear that only
Marian and Pietro were hacked with
bolos. There was no showing that the

Succession;

Proof

of

Death

between

baby was also hacked to death. The

persons called to succeed each other

baby's death could have been due to lack

(2008)

of nutrition.

No. II. At age 18, Marian found out that she

ALTERNATIVE ANSWER:

was pregnant. She insured her own life and


named her unborn child

as her sole

The baby is presumed to have died ahead

beneficiary. When she was already due to

of Marian. Under Par. 5, rule 131, Sec. 5

give birth, she and her boyfriend Pietro, the

(KK) of the Rules of Court, if one is

father

of

her

unboarn

child,

were

kidnapped in a resort in Bataan where they


were vacationing. The military gave chase
and after one week, they were found in an
abandoned hut in Cavite. Marian and Pietro
were hacked with bolos. Marian and the

under 15 or above 60 and the age of the


other is in between 15 and 60, the latter
is presumed to have survived. In the
instant case, Marian was already 18
when

she

found

out

that

she

was

baby delivered were both found dead, with

pregnant. She could be of the same age

the baby's umbilical cord already cut. Pietro

or maybe 19 years of age when she gave

survived.

birth.

father of the baby, be entitled to claim the


(C). Will Pietro, as surviving biological

Never Let The Odds Keep You From Pursuing What You Know In Your Heart You Were Meant To Do.Leroy Satchel Paige

Page 55 of 180

Civil Law Q&As (2007-2013)

hectorchristopher@yahoo.com

proceeds of the life insurance on the life of


Marian? (2%)
SUGGESTED ANSWER:
Pietro, as the biological father of the
baby, shall be entitled to claim the
proceeds of life insurance of the Marian
because he is a compulsory heir of his
child.

Succession; Rule on Survivorship (2009)


No. II. Dr. Lopez, a 70-year old widower, and
his son Roberto both died in a fire that
gutted their home while they were sleeping
in their air-conditioned rooms. Robertos
wife, Marilyn, and their two children were
spared because they were in the province at
the time. Dr. Lopez left an estate worth
P20M and a life insurance policy in the
amount of P1M with his three children --one of whom is Roberto --- as beneficiaries.
Marilyn is now claiming for herself and her
children her husbands share in the estate
left by Dr. Lopez, and her husbands share
in the proceeds of Dr. Lopezs life insurance
policy. Rule on the validity of Marilyns
claims with reasons. (4%)
SUGGESTED ANSWER :

dbaratbateladot@gmail.com

As to the Estate of Dr. Lopez:

three children of Dr. Lopez . Marilyn


cannot represent her husband Roberto
Marilyn is not entitled to a share in the

because the right is not given by the law

estate of Dr. Lopez. For purpose of

to a surviving spouse.

succession,

Dr.

Lopez

and

his

son

Roberto are presumed to have died at


the same time, there being no evidence
to prove otherwise, and there shall be no
transmission of rights from one to the
other (Article 43, NCC). Hence, Roberto,
inherited nothing from his father that
Marilyn

would

Roberto

.The

in

turn

children

inherit
of

from

Roberto,

however, will succeed their grandfather,


Dr. Lopez ,in representation of their
father Roberto and together Roberto will
receive 1/3 of the estate of Dr. Lopez
since their father Roberto was one of the

As to the proceeds of the insurance on


the life of Dr. Lopez:

Since
regards

succession is
the

not

insurance

involved as

contract,

the

provisions of the Rules of Court (Rule


131, Sec. 3 , [jj] [5] ) on survivorship
shall apply. Under the Rules, Dr. Lopez,
who was 70 years old, is presumed to
have died ahead of Roberto who is
presumably between the ages 15 and 60.
Having survived the insured, Roberto's
right as a beneficiary became vested

Never Let The Odds Keep You From Pursuing What You Know In Your Heart You Were Meant To Do.Leroy Satchel Paige

Page 56 of 180

Civil Law Q&As (2007-2013)

hectorchristopher@yahoo.com

testator himself, Rosas claimed that the

upon the death of Dr. Lopez. When


Roberto died after Dr. Lopez, his right to
receive the insurance became part of his
hereditary estate, which in turn was
inherited in equal shares by his legal
heirs, namely, his spouse and children.
Therefore, Roberto's children and his
spouse are entitled to Roberto's onethird share in the insurance proceeds.

Wills; Holographic Wills; Insertions &


Cancellations (2012)
No.VII.a) Natividads holographic will, which
had only one (1) substantial provision, as
first written, named Rosa as her sole heir.
However, when Gregorio presented it for
probate, it already contained an alteration,
naming Gregorio, instead of Rosa, as sole
heir, but without authentication by
Natividads signature. Rosa opposes the
probate

alleging

authentication.

such
She

dbaratbateladot@gmail.com

lack

claims

of

proper

that

the

unaltered form of the will should be given


effect. Whose claim should be granted?
Explain. (5%)
SUGGESTED ANSWER:
It depends. If the cancellation of Rosas
name in the will was done by the

holographic will in its original tenor

The writing of Gregorios name as sole


should be given effect must be denied.
The said cancellation has revoked the
entire will as nothing remains of the will
after the name of Rosa was cancelled.
Such cancellation is valid revocation of
the

will

and

does

not

require

authentication by the full signature of


the testator to be effective.

heir was ineffective, even though written


by the testator himself, because such is
an

alteration

that

requires

authentication by the full signature of


the testator to be valid and effective. Not
having an authenticated, the designation
of Gregorio as an heir was ineffective,
(Kalaw v. Relova, G.R. No. L-40207, Sept
28, 1984).

However, if the cancellation of Rosas


name was not done by the testator
himself, such cancellation shall not be

Wills; Holographic Wills; Probate (2009)

effective and the will in its original tenor


shall remain valid. The effectively of the

No.VI. On December 1, 2000, Dr. Juanito

holographic will cannot be left to the

Fuentes

mercy of unscrupulous third parties.

wherein he gave nothing to his recognized

executed

holographic

will,

illegitimate son, Jay. Dr. Fuentes left for the

Never Let The Odds Keep You From Pursuing What You Know In Your Heart You Were Meant To Do.Leroy Satchel Paige

Page 57 of 180

Civil Law Q&As (2007-2013)

hectorchristopher@yahoo.com

Civil Code (Art. 816, NCC). Since Dr.

United States, passed the New York medical


licensure examinations, resided therein,
and became a naturalized American citizen.
He died in New York in 2007. The laws of
New York do not recognize holographic wills
or compulsory heirs.
(A). Can the holographic will of Dr. Fuentes
be admitted to probate in the Philippines?
Why or why not? (3%)
SUGGESTED ANSWER:
Yes, the holographic will of Dr. Fuentes
may be admitted to probate in the
Philippines because there is no public
policy violated by such probate. The only
issue at probate is the due execution of
the

will

which

includes

the

formal

validity of the will. As regards formal


validity, the only issue the court will
resolve at probate is whether or not the
will was executed in accordance with the
form prescribed by the law observed by
the testator in the execution of his will.
For

purposes

Philippines,

of

an

probate

alien

in

the

testator

may

observe the law of the place where the


will was executed (Art 17, NCC), or the
formalities of the law of the place where
he

resides,

or

dbaratbateladot@gmail.com

according

to

the

formalities of the law of his own country,


or in accordance with the Philippine

Fuentes executed his will in accordance


with the Philippine law, the Philippine

SUGGESTED ANSWER:
court shall apply the New Civil Code in
determining the formal validity of the
holographic will. The subsequent change
in the citizenship of Dr. Fuentes did not
affect the law governing the validity of
his will. Under the new Civil Code, which
was the law used by Dr. Fuentes, the law
enforced at the time of execution of the
will shall govern the formal validity of
the will (Art. 795, NCC).
(B). Assuming that the will is probated in

No, Jay cannot insist because under New


York law he is not a compulsory heir
entitled to a legitime.
The

national

law

of

the

testator

determines who his heirs are, the order


that

they

succeed, how

much

their

successional rights are, and whether or


not a testamentary disposition in his will
is valid (Art 16, NCC). Since, Dr. Fuentes
was a US citizen, the laws of the New
York determines who his heirs are. And

the Philippines, can Jay validly insist that

since

the

New

York

law

does

not

he be given his legitime? Why or why not?

recognize the concept of compulsory

(3%)

heirs, Jay is not a compulsory heir of Dr.


Fuentes entitled to a legitime.

Never Let The Odds Keep You From Pursuing What You Know In Your Heart You Were Meant To Do.Leroy Satchel Paige

Page 58 of 180

Civil Law Q&As (2007-2013)

hectorchristopher@yahoo.com

valid? (2%)

Wills; Joint Wills (2008)


No. XI. John and Paula, British citizens at
birth, acquired Philippine citizenship by
naturalization after their marriage. During
their

marriage

the

couple

acquired

substanial landholdings in London and in


Makati. Paula bore John three children,
Peter, Paul and Mary. In one of their trips to
London, the couple executed a joint will
appointing each other as their heirs and
providing that upon the death of the
survivor between them the entire estate
would go to Peter and Paul only but the two
could not dispose of nor divide the London
estate as long as they live. John and Paul
died

tragically

in

the

London

Subway

terrorist attack in 2005. Peter and Paul filed


a petition for probate of their parent's will
before a Makati Regional Trial Court.
(A). Should the will be admitted to probate?
(2%)
SUGGESTED ANSWER:
No. The will cannot be admitted to
probate because a joint will is expressly
prohibited under Art. 818 of the Civil
Code. This provision applies John and
Paula

became

Filipino

citizens

after

their marriage.
(B).

Are

the

testamentary

dispositions

dbaratbateladot@gmail.com

No.VII.b) John Sagun and Maria Carla


Camua, British citizens at birth, acquired

SUGGESTED ANSWER:

Philippine

No. The testamentary dispositions are


not valid because (a) omission of Mary, a
legitimate
preterition

child,

is

which

tantamount

shall

annul

to
the

institution of Peter and Paul as heirs


(Art.

854,

Civil

Code);

and,

(b)

the

disposition that Peter and Paul could not

citizenship

by

naturalization

after their marriage. During their marriage,


the

couple

acquired

substantial

landholdings in London and in Makati.


Maria

begot

three

(3)

children,

Jorge,

Luisito, and Joshur. In one of their trips to


London, the couple executed a joint will
appointing each other as their heirs and
providing that upon the death of the

dispose of nor divide the London estate

survivor between them, the entire estate

for more than 20 years is void (Art. 870,

would go to Jorge and Luisito only but the

Civil Code).

two (2) could not dispose of nor divide the


London estate as long as they live. John
and Maria died tragically in the London

Wills; Joint Wills; Probate (2012)

subway terrorist attack in 2005. Jorge and


Luisito filed a petition for probate of their
parents will before a Makati Regional Trial

Never Let The Odds Keep You From Pursuing What You Know In Your Heart You Were Meant To Do.Leroy Satchel Paige

Page 59 of 180

Civil Law Q&As (2007-2013)

hectorchristopher@yahoo.com

Court. Joshur vehemently objected because


he was preterited.
(1) Should the will be admitted to probate?
Explain. (2%)
SUGGESTED ANSWER:
No, the will should not be admitted to
probate. Since the couples are both
Filipino citizens, Art 818 and 819 of the
NCC shall apply. Said articles prohibits
the execution of joint wills and make
them void, even though authorized of
the country where they were executed.
(2) Are the testamentary dispositions valid?
Explain. (2%)
SUGGESTED ANSWER:
Since the joint will is void, all the
testamentary disposition written therein
are also void. However, if the will is
valid, the institutions of the heirs shall
be

annulled

because

Joshur

was

preterited. He was preterited because he


will receive nothing from the will, will
receive nothing in testacy, and the facts
do not show that he received anything as
an advance on his inheritance. He was
totally excluded from the inheritance of
his parents.

dbaratbateladot@gmail.com

(3) Is the testamentary prohibition against


the division of the London estate valid?
Explain. (1%)

that the house and lot where he lived


SUGGESTED ANSWER:

cannot be partitioned for as long as the


youngest of his four children desires to stay

Assuming the will of John and Maria was

there. As coheirs and co-owners, the other

valid, the testamentary prohibition on the

three may demand partition anytime. (1%)

division of the London estate shall be valid


but only for 20 years. Under Arts 1083 and
494 of the NCC, a testamentary disposition
of the testator cannot forbid the partition of
all or part of the estate for a period longer
than twenty (20) years.

SUGGESTED ANSWER:
FALSE, The other three co heirs may
not anytime demand the partition of the
house and lot since it was expressly
provided by the decedent in his will that
the same cannot be partitioned while his

Wills; Prohibition to Partition of a CoOwned Property (2010)


No.I. True or False.

youngest child desires to stay there.


Article 1083 of the New Civil Code allows
a decedent to prohibit, by will, the
partition of a property and his estate for
a period not longer than 20 years no

(B) X, a widower, died leaving a will stating


Never Let The Odds Keep You From Pursuing What You Know In Your Heart You Were Meant To Do.Leroy Satchel Paige

Page 60 of 180

Civil Law Q&As (2007-2013)

hectorchristopher@yahoo.com

Code.

matter what his reason maybe. Hence,


the three co-heir cannot demand its
partition at anytime but only after 20
years from the death of their father.
Even if the deceased parent did not leave
a will, if the house and lot constituted
their family home, Article 159 of the
Family Code prohibits its partition for a
period of ten (10) years, or for as long as
there is a minor beneficiary living in the
family home.

Wills; Notarial Wills; Blind Testator;


Requisites (2008)
No. XIV. Stevie was born blind. He went to
school for the blind, and learned to read in
Baille

Language.

He

Speaks

English

fluently. Can he:


(A). Make a will? (1%)
SUGGESTED ANSWER:
Assuming that he is of legal age (Art.
797, Civil Code) and of sound mind at
the time of execution of the will (Art.
798, Civil Code), Stevie, a blind person,
can make a notarial will, subject to
compliance with the "two-reading rule"
(Art. 808, Civil Code) and the provisions
of Arts. 804, 805 and 806 of the Civil

dbaratbateladot@gmail.com

read to him twice, once by one of the


(B). Act as a witness to a will? (1%)

subscribing witnesses, and again, by the


notary public before whom the will is

SUGGESTED ANSWER:

acknowledged (Art. 808, Civil Code).

Stevie cannot be a witness to a will. Art.


820 of the Civil Code provides that "any
person of sound mind and of the age of
eighteen years or more, and not blind,
deaf or dumb, and able to read and write,
may be a witness to the execution of a
will.

Wills; Testamentary Disposition; Period


to Prohibit Partition (2008)
No. XI. John and Paula, British citizens at
birth, acquired Philippine citizenship by
naturalization after their marriage. During
their

(C). In either of the above instances, must


the will be read to him? (1%)
SUGGESTED ANSWER:
If Stevie makes a will, the will must be

marriage

the

couple

acquired

substanial landholdings in London and in


Makati. Paula bore John three children,
Peter, Paul and Mary. In one of their trips to
London, the couple executed a joint will
appointing each other as their heirs and

Never Let The Odds Keep You From Pursuing What You Know In Your Heart You Were Meant To Do.Leroy Satchel Paige

Page 61 of 180

Civil Law Q&As (2007-2013)

hectorchristopher@yahoo.com

providing that upon the death of the


survivor between them the entire estate
would go to Peter and Paul only but the two
could not dispose of nor divide the London
estate as long as they live. John and Paul
died

tragically

in

the

London

Subway

terrorist attack in 2005. Peter and Paul filed


a petition for probate of their parent's will
before a Makati Regional Trial Court.
(C). Is the testamentary prohibition against
the division of the London estate valid? (2%)
SUGGESTED ANSWER:
No. the testamentary prohibition against
the division of the London estate is void
(Art.

870,

Civil

Code).

testator,

however, may prohibit partition for a


period which shall not exceed twenty
(20) years (Art. 870 in relation to Art.
494, par 3, Civil Code).

Wills; Witnesses to a Will, Presence


required;

Thumbmark

as

Signature

(2007)
No.VI. Clara, thinking of her mortality,
drafted a will and asked Roberta, Hannah,
Luisa

and

Benjamin

dbaratbateladot@gmail.com

signing her will by thumb mark and said

to

be

witnesses.

During the day of signing of her will, Clara


fell down the stairs and broke her arms.
Coming from the hospital, Clara insisted on

Probate
that she can sign her full name later. While

should

be

denied.

The

requirement that the testator and at

Roberta

least three (3) witnesses must sign all in

experienced a stomach ache and kept going

the "presence" of one another was not

to the restroom for long periods of time.

complied with. Benjamin who notarized

Hannah, while waiting for her turn to sign

the will is disqualified as a witness,

the

will

was

being

signed,

the will, was reading the 7

th

Harry Potter

hence he cannot be counted as one of

book on the couch, beside the table on

the three witnesses (Cruz v. Villasor, 54

which everyone was signing. Benjamin,

SCRA 31, 1973). The testatrix and the

aside from witnessing the will, also offered


to notarize it. A week after, Clara was run
over by a drunk driver while crossing the
street in Greenbelt.

other witnesses signed the will not in


the presence of Roberta because she was
in the restroom for extended periods of
time. Inside the restroom, Roberta could

May the will of Clara be admitted to

not have possibly seen the testatrix and

probate? Give your reasons briefly. (10%)

the other witnesses sign the will by

SUGGESTED ANSWER:

merely casting her eyes in the proper


direction (Jaboneta v. Gustilo, 5 Phil
541, 1906; Nera v. Rimando, 18 Phil

Never Let The Odds Keep You From Pursuing What You Know In Your Heart You Were Meant To Do.Leroy Satchel Paige

Page 62 of 180

Civil Law Q&As (2007-2013)

451,

1914).

Therefore,

hectorchristopher@yahoo.com

the

testatrix

signed the will in the presence of only


two witnesses, and only two witnesses
signed the will in the presence of the
testatrix and of one another.
It is to be noted, however, that the
thumb mark intended by the testator to
be his signature in executing his last will
and

testament

is

valid

(Payad

v.

Tolentino, 62 Phil 848, 1936; Matias v.


Salud, L-104 Phil 1046, 23 June, 1958).
The problem, however, states that Clara
"said that she can sign her full name
later;" Hence, she did not consider her
thumb mark as her "complete" signature,
and intended further action on her part.
The testatrix and the other witness
signed

the

will

in

the

dbaratbateladot@gmail.com

bedroom. In 1990, Brad and Jennifer broke

presence

of

Hannah, because she was aware of her


function and role as witness and was in a
position to see the testatrix and the
other witnesses sign by merely casting
her eyes in the proper direction.

Donation
Donations; Formalities; In Writing (2007)
No. VIII. In 1986, Jennifer and Brad were
madly in love. In 1989, because a certain
Picasso painting reminded Brad of her,
Jennifer acquired it and placed it in his

up. While Brad was mending his broken


heart, he met Angie and fell in love.

owner of the Picasso painting, Brad


Because the Picasso painting reminded

cannot validly bequeath the same to

Angie of him, Brad in his will bequeathed

Angie (Art. 930, NCC). Even assuming

the painting to Angie. Brad died in 1995.

that the painting was impliedly given or

Saddened by Brad's death, Jennifer asked

donated

for the Picasso painting as a remembrance

donation is nevertheless void for not

of him. Angie refused and claimed that

being in writing. The Picasso painting

Brad, in his will, bequeathed the painting

must be worth more than 5,000 pesos.

to her. Is Angie correct? Why or why not?


(10%)
SUGGESTED ANSWER:

by

Jennifer

to

Brad,

the

Under Art. 748, NCC, the donation and


acceptance of a movable worth more
than 5,000 pesos must be in writing,
otherwise the donation is void. The

NO. Angie is not correct. The Picasso

donation being void, Jennifer remained

painting is not given or donated by

the owner of the Picasso painting and

Jennifer to Brad. She merely "placed it

Brad could not have validly disposed of

in his bedroom." Hence, she is still the

said painting in favor of Angie in his will.

owner of the painting. Not being the


ALTERNATIVE ANSWER:
Never Let The Odds Keep You From Pursuing What You Know In Your Heart You Were Meant To Do.Leroy Satchel Paige

Page 63 of 180

Civil Law Q&As (2007-2013)

hectorchristopher@yahoo.com

be disregarded but the donation remains

YES. Angie is correct. Even assuming


that there was void donation because the
same was not in writing, Brad was in
uninterrupted possession of the Picasso
painting from 1989 to 1995, lasting for
six (6) years prior to his death. Brad has
already

acquired

ownership

painting

through

prescription.

Under

ownership

of

of

the

acquisitive

Art.

movables

1132,

NCC,

prescribes

through continuous possession for four


(4) years in good faith and for eight (8)
years without need of other conditions.
A void donation may be the basis of
possession in the concept of owner and
of just title for purposes of acquisitive
prescription.

Donations;

Illegal

dbaratbateladot@gmail.com

&

Impossible

Conditions (2007)
No.I. Distinguish the following concepts:
(B). Illegal and impossible conditions in a
simple donation v. illegal and impossible
conditions in an onerous donation. (5%)
SUGGESTED ANSWER:
Illegal and impossible conditions in a
simple donation are considered as not
written. Such conditions, shall therefore,

valid (Art. 727, NCC). On the other hand,

given

to

me,

hereby

freely,

voluntarily and irrevocably donate to

illegal and impossible donations imposed

her my one-hectare rice land covered

in an onerous donation shall annul the

by TCT No. 11550, located in San

donation (Art. 1183, NCC). This is so,

Fernando, Pampanga. This donation

because onerous donations are governed

shall take effect upon my death."

by the law on contracts (Art. 733, NCC).

The deed also contained Jennifer's signed


acceptance, and an attached notarized
Donation; Inter Vivos (2013)

declaration by Josefa and Jennifer that the


land will remain in Josefa's possession and

No.V. Josefa executed a deed of donation

cannot be alienated, encumbered, sold or

covering a one-hectare rice land in favor of

disposed of while Josefa is still alive.

her daughter, Jennifer. The deed specifically


provides that:

Advise Jennifer on whether the deed is a


donation inter vivos or mortis causa and

"For and in consideration of her love

explain the reasons supporting your advice.

and service Jennifer has shown and

(8%)

Never Let The Odds Keep You From Pursuing What You Know In Your Heart You Were Meant To Do.Leroy Satchel Paige

Page 64 of 180

Civil Law Q&As (2007-2013)

hectorchristopher@yahoo.com

dbaratbateladot@gmail.com

donation which state that the same will


only take effect upon the death of the

SUGGESTED ANSWER:

donor and that there is a prohibition to


The donation is a donation inter vivos.
When

the

donor

intends

that

alienate, encumber, dispose, or sell the

the

donation shall take effect during the


lifetime

of

the

donor,

though

the

property shall not be delivered till after


the

donors

death,

this

shall

be

donation inter vivos (Art. 729, Civil


Code).
The

Civil

Code

transmissions.

prefers

Moreover,

inter

vivos

mortis

causa

donations should follow the formalities of a


will (Art. 728, Civil Code). Here there is no
showing

that

such

formalities

were

followed. Thus, it is favorable to Jennifer


that the deed is a donation inter vivos.

Furthermore, what is most significant in


determining the type of donation is the
absence of stipulation that the donor
could

revoke

the

donation;

on

the

contrary, the deeds expressly declare


them

to

be

irrevocable,

quality

absolutely incompatible with the idea of


conveyances

mortis

causa

where

revocability is the essence of the act, to


the

extent

that

testator

cannot

lawfully waive or restrict his right of


revocation. The provisions of the deed of

As the donation is in the nature of a


same should be harmonized with its
express irrevocability (Austria-Magat v.
CA, G.R. No. 106755, Feb 1, 2002).

mortis causa disposition, the formalities


of a will should have been complied with
under

Art.

728

of

the

Civil

Code,

otherwise, the donation is void and


ALTERNATIVE ANSWER:

would produce no effect (The National

The donation is donation mortis causa.


The deed clearly states that the donation
shall take effect upon the death of the
donor,

Josefa.

retained

The

ownership

donor,
of

moreover,

the

subject

property as it was declared that the


property

cannot

be

Treasure of the Philippines v. Vda. de


Meimban, G.R. No. L-61023, Aug 22,
1984).

Property
Accretion; Alluvium (2008)

alienated,

No. IX. The properties of Jessica and Jenny,

encumbered, sold or disposed of while

who are neighbors, lie along the banks of

the donor is still alive.

the Marikina River. At certain times of the


year, the river would swell and as the water
recedes, soil, rocks and other materials are

Never Let The Odds Keep You From Pursuing What You Know In Your Heart You Were Meant To Do.Leroy Satchel Paige

Page 65 of 180

Civil Law Q&As (2007-2013)

deposited
properties.

on

hectorchristopher@yahoo.com

not formed solely by the natural effect of

Jessica's

This

pattern

and
of

Jenny's
the

river

swelling, receding and depositing soil and


other materials being deposited on the
neighbors' properties have gone on for
many years. Knowing his pattern, Jessica
constructed a concrete barrier about 2
meters

from

her

property

line

and

extending towards the river, so that when


the water recedes, soil and other materials
are trapped within this barrier. After several
years, the area between Jessica's property
line to the concrete barrier was completely
filled

with

soil,

effectively

increasing

Jessica's property by 2 meters. Jenny's


property, where no barrier was constructed,
also increased by one meter along the side
of the river.
(A). Can Jessica and Jenny legally claim
ownership over the additional 2 meters and
one meter, respectively, of land deposited
along their properties?(2%)
SUGGESTED ANSWER:
Only Jenny can claim ownership over
the

additional

one

dbaratbateladot@gmail.com

meter

of

land

deposited along her property. Art. 457 of


the Civil Code provides that "to the
owners of lands adjoining the banks of
river belong the accretion which they
gradually receive from the effects of the
current of the water." Where the land is

the water current of the river bordering

(B). If Jessica's and Jenny's properties are


land but is also the consequences of the
direct and deliberate intervention of

registered,

will

the

benefit

of

such

registration extend to the increased area of


their properties? (2%)

man, it is man-made accretion and a


part of the public domain (Tiongco v.
Director of Lands, 16 C.A. Rep 211, cited
in Nazareno v. C.A., G.R. No. 98045, 26
June 1996). Thus, Jessica cannot legally
claim ownership of the additional 2
meters
because

of

land

she

along

her

constructed

property
concrete

barrier about 2 meters from her property


causing

deposits

of

soil

and

other

materials when the water recedes. In


other words, the increase in her property
was not caused by nature but was manmade.

SUGGESTED ANSWER:
If the properties of Jessica and Jenny
are

registered,

registration
increased
Accretion

the

does
area
does

benefit

not
of

extend

their
not

of
to

such
the

properties.

automatically

become registered land because there is


a specific technical description of the lot
in its Torrens title. There must be a
separate application for registration of
the alluvial deposits under the Torrens
System (Grande v. CA, G.R. No. L-17652,
30 June, 1962).

Never Let The Odds Keep You From Pursuing What You Know In Your Heart You Were Meant To Do.Leroy Satchel Paige

Page 66 of 180

Civil Law Q&As (2007-2013)

hectorchristopher@yahoo.com

(C). Assume the two properties are on a cliff


adjoining the shore of Laguna Lake. Jessica
and Jenny had a hotel built on the
properties. They had the erath and rocks
excavated from the properties dumped on
the adjoining shore, giving rise to a new
patch of dry land. Can they validly lay claim
to the patch of land? (2%)
SUGGESTED ANSWER:
No. Jessica and Jenny cannot validly lay
claim to the patch of land because in
order to acquire land by accretion, there
should

be

natural

dbaratbateladot@gmail.com

gradually the land owned by Marciano

and

actual

continuity of the accretion to the land of


the riparian owner caused by natural ebb
and flow of the current of the river
(Delgado v. Samonte, CA-G.R. No. 34979R, 10 Aug 1966).

Accretion; Rights of the Riparian Owner


(2009)
No.XVI. Marciano is the owner of a parcel of
land through which a river runs out into
the sea. The land had been brought under
the Torrens System, and is cultivated by
Ulpiano and his family as farmworkers
therein. Over the years, the river has
brought silt and sediment from its sources
up in the mountains and forests so that

increased in area by three hectares.

(A). Is Marciano correct? Explain. (3%)


Ulpiano built three huts on this additional
area,

where

he

and

his

two

married

children live. On this same area, Ulpiano


and his family planted peanuts, monggo
beans

and

vegetables.

Ulpiano

also

SUGGESTED ANSWER:
Marcianos contention is correct. Since
that accretion was deposited on his land
by the action of the waters of the river

regularly paid taxes on the land, as shown

and he did not construct any structure

by tax declarations, for over thirty years.

to increase the deposition of soil and


silt, Marciano automatically owns the

When Marciano learned of the increase in


the size of the land, he ordered Ulpiano to
demolish the huts, and demanded that he
be paid his share in the proceeds of the
harvest. Marciano claims that under the
Civil Code, the alluvium belongs to him as a
registered riparian owner to whose land the

accretion. His real right of ownership is


enforceable

against

the

whole

world

including Ulpiano and his two married


children. Although Marcianos land is
registered, the three (3) hectares land
deposited through accretion was not

accretion attaches, and that his right is

automatically

enforceable against the whole world.

unregistered

registered.
land,

it

is

As

an

subject

to

acquisitive prescription by third persons.


Never Let The Odds Keep You From Pursuing What You Know In Your Heart You Were Meant To Do.Leroy Satchel Paige

Page 67 of 180

Civil Law Q&As (2007-2013)

hectorchristopher@yahoo.com

dbaratbateladot@gmail.com

he has the right to deduct from the


value of the fruits the expenses for

Although Ulpiano and his children live in


the three (3) hectare unregistered land
owned

by

Marciano,

they

are

farm

workers; therefore, they are possessors


not in the concept of owners but in the
concept of mere holders. Even if they
possess the land for more than 30 years,
they cannot become the owners thereof
through

extraordinary

acquisitive

prescription, because the law requires


possession in the concept of the owner.
Payment of taxes and tax declaration are
not enough to make their possession one
in the concept of owner. They must
repudiate the possession in the concept
of holder by executing unequivocal acts
of repudiation amounting to ouster of
Marciano, known to Marciano and must
be

proven

evidence.

by
Only

clear

and

then

convincing
would

his

possession become adverse.


(B). What rights, if any, does Ulpiano have
against Marciano? Explain. (3%)
SUGGESTED ANSWER:
Although Ulpiano is a possessor in bad
faith, because he knew he does not own
the land, he will lose the three huts he
built

in

bad

faith

and

make

an

accounting of the fruits he has gathered,

CRC likewise sold to the spouses Rodriguez,


production, gathering and preservation
of the fruits (Art 443, NCC).

He may also ask for reimbursement of

a 700-square meter land (Lot B) which is


adjacent to Lot A. Lot B has a present fair
market value of P1,500,000.
The spouses Dela Cruz constructed a house

the taxes he has paid, as these are

on Lot B, relying on their presentation of

charges on the land owned by Marciano.

the CRC sales agent that it is the property

This obligation is based on a quasi-

they purchased. Only upon the completion

contract (Art 2175, NCC).

of their house did the spouses Dela Cruz


discovered that they had built on Lot B
owned by the spouses Rodriguez, not on Lot

Builder; Good Faith; Requisites (2013)


No.VIII. Ciriaco Realty Corporation (CRC)
sold to the spouses Del a Cruz a500-square
meter land (Lot A) in Paranaque. The land
now has a fair market value of Pl,200,000.

A that they purchased. They spent P 1


000,000 for the house.
As their lawyer, advise the spouses Dela
Cruz on their rights and obligations under
the given circumstances, and the recourses

Never Let The Odds Keep You From Pursuing What You Know In Your Heart You Were Meant To Do.Leroy Satchel Paige

Page 68 of 180

Civil Law Q&As (2007-2013)

hectorchristopher@yahoo.com

in Art 546 and 548, or

and options open to them to protect their


interests. (8%)

Based on the fact as stated, the spouses


Dela Cruz as builders and the spouses
Rodriguez as land owners, are both in
good faith. The spouses Dela Cruz are
builder in good faith because before
constructing the house they exercised
due diligence by asking the Agent of CRC
the location of the lot A, and they relied
on the information given by the agent
who is presumed to know the identity of
the lot purchased by the Dela Cruz
spouses (Pleasantville v. CA, 253 SCRA
10, 1996). On the other hand, there is no
showing that the land owners, spouse
Rodriguez acted in bad faith. The facts
do not show that the building was done
their

(2) to oblige the one who built to pay the


price of the land.

SUGGESTED ANSWER:

with

knowledge

dbaratbateladot@gmail.com

and

without

opposition on their part (Art 453, Civil


Code). The good faith is always presumed
(Art. 527, Civil Code).
The owner of the land on which anything
has been built, sown, or planted in good
faith shall have the right:
(1) to appropriate as his own the works after
payment of the indemnity provided for

house,
However, the builder cannot be obliged
to

buy

the

considerable

land
more

if

its

than

value

that

of

is
the

building.. In such case, he shall pay


reasonable rent of the owner of the land
does

not

choose

to

appropriate

the

building or trees after proper indemnity


(Art 448, Civil Code).

the

spouses

Dela

Cruz

are

entitled to the right of retention pending


reimbursement of the expenses they
incurred or the increase in value which
the thing may have acquired by reason of
the improvement (Art 546, Civil Code).
Thus,

the

spouses

Dela

Cruz

may

demand P1,000,000.00 as payment of


the expenses in building the house or
increase in value of the land because of

The house constructed by the spouses

the house as a useful improvement, as

Dela Cruz is considered as a useful

may be determined by the court form

expense, since it increased the value of

the evidence presented during the trial

the lot. As such, should the spouses

(Depra v. Dumlao, 136 SCRA 475, 1985;

Rodriguez decides to appropriate the

Technogas Phils v. CA, 268 SCRA 5,


1997).

Never Let The Odds Keep You From Pursuing What You Know In Your Heart You Were Meant To Do.Leroy Satchel Paige

Page 69 of 180

Civil Law Q&As (2007-2013)

Easement;

Prescription;

hectorchristopher@yahoo.com

Acquisitive

Prescription (2009)

dbaratbateladot@gmail.com

In 2006, Brand0 fenced off his property,


thereby blocking Andres' access to the
national highway. Andres demanded that

No. XI. TRUE or FALSE. Answer TRUE if

part of the fence be removed to maintain

the statement is true, or FALSE if the

his

statement is false. Explain your answer in

(pathway A), but Brando refused, claiming

not more than two (2) sentences.

that there was another available pathway

(C). Acquisitive prescription of a negative


easement runs from the time the owner of
the dominant estate forbids, in a notarized
document, the owner of the servient estate
from executing an act which would be
lawful without the easement. (1%)

old

access

route

to

the

highway

(pathway B) for ingress and egress to the


highway. Andres countered that pathway B
has defects, is circuitous, and is extremely
inconvenient to use.
To settle their dispute, Andres and Brando
hired Damian, a geodetic and civil engineer,
to survey and examine the two pathways

SUGGESTED ANSWER:
True. In negative easements, acquisitive
prescription runs from the moment the

and

the

determine
prejudicial

surrounding
the
way

areas,

shortest
through

and
the

and
the

to

least

servient

owner of the dominant estate forbade, by

estates. After the survey, the engineer

an

before

concluded that pathway B is the longer

notary public, the owner of the servient

route and will need improvements and

estate from executing an act which

repairs, but will not significantly affect the

would be lawful without the easement

use of Brando's property. On the other

(Art. 621, NCC).

hand, pathway A that had long been in

instrument

acknowledged

place, is the shorter route but would


significantly affect the use of Brando's
Easement; Right of Way (2013)
No.VII.In 2005, Andres built a residential
house on a lot whose only access to the
national highway was a pathway crossing
Brando's property. Andres and others have
been using this pathway (pathway A) since
1980.

property.
In light of the engineer's findings and the
circumstances of the case, resolve the
parties' right of way dispute. (6%)
SUGGESTED ANSWER:

Never Let The Odds Keep You From Pursuing What You Know In Your Heart You Were Meant To Do.Leroy Satchel Paige

Page 70 of 180

Civil Law Q&As (2007-2013)

hectorchristopher@yahoo.com

complied with. First, there is another

Andres is not entitled to the easement of


right of way for Pathway A. Pathway B
must be used.
The owner of a dominant estate may
validly obtain a compulsory right of way
only

after

he

has

established

the

existence of four requisites, to wit:


(1) The (dominant) estate is surrounded by
other

immovables

and

is

without

adequate outlet to a public highway;


(2) After payment of the proper indemnity;
(3) The

isolation

was

not

due

to

the

proprietors own acts; and


(4) The right of way claimed is at a point
least prejudicial to the servient estate,
and insofar as consistent with this rule,
where the distance from the dominant
estate to the public highway maybe the
shortest (Art 650, civil Code).
However,

the

dbaratbateladot@gmail.com

Supreme

Court

has

consistently ruled that in case both


criteria cannot be complied with, the
right of way shall be established at the
point least prejudicial to the servient
estate.
The first and fourth requisites are not

available outlet to the national highway

established at the point least prejudicial


(Pathway B). Second, the right of way
obtained (Pathway A) is not the least
prejudicial

to

Brandos

property,

as

evidence by the reports of the geodetic


and civil engineer.
When

there

adequate

is

outlet

(Quimen v. Quimen and CA, G.R. No.


112331, May 29, 1996).
(Note: It is not clear from the problem if there
exists an easement in favor of the lot

already
from

an

the

existing
dominant

estate to the public highway, even if the


said outlet, for one reason or another, be
inconvenient,

to the servient estate is controlling

created as legal easement or as a voluntary

tolerance, then Brando may close it. Andres

(Costabella Corporation v. CA, G.R. No.

must ask for the constitution of a legal

80511, Jan 25, 1991). The rule that the

easement through Brandos lot by proving

easement

the four requisites required by Art 649 and

way

open

easement burdening Brandos lot, was it

another servitude is entirely unjustified

of

to

way as a servient estate. If there is such an

easement. If the used pathway was only a

right

need

burdened as a servient estate by a right of

up

of

the

belonging to Andres and if Brandos lot is

shall

be

65, Civil Code).

Never Let The Odds Keep You From Pursuing What You Know In Your Heart You Were Meant To Do.Leroy Satchel Paige

Page 71 of 180

Civil Law Q&As (2007-2013)

hectorchristopher@yahoo.com

Easement; Right of Way (2010)


No.XIII. Franz was the owner of Lot E which
was surrounded by four (4) lots one of
which Lot C he also owned. He promised
Ava that if she bought Lot E, he would give
her a right of way in Lot C.
Convinced, Ava bought Lot E and, as
promised, Franz gave her a right of way in
Lot C.
Ava cultivated Lot E and used the right of
way granted by Franz.
Ava later found gainful employment abroad.
On her return after more than 10 years, the
right of way was no longer available to her
because Franz had in the meantime sold
Lot C to Julia who had it fenced.
(A). Does Ava have a right to demand from
Julia the activation of her right of way?
Explain. (2.5%)
SUGGESTED ANSWER:
Yes. Ava has the right to demand from
Julia the activation of the right of way,
for the following reasons:
(1) The easement of the right of way is a
real right which attaches to, and is
inseperable from, the estate to which it
belongs.

dbaratbateladot@gmail.com

(2) The sale of the property includes the

discontinuous easement, the period of


ten years of non-user, shall be computed

easement or servitude, even if the deed

from the day it ceased to be used under

of sale is silent on the matter.

Act 6341 (2) CC.


(3) The vendee of the property in which a
servitude

or

easement

exists

cannot

close or put obstructions thereon to


prevent the dominant estate from using
it.

(5)

Renunciation

or

waiver

of

an

easement must be specific, clear, express


and made in a public instrument in
accordance of Art 1358 of the New Civil
Code.

(4) Avas working abroad for more than ten

ALTERNATIVE ANSWER:

(10) years should not be construed as

Yes. Ava has the right to demand from

non-user, because it cannot be implied

Julia the activation of the right of way. A

from the fact that she or those she left

voluntary easement of right of way, like

behind to cultivate the lot no longer use

any

the right of way.

extinguished only by mutual agreement

other

contract,

could

be

or by renunciation of the owner of the


Note:

Since

right

of

way

is

dominant estate. Also, like any other

Never Let The Odds Keep You From Pursuing What You Know In Your Heart You Were Meant To Do.Leroy Satchel Paige

Page 72 of 180

Civil Law Q&As (2007-2013)

contract,

an

hectorchristopher@yahoo.com

easement

is

generally

effective between parties, their heirs and


assignees, except in case where the
rights and obligations arising from the
contract are not transmissible by their
nature, or by stipulations or by provision
of law (Unisource Commercial v. Chung,
593 SCRA 530 [2009]).
(B). Assuming Ava opts to demand a right of
way from any of the owners of Lots A, B,
and D, can she do that? Explain. (2.5%)
SUGGESTED ANSWER:
Yes. Ava has the option to demand a
right of way on any of the remaining lots
of Franz more so after Franz sold lot C
to Julia. The essential elements of a
legal right of way under Art 649 and 650
of the New Civil Code are complied with.
ALTERNATIVE ANSWER:
Yes. Ava has the option to demand a
right of way from the other lots. The law
provides that whenever a piece of land
acquired by sale, exchange or partition is
surrounded

by

other

estates

of

the

vendor, exchanger, or co-owner, he shall


be obliged to grant a right of way
without indemnity (Art 652, NCC).
ALTERNATIVE ANSWER:

dbaratbateladot@gmail.com

No. There was merely a promise to Ava


that a right of way shall be granted to

engaged by Blas to construct a house on a


her in lot C if Ava purchase lot E. The
promise was not reduced to writing (Obra
v. Baldria, 529 SCRA 621 [2007]). Hence,

lot which he (Blas) owns. While digging on


the lot in order to lay down the foudation of
the house, Adam hit a very hard object. It
turned out to be the vault of the old Banco

it was not or could not have been

de las Islas Filipinas. Using a detonation

registered as to warn buyers of lot C

device, Adam was able to open the vault

about the existence of the easement on

containing old notes and coins which were

the property. Not having been annotated

in circulation during the Spanish era. While

on the TCT to lot C, the buyer acquired

the notes and coins are no longer legal

lot C free from such right of way granted

tender, they were valued at P100 million

to Ava.

because of their historical value and the


coins silver nickel content. The following
filed legal claims over the notes and coins:

Hidden Treasure (2008)

(i). Adam, as finder;

No. VIII. Adam, a building contractor, was

(ii). Blas, as owner of the property where


they were found;

Never Let The Odds Keep You From Pursuing What You Know In Your Heart You Were Meant To Do.Leroy Satchel Paige

Page 73 of 180

Civil Law Q&As (2007-2013)

hectorchristopher@yahoo.com

the State and by chance, one-half of it

(iii). Bank of the Philippine Islands, as


successor-in-interest of the owner of the
vault; and
(iv). The Philippine Government because of
their historical value.
(A). Who owns the notes and coins? (4%)
SUGGESTED ANSWER:
The notes and coins are no longer owned
by the Bank of the Philippine Islands,
which has either lost or abandoned the
vault and its contents, and it has not
taken any effort to search, locate or
recover the vault. In any case, since the
vault is in actual possession of Adam,
BPI may attempt, in a judicial action to
recover, to rebut the presumption of
ownership in favor of Adam and Blas
(Art. 433, Civil Code). Hidden treasure is
any hidden and unknown deposit of
money,

jewelry,

or

other

precious

objects, the lawful ownership of which


does not appear. Given the age and
importance of the items found, it would
be safe to consider the vault, notes and
coins

abandoned

by

dbaratbateladot@gmail.com

BPI

and

its

predecessor (Art. 439, Civil Code). It


belongs to the owner of the land on
which it is found. When the discovery is
made on the property of another, or of

shall belong to the finder who is not a


trespasser (Art. 438, Civil Code). In the

present case, Adam, as finder, and Blas,


as owner of the land, are entitled to
share

50-50

in

the

treasure.

The

government can only claim if it can


establish that the notes and coins are of
interest to science or the arts, then it

If either or both Adam and Blas are


adjudged as owners, the notes and coins
shall be deemed part of their absolute
community or conjugal partnership of
gains with their respective spouses (Art.
117, par 4, FC).

must pay just price of the things found,


to be divided equally between Adam and
Blas (Art. 438, Civil Code).

Mortgage; Public or Private Instrument


(2013)

(B). Assuming that either or both Adam and


Blas are adjudged as owners, will the notes

No.VI. Lito obtained a loan of P1,000,000

and coins be deemed part of their absolute

from Ferdie, payable within one year. To

community or conjugal partnership of gains

secure payment, Lito executed a chattel

with their respective spouses? (2%)

mortgage on a Toyota Avanza and a real


estate mortgage on a 200-square meter

SUGGESTED ANSWER:

piece of property.

Never Let The Odds Keep You From Pursuing What You Know In Your Heart You Were Meant To Do.Leroy Satchel Paige

Page 74 of 180

Civil Law Q&As (2007-2013)

hectorchristopher@yahoo.com

(A) Would it be legally significant - from the


point of view of validity and enforceability if the loan and the mortgages were in public
or private instruments? (6%)
SUGGESTED ANSWER:
From the point of view of validity and
enforceability,

there

would

be

legal

significance if the mortgage was in a


public or private instrument. As for the
loan,

there

is

no

legal

significance

except of interest were charged on the


loan, in which case, the charging of
interest must be in writing.
A contract of loan is a real contract and
is perfected upon delivery of the object
of the obligation (Art 1934, Civil Code).
Thus, a contract of loan is valid and
enforceable even if it is neither in a
private nor in a public document.
As a rule, contracts shall be obligatory in
whatever

form

they

may

have

been

entered into provided all the essential


requisites for their validity are present.
With regards to its enforceability, a
contact of loan is not among those
enumerated under Art. 1403 (2) of the
Civil Code, which are covered by the
Statute of Frauds.

dbaratbateladot@gmail.com

It is important to note that under Art.


1358 of the Civil Code, all the other
contracts where the amount involved

order that a mortgage may be validly


exceeds Five Hundred pesos (P500.00)
must appear in writing, even in private
one. However, the requirement is not for
validity of the contract, but only for its

constituted the document in which it


appears be recorded. If the instrument is
not

recorded,

the

mortgage

is

nevertheless valid and binding between


the parties. Hence, for validity of both

greater efficacy.

chattel and real estate mortgages, they


With regard to the chattel mortgage, Art.

must appear in a public instrument. But

1508, the Chattel Mortgage Law, requires

the

an affidavit of good faith stating that the

submitted that the form of the contract,

chattel mortgage is supposed to stand as

whether in a public or private document,

security

would

of

the

loan;

thus,

for

the

validity of the chattel mortgage, it must

purpose

be

of

enforceability,

immaterial

(Mobil

it

Oil

is

v.

Diocaresa, 29 SCRA 656, 1969).

be in a public document and recorded in


the Chattel Mortgage Register in the

Also, under Art 1358, acts and contracts

Register

which have for their object the creation

of

Deeds.

real

estate

mortgage, under the provisions of Art.


2125 of the Civil Code, requires that in

or

transmission

of

real

rights

over

immovable property must be in a public

Never Let The Odds Keep You From Pursuing What You Know In Your Heart You Were Meant To Do.Leroy Satchel Paige

Page 75 of 180

Civil Law Q&As (2007-2013)

hectorchristopher@yahoo.com

intention to become the owner thereof.

document for greater efficacy and a real


estate mortgage is a real right over
immovable property.

Occupation vs. Possession (2007)


No.I. Distinguish the following concepts:
(A). Occupation v. possession. (5%)
SUGGESTED ANSWER:
Occupation

is

an

original

mode

of

acquiring ownership (Art. 712, NCC).


Things appropriable by nature which are
without an owner, such as animals that
are the object of hunting and fishing,
hidden

treasure

and

abandoned

movables, are acquired by occupation


(Art. 713, NCC). However, ownership of a
piece of land cannot be acquired by
occupation (Art. 714, NCC).
ALTERNATIVE ANSWER:
Occupation

is

mode

of

acquiring

dominion by the seizure of corporeal


things which have no owner, with the
intention of acquiring the ownership
thereof.

It

is

an

dbaratbateladot@gmail.com

original

mode

of

acquiring ownership upon seizure of a


res nullius by the occupant who has the

Possession, on the other hand, is the


holding of the thing or an enjoyment of a

right. Possession may be the real right of


possession or jus possessiones or it can
be merely the right to possess or jus
possedendi, which are among the basic

No. XI. TRUE or FALSE. Answer TRUE if


the statement is true, or FALSE if the
statement is false. Explain your answer in
not more than two (2) sentences.

rights of ownership. If the real right of

(D). The renunciation by a co-owner of his

possession is possession in the concept

undivided share in the co-owned property

of

in lieu of the performance of his obligation

owner,

limitations,

but
it

subject
may

to

ripen

certain
into

full

ownership of the thing or property right


through

acquisitive

prescription

depending on whether it is a case of


ordinary or extraordinary prescription

to contribute to taxes and expenses for the


preservation of the property constitutes
dacion en pago. (1%)
SUGGESTED ANSWER:
True, Under the Civil Code, a co-owner

and whether the property is movable or

may renounce his share in the co-owned

immovable.

property in lieu of paying for his share in


the

Ownership; Co-Ownership (2009)

taxes

and

expenses

for

the

preservation of the co-owned property.

Never Let The Odds Keep You From Pursuing What You Know In Your Heart You Were Meant To Do.Leroy Satchel Paige

Page 76 of 180

Civil Law Q&As (2007-2013)

In

effect,

there

is

hectorchristopher@yahoo.com

dacion

en

pago

because the co-owner is discharging his


monetary obligation by paying it with his
non-monetary interest in the co-owned
property. The fact that he is giving up
his entire interest simply means that he
is accepting the value of his interest as
equivalent to his share in the taxes and
expenses of preservation.

Ownership; Co-Ownership (2008)


No. VI. Alex died without a will, leaving only
an undeveloped and untitled lot in Tagiug
City. He is survived by his wife and 4
children. His wife told the children that she
is waiving her share in the property, and
allowed Bobby, the eldest son who was
about to get married, to construct his
house on of the lot, without however
obtaining the consent of his siblings. After
settlement of Alex's estate and partition
among the heirs, it was discovered that
Bobby's house was constructed on the
portion allocated to his sister, Cathy asked
Bobby to demolish his house and vacate the
portion alloted to her. In leiu of demolition,
Bobby offered to purchase from Cathy the
lot

dbaratbateladot@gmail.com

(A). Can Cathy lawfully ask for demolition of

portion

on

which

his

constructed.

At

that

time,

house
the

constructed was valued at P350.000.

was
house

Bobby's house? (3%)

dominion

or

ownership

such

as

construction of a house." In the present

SUGGESTED ANSWER:

case, of Alex is the real owner of the


Yes, Cathy can lawfully ask for the

undeveloped and untitled lot in Taguig,

demolition

Where

co-ownership is created among his wife

there are two or more heirs, the whole

and four children over said property

estate

before

upon his death. Since the construction

partition, owned in common by such

of the house by Bobby was done without

heirs, subject to the payment of debts of

obtaining the consent of his siblings, the

the deceased (Art. 1078, Civil Code),

alteration effected is illegal. Bobby is

Under the rules on co-ownership, "none

considered to be in bad faith and as a

of

sanction for his conduct, he can be

the

of

of

the

Bobby's

house.

decedent,

co-owners

shall,

is,

without

the

consent of the others make alterations

compelled

in the thing owned in common, even

remove

though benefits for all would results

expense.

by

the

Cathy

to

structure

demolish
at

his

or
own

therefrom." In Cruz v. Catapang, G.R. No.


164110, 12 Feb., 2008, the Court held
that "alterations include any act of strict

(B). Can Bobby legally insist on purchasing


the land? (2%)

Never Let The Odds Keep You From Pursuing What You Know In Your Heart You Were Meant To Do.Leroy Satchel Paige

Page 77 of 180

Civil Law Q&As (2007-2013)

hectorchristopher@yahoo.com

though floating, are intended by their

SUGGESTED ANSWER:
No.

Bobby

cannot

legally

insist

on

purchasing the land. Being in bad faith,


he has no option to pay for the price of
the lot (Art. 450, Civil Code).

Property; Movable or Immovable (2007)


No.II. Manila Petroleum Co. owned and
operated a petroleum operation facility off
the coast of Manila. The facility was located
on a floating platform made of wood and
metal,

upon

which

was

permanently

attached the heavy equipment for the


petroleum operations and living quarters of
the crew. The floating platform likewise
contained a garden area, where trees,
plants

and

flowers

were

planted.

The

platform was tethered to a ship, the MV


101, which was anchored to the seabed.
Please briefly give the reason for your
answers. (10%)
(A).Is the platform movable or immovable
property?
SUGGESTED ANSWER:
The platform is an immovable property
under Art. 415 (9) NCC, which provides
that

"docks

and

dbaratbateladot@gmail.com

structures

which,

hence, it remains a movable property. If


nature and object to remain at a fixed
place on a river, lake or coast." Since the
floating
operation

platform
facility,

is
it

a
is

petroleum
intended

to

remain permanently where it is situated,


even if it is tethered to a ship which is

the intention was to make the platform


stay permanent where it was moored, it
would not have been simply tethered to a
vessel but itself anchored to the seabed.
(B). Are the equipment and living quarters
movable or immovable property?

anchored to the seabed.


SUGGESTED ANSWER:
ALTERNATIVE ANSWER:
The thing and living quarters of the crew
The platform is a movable property

are immovable property under Art. 415

because it is attached to a movable

(3) NCC, classifies as an immovable

property,

was

"everything attached to an immovable in

merely anchored to the seabed. The fact

a fixed manner, in such a way that it

that the vessel is merely anchored to the

cannot be separated therefrom without

sea

breaking the material or deterioration of

bed

i.e.

only

the

vessel

shows

that

which

it

is

not

intended to remain at a fixed place;

Never Let The Odds Keep You From Pursuing What You Know In Your Heart You Were Meant To Do.Leroy Satchel Paige

Page 78 of 180

Civil Law Q&As (2007-2013)

hectorchristopher@yahoo.com

the object." Both the equipment and the


living quarters are permanently attached
to

the

platform

which

is

also

an

immovable. The equipment can also be


classified

as

an

immovable

property

under Art. 415 (5) NCC because such


equipment are "machinery, receptacles,
instruments or implements intended by
the

owner

of

the

tenement

for

an

industry or works which may be carried


on in a building or on a piece of land and
which tend directly to meet the needs of
the industry or works." It is logically
assumed that the petroleum industry
may be carried on in a building or on a
piece

of

land

and

the

platform

is

analogous to a building.
ALTERNATIVE ANSWER:
The equipment and living quarters of the
crew are movable properties since they
are attached to a platform which is also
a movable property, because it is simply
attached

to

vessel

is

likewise

movable property since it was merely


anchored on the seabed only shows that
it is not intended to remain at a fixed
place;

hence,

it

dbaratbateladot@gmail.com

immovable or movable property?

remains

movable

property.
(C). Are the trees, plants and flowers

SUGGESTED ANSWER:

immovable.
The trees, plants and flowers planted in
the garden area of the platform are
immovable property under Art. 415 (2)
NCC which classifies as an immovable
property

"trees,

plants

and

growing

fruits, while they are attached to the


land or form an integral part of an
immovable,

the

petroleum

operation

facility.

The

platform

is

not

an

immovable property for the same reason


already given in the Alternative Answer
to Item (a) above.

Land Titles and Deeds


Acquisition

of

Lands;

Sale

of

Real

Property to an Alien (2009)


No.XIX. In 1972, Luciano de la Cruz sold to
Chua Chung Chun, a Chinese citizen, a

ALTERNATIVE ANSWER:

parcel of land in Binondo. Chua died in

The trees, plants and flowers planted in

1990, leaving behind his wife and three


children,

one

of

whom,

Julian,

is

the garden area of the platform are

naturalized Filipino citizen. Six years after

movable property because they are not

Chuas

permanently attached t the land and do

extrajudicial settlement of estate, and the

not

parcel of land was allocated to Julian. In

form

an

integral

part

of

an

death,

the

heirs

executed

an

Never Let The Odds Keep You From Pursuing What You Know In Your Heart You Were Meant To Do.Leroy Satchel Paige

Page 79 of 180

Civil Law Q&As (2007-2013)

hectorchristopher@yahoo.com

2007, Luciano filed suit to recover the land

dbaratbateladot@gmail.com

Non-Registrable Properties (2007)

he sold to Chua, alleging that the sale was


void

because

it

contravened

the

Constitution which prohibits the sale of


private lands to aliens. Julian moved to
dismiss the suit on grounds of pari delicto,
laches and acquisitive prescription. Decide

No.IV.

(B).

What

properties

are

not

registrable? (5%)
Supply this information.
SUGGESTED ANSWER:

the case with reasons. (4%)


The

following

properties

are

not

SUGGESTED ANSWER:

registrable:

The case must be dismissed. Julian, who

(1.) Properties of the Public dominion;

is a naturialized Filipino citizen and to


whom the property was allocated in a n
extra-judicial partition of the estate, is
now the owner of the property. The
defect in ownership of the property of

(2.) Properties for public use or public


service;
(3.)

Inalienable

lands

of

the

public

domain;

Julians alien father has already been


cured by its transfer to Julian. It has

(4.) Military installations, civil and quasi-

been validated by the transfer of the

public lands; and

property to a Filipino citizen. Hence,


there

is

no

more

violation

of

the

Constitution because the subject real


property is now owned by a Filipino

(5.) All lands not classified as alienable


and disposable.
ALTERNATIVE ANSWER:

citizen (Halili v. CA, 287 SCRA 465,


[1998]). Further, after the lapse of 35

(1).

Properties

year, laches has set in and the motion to

intended

dismiss may be granted, for the failure of

canals,

Luciano to question the ownership of

bridges constructed by the State, banks,

Chua before its transfer of ownership to

shores, roadsteads, and the like, are

Julian.

incapable of private appropriation, much

for

of

public

rivers,

public
use,

torrents,

dominium
like
ports

roads,
and

less registration (Art. 420 NCC). This

includes public markets, public plazas,

municipal streets and public buildings

Never Let The Odds Keep You From Pursuing What You Know In Your Heart You Were Meant To Do.Leroy Satchel Paige

Page 80 of 180

Civil Law Q&As (2007-2013)

hectorchristopher@yahoo.com

acquired by lease if not needed by the

(Municipality of Antipolo v. Zapanta, 133


SCRA 820, 1986; Martinez v. CA, 56
SCRA 647, 1974; Navera v. Quicho, 5
SCRA 454, 1962).
(2.) Lands proclaimed or classified as
forest, timberlands, mineral lands and
national parks. Under Sec 2, Art XII,
Constitution of the Philippines, these
lands are inalienable.
(3.) Lands that are reserved by law or
Presidential proclamation for military,
civic or quasi-public purpose, Under Sec
88, Chapter XII of the Public Land Act,
such lands shall be inalienable and shall
not be subject to occupation, entry, sale,
lease or other disposition.
(4.) In general, all lands of the public
domain that has not been classified as
alienable and disposable under the Public
Land Act.
(5.) Lands that form part of the seabed,
riverbed or lakebed. These lands are not
susceptible to private appropriation.
(6.) Foreshore lands is that strip of land
that lies between the high and low water
marks

and

alternately

dbaratbateladot@gmail.com

wet

and

dry

according to the flow of the tide belong


to the public domain, and can only be

government for public or quasi-public


purposes.

purposes, and religiously paid all taxes due


(7.) Lands reclaimed by the government
from the sea, lakes, or other bodies of
water are disposed or acquisible only by
lease

and

not

otherwise,

under

the

Public Land Act.

on the property. Anthony is not aware of the


defect in Bert's title, but has been in actual
physical possession of the property from
the time he bought it from Bert, who had
never been in possession of the property for
one year.
(A). Can Anthony acquire ownership of the

Prescription;

Acquisitive

Prescription

(2008)
No. VII. Anthony bought a piece of untitled
agricultural land from Bert. Bert, in turn,
acquired the property by forging carlo's
signature in a deed of sale over the
property. Carlo had been in possession of
the property for 8 years, declared it for tax

property by acquisitive prescription? How


many more years does he have possess it to
acquire ownership? (2%)
SUGGESTED ANSWER:
Yes, Anthony can acquire ownership of
the

property

prescription.

through
In

the

acquisitive

present

case,

Anthony is a buyer/possessor in good

Never Let The Odds Keep You From Pursuing What You Know In Your Heart You Were Meant To Do.Leroy Satchel Paige

Page 81 of 180

Civil Law Q&As (2007-2013)

hectorchristopher@yahoo.com

property once he recovers possession.

faith because he was not aware of the


defect in Bert's title (Art. 526, Civil
Code). As such, Anthony can acquire
ownership and other real rights over
immovable

property

through

open,

continuous possession of 10 years (Art.


1134, Civil Code). Anthony needs nine
(9) more years of possession, in addition
to his one (1) year of possession in good
faith.
(B).If Carlo is able to legally recover his
property,

can

he

require

Anthony

to

account for all the fruits he has harvested


from the property while in possession? (2%)
SUGGESTED ANSWER:
If Carlo is able to legally recover his
property, he cannot require Anthony to
account for all the fruits harvested from
the property. Anthony is entitled to the
fruits harvested in good faith before his
possession was legally interrupted (Art.
544, Civil Code).
(C).If there are standing crops on the
property when Carlo recovers possession,
can Carlo appropriate them? (2%)
SUGGESTED ANSWER:
Yes,

Carlos

can

dbaratbateladot@gmail.com

appropriate

only

portion of the standing crops on the

Anthony being a possessor in good faith,

shall have a right to a part of the

SUGGESTED ANSWER:

expenses of cultivation, and to a part of

Yes, it prescribes in five (5) years. If the

the net harvest of the standing crops,

real property mortgaged is judicially

both in proportion to the time of the

foreclosed,

possession (Art 545, Civil Code).

foreclosure should be filed within a

the

action

for

judicial

period of ten (10) years. The request for


issuance of a writ of possession should
Prescription; Judicially Foreclosed Real

be filed upon motion of the winning

Property Mortgage (2012)

bidder within five (5) years after the


judgment of foreclosure. The writ of

No.IX.a) Does the right to request for the


issuance of a writ of possession over a
foreclosed real property prescribe in five (5)
years? (5%)

possession is an order commanding the


sheriff to place a person named therein
in possession of real property (BPI v.
Icot. G.R. No. 168081, Oct 12, 2009).

Never Let The Odds Keep You From Pursuing What You Know In Your Heart You Were Meant To Do.Leroy Satchel Paige

Page 82 of 180

Civil Law Q&As (2007-2013)

hectorchristopher@yahoo.com

Purchaser in Good Faith; Mortgaged


Property (2008)
No. XIX. Juliet offered to sell her house and
lot, together with all the furniture and
appliances

therein

to

Dehlma.

Before

agreeing to purchase the property, Dehlma


went to the Register of Deeds to verify
Juliet's title. She discovered that while the
property was registered in Juliet's name
under

the

amended

Land

by

the

Registration
Property

Act,

as

Registration

Decree, it property, Dehlma told Juliet to


redeem the property from Elaine, and gave
her an advance payment to be used for
purposes of realesing the mortgage on the
property. When the mortgage was released,
Juliet executed a Deed of Absolute Sale over
the property which was duly registered with
the Registry of Deeds, and a new TCT was
issued

in

Dehlma's

name.

Dehlma

immediately took possession over the house


and

lot

and

the

movables

therein.

Thereafter, Dehlma went to theAssessor's


Office to get a new tax declaration under
her name. She was surprised to find out
that the property was already declared for
tax purposes in the name of XYZ Bank
which had foreclosed the mortgage on the
property before it was sold to her. XYZ Bank
was also the purchaser in the foreclosure
sale of the property. At that time, the
property was still unregistered but XYZ
Bank

registered

the

dbaratbateladot@gmail.com

Conveyance in the day book of the Register

Sheriff's

Deed

of

the claims of other persons that Dehlma


of Deeds under Act. 3344 and obtained a

bought the property. Thus, Dehlma is a


purchaser in good faith (Mathay v. CA,

tax declaration in its name.

G.R. No. 115788, 17 Sept, 1998).


(A). Was Dehlma a purchaser in good faith?
(2%)

(B). Who as between Dehlma and XYZ Bank


has a better right to the house and lot? (2%)

SUGGESTED ANSWER:

SUGGESTED ANSWER:
Yes, Dehlma is a purchaser in good faith.
Dehlma

Between Dehlma and XYZ Bank, Dehlma

bought the property, she went to the

has a better right to the house and lot.

Register of Deeds to verify Juliet's title.

After the release of the mortgage, the

When she discovered that the property

Deed of Absolute Sale was registered and

was mortgaged to Elaine, she gave an

a new title was issued in Dehlma's name.

advance payment so that Juliet could

Act 3344 is applicable exclusively to

release the mortgage. It was only after

instruments resulting from agreement of

the mortgage was released and free from

parties thereto and does not apply to

In

the

present

case,

before

deeds of a sheriff conveying to a

Never Let The Odds Keep You From Pursuing What You Know In Your Heart You Were Meant To Do.Leroy Satchel Paige

Page 83 of 180

Civil Law Q&As (2007-2013)

hectorchristopher@yahoo.com

purchaser unregistered lands sold to him


under execution (Williams v. Suer, 49
Phil. ,534).
(C). Who owns the movables inside the
house? (2%)
SUGGESTED ANSWER:
Dehlma

owns

the

movables

because

when she acquired the house and lot


from

Juliet,

all

the

furniture

and

appliances therein were included in the


sale. As owner of the real property,
Dehlma also owns the movables found
therein (Art. 542, Civil Code).

Registration; Governing Law (2007)


No.IV.

Bedrock

Development

Land

Corp.

is

&
a

Property

development

company engaged in developing and selling


subdivisions,

condominium

units

and

industrial estates. In order to replenish its


inventories, it embarked on an aggressive
land banking program. It employed "scouts"
who roam all over the Philippines to look for
and conduct investigations on prospective
sites

for

acquisition

dbaratbateladot@gmail.com

pertinent laws and regulations relating to

and

development,

whether developed, semi-developed or raw


land. The management of Bedrock asks you
as the company counsel to prepare a
manual containing a summary of the

description of the law.]


land registration and acquisition of title to
land.

The

manual

should

include

the

following items:

ALTERNATIVE ANSWER:
In general, the governing law relating to
registration and acquisition of title to

(A). What is the governing law? (5%)

land is Act 496 of 1902 as amended by


SUGGESTED ANSWER:
The

governing

law

PD 1529, otherwise known as Property


is

the

Land

Registration Decree of June 11, 1978.

Registration Act as amended by Property

(1.)

Registration Decree (Act 496 as amended

registration

by PD 1529).

Torrens System by voluntary ordinary

[Note: It is respectfully recommended

Chapter
of

III-I
land

governs
title

original

under

the

judicial proceedings.

that full credit be given to examinees

(2.) Chapter II-II governs compulsory

who did not give the exact title or

registration of lands through cadastral

number of the law but merely stated a

proceedings.

Never Let The Odds Keep You From Pursuing What You Know In Your Heart You Were Meant To Do.Leroy Satchel Paige

Page 84 of 180

Civil Law Q&As (2007-2013)

hectorchristopher@yahoo.com

(3.) Section 103 governs registration of


homestead, sales, free patent under CA
No. 141, as amended, otherwise known
as the Public Land Act.
(4.) Section 104 governs registration of
certificates

of

land

transfers,

emancipation patents and Certificates of


Land Ownership Award (CLOA) under
Comprehensive Land Reform Law.
(5.) Chapter V governs the registration of
land dealings on registered land like
conveyances,

transfers,

mortgages,

leases, powers of attorney, trusts and


similar contracts inter vivos.
(6.) Chapter V-II governs the registration
of involuntary dealings on registered
land like attachments, adverse claims,
enforcement of liens on registered land,
notices of lis pendens. (7.) Chapter VI
governs the registration of judgments,
orders and partitions, condemnation in
eminent domain proceedings, judicial
and extra-judicial settlement of estates.
(8.) Sections 107, 108 and 109 govern
petitions

and

registration

actions

after

original

like:

(a).Compulsory

surrender of withheld owner's duplicate


certificate of title;

(b)

Amendment

certificate of title;

dbaratbateladot@gmail.com

and

alteration

of

register a deed or when he is in doubt as


(c) Replacement of lost or destroyed
owner's duplicate certificate of title.
(9.)

R.A.

No.

reconstitution

26
of

governs
lost

or

R.A.

No.

6732

presented for registration.

judicial
destroyed

originals of the certificate of title.


(10.)

to what action to take on an instrument

Registration;

Party

Who

First

took

Possession (2013)

governs

No.IX.Rica petitioned for the annulment of

administrative reconstitution of lost or

her

destroyed original certificates of title.

Richard hired Atty. Cruz to represent him in

ten-year

old

marriage

to

Richard.

the proceedings. In payment for Atty. Cruz's


(11.)

Section

registration

of

113

governs

instruments

the

acceptance and legal fees, Richard conveyed

affecting

to Atty. Cruz a parcel of land in Taguig that

unregistered private lands.

he

recently

purchased

with

his

lotto

winnings. The transfer documents were


(12.) Section 117 governs "consultas,"

duly signed and Atty. Cruz

where the Register of Deeds refuses to

Never Let The Odds Keep You From Pursuing What You Know In Your Heart You Were Meant To Do.Leroy Satchel Paige

Page 85 of 180

Civil Law Q&As (2007-2013)

hectorchristopher@yahoo.com

object of the litigation, the annulment of

immediately took possession by fencing off


the property's entire perimeter.
Desperately needing money to pay for his
mounting legal fees and his other needs
and despite the transfer to Atty. Cruz,
Richard offered the same parcel of land for
sale to the spouses Garcia. After inspection
of the land, the spouses considered it a
good investment and purchased it from
Richard. Immediately after the sale, the
spouses

Garcia

commenced

the

construction of a three-story building over


the land, but they were prevented from
doing this by Atty. Cruz who claimed he has
a

better

right

in

light

of

the

prior

conveyance in his favor.


Is Atty. Cruz's claim correct? (8%)
SUGGESTED ANSWER:
No. Atty. Cruz is not correct. At first
glance, it may appear that Atty. Cruz is
the one who has the better right because
he first took possession of the property.
However, a lawyer is prohibited under
Art

1491

of

the

Civil

dbaratbateladot@gmail.com

Code

from

acquiring the property and rights which


may be the object of any litigation in
which they may take part by virtue of
their profession. While the suit is for
annulment of marriage and it may be
urged that the land itself is not the

marriage, if granted, will carry with it

any disposition or encumbrance of the


the

liquidation

of

the

absolute

community or conjugal partnership of


the spouses as the case may be (Art. 50
in relation to Art 43 of the Family Code).

property

of

the

community

or

the

conjugal property without the consent of


the other spouse is void (Art 96 and Art
124, Family Code).

Richard purchased the land with his


lotto winnings during the pendency of
the suit for annulment and on the
assumption
governed

that

the

parties

are

by the regime of absolute

community

or

conjugal

partnership,

Registration; Requisites; Proof (2013)


No.X. Manuel was born on 12 March 1940
in a 1 000-square meter property where he
grew

up

helping

his

father,

Michael,

winnings from gambling or betting will

cultivate the land. Michael has lived on the

form part thereof. Also, since the land is

property since the land was opened for

part

settlement

of

the

absolute

community

or

at

about

the

time

of

the

and

Commonwealth government in 1935, but

Rica, it may not be sold or alienated

for some reason never secured any title to

without the consent of the latter and

the property other than a tax declaration in

conjugal

partnership

of

Richard

his name. He has held the property through


Never Let The Odds Keep You From Pursuing What You Know In Your Heart You Were Meant To Do.Leroy Satchel Paige

Page 86 of 180

Civil Law Q&As (2007-2013)

hectorchristopher@yahoo.com

notorious

the years in the concept of an owner and


his stay was uncontested by others. He has
also conscientiously and continuously paid
the realty taxes on the land.
Michael died in 2000 and Manuel - as
Michaels only son and heir -now wants to
secure and register title to the land in his
own name. He consults you for legal advice
as he wants to perfect his title to the land
and secure its registration in his name.
(A) What are the laws that you need to
consider in advising Manuel on how he can
perfect his title and register the land in his
name? Explain the relevance of these laws
to your projected course of action. (4%)
SUGGESTED ANSWER:
(Note:

With

all

due

respect,

it

is

recommended that the examiner accept and


give full credit to any of the answers given in
each of the following paragraphs.)
I

would

advice

Manuel

to

file

an

application for registration under Sec 14


of Pres. Decree No. 1529, or the Property
Registration Decree (PRD), specifically
Sec14 (1) which requires (a) that the land
applied for forms part of the alienable
and disposable (A & D) portion of the
public domain, and (b) that the applicant
has

been

in

open,

continuous

and

dbaratbateladot@gmail.com

possession

and

occupation

thereof under bona fide claim of

ownership

since

June

12,

1945,

or

earlier. However, it is only necessary


that the land is already declared A & D
land at the time for application for
registration

is

filed

(Malabanan

v.

Republic, G.R. No. 180067, June 30,


2009).

confirmation of imperfect or incomplete


title

through

judicial

legalization

under Sec. 48 (b) of CA no. 141, or the


Public Land Act (PLA). But, as held in
Malabanan,

there

is

no

substantial

difference between this provision and


Sec 14 (1) of the PRD. Both refer to

Manuel could also invoke Sec 14 (2) of


the

Manuel could also file an application for

same

Decree,

which

allows

registration through ordinary acquisitive


prescription for thirty years, provided,
however, that the land is patrimonial
in character, i.e. already declared by the
government (a) as A & D, and (b) no
longer needed for public use or public
service (Malabanan, supra).

agricultural lands already classified as


alienable and disposable at the time the
application

is

filed,

and

require

possession and occupation since June


12, 1945. The only difference is that
under the PRD, there already exists a
title which is to be confirmed, whereas
under the PLA, the presumption is that
land is still public land (Republic v.

Never Let The Odds Keep You From Pursuing What You Know In Your Heart You Were Meant To Do.Leroy Satchel Paige

Page 87 of 180

Civil Law Q&As (2007-2013)

hectorchristopher@yahoo.com

Manuel's

Aquino, G.R. No. L-33983, January 27,


1983).

acquired under Rep. Act. No. 1942, dated


June 2, 1957, which amended Sec. 48 (b)
of the PLA by providing for a prescriptive
of

thirty

years

for

judicial

confirmation of imperfect title. It must


only be demonstrated that possession
and occupation commenced on January
24, 1947 and the 30-year period was
completed prior to the effectivity of PD
No. 1073 on January 25, 1977. PD No.
1073

now

occupation

requires
since

possession
June

12,

and
1945

(Republic v. Espinosa, G.R. No. 171514,


July 18, 2012).
Another alternative is for Manuel to
secure

title

through

administrative

proceedings under the homestead or free


patent provisions of the PLA. The title
issued has the same efficacy and validity
as

title

issued

through

judicial

proceedings, but with the limitations


that the land cannot be sold or disposed
of within five years from the issuance of
patent

(Sec.

118,

CA

No.

141,

as

amended).
(B) What do you have to prove to secure

and

documentation are necessary? (4%)


SUGGESTED ANSWER:

Manuel may also invoke vested rights

period

dbaratbateladot@gmail.com

objectives

what

valuable improvements like fencing the


Manuel has a the burden to overcome

land, constructing a residential house

the presumption of State ownership by

thereon,

well-nigh

planting fruit bearing trees, declaring

(Ong

v.

incontrovertible
Republic,

G.R.

evidence

No.

175746,

March 12, 2008). Accordingly, he must


show that ht eland is already classified
as A & D at the time the application for
registration is filed and that he has
been

in

possession

and

occupation

thereof in the manner required by law


since June 12, 1945, or earlier.
Manuel may tack his possession to that
of his predecessor-in-interest (Michael)
by the testimony of disinterested and
knowledgeable eyewitnesses. Overt acts
of possession may consist in introducing

the

land

cultivating

for

the

taxation

land

purposes

and

and

paying realty taxes, all of which are


corroborative proof of possession.
To identify the land, he must submit the
tracing cloth plan or a duly-certified
blueprint

or whiteprint

copy

thereof

(Director of Lands v. Reyes, G.R. No. L27594, November 28, 1975; Director of
Lands v. CA and Iglesia ni Cristo, G.R.
No. L-56613, March 14, 1988).
To show the classification of the land as
A

&

D,

the

application

must

be

accompanied by (1) a CENRO or PENRO

Never Let The Odds Keep You From Pursuing What You Know In Your Heart You Were Meant To Do.Leroy Satchel Paige

Page 88 of 180

Civil Law Q&As (2007-2013)

hectorchristopher@yahoo.com

certification; and (2) a certified true


copy

of

approved

the
by

original
the

classification

DENR

Secretary

(Republic v. Bantigue, G.R.No. 162322,


March

14,

2012).

presidential

or

legislative act may also be considered.

Remedies; Fraud; Rights of Innocent


Purchaser (2009)
No.IX. Before migrating to Canada in 1992,
the spouses Teodoro and Anita entrusted
all their legal papers and documents to
their nephew, Atty. Tan. Taking advantage
of the situation, Atty. Tan forged a deed of
sale, making it appear that he had bought
the couples property in Quezon City. In
2000, he succeeded in obtaining a TCT over
the property in his name. Subsequently,
Atty. Tan sold the same property to Luis,
who built an auto repair shop on the
property. In 2004, Luis registered the deed
of conveyance, and title over the property
was transferred in his name.
In 2006, the spouses Teodoro and Anita
came to the Philippines for a visit and
discovered what had happened to their
property. They immediately hire you as
lawyer. What action or actions will you
institute in order to vindicate their rights?
Explain fully. (4%)

dbaratbateladot@gmail.com

SUGGESTED ANSWER:

(c). A criminal action for forgery or


falsification of public document;
I will institute the following actions
against Atty. Tan:

(d).

complaint

with

the

Supreme

Court/Integrated Bar of the Philippines


(a). A civil action for damage for the

to

disbar

or

suspend

him

or

other

fraudulent transfer of the title in his

disciplinary action for violation or the

name and to recover the value of the

Code of Professional Ethics.

property;
Any action against Luis will not prosper
National

because he is an innocent purchaser for

Treasurer for compensation from the

value. The Title to the land he bought

State Assurance Fund which is set aside

was already in the name of the person

by law to pay those who lose their land

who sold the property to him, and there

suffer damages as a consequence of the

is nothing on the title which will make

operation of the Torrens system;

him suspect about the fraud committed

(b).

An

action

against

the

by Atty. Tan.

Never Let The Odds Keep You From Pursuing What You Know In Your Heart You Were Meant To Do.Leroy Satchel Paige

Page 89 of 180

Civil Law Q&As (2007-2013)

hectorchristopher@yahoo.com

Contracts

dbaratbateladot@gmail.com

Rescission of Contract; Fortuitous Event


(2008)

Contract to Sell vs. Conditional Contract


of Sale (2012)

No.XVIII. AB Corp. entered into a contract


with XY Corp. whereby the former agreed to

No.X.a) A contract to sell is the same as a

construct

the

research

and

laboratory

conditional contract of sale. Do you agree?

facilities of the latter. Under the terms of

Explain your answer. (5%)

the contract, AB Corp. agreed to complete


the facility in 18 months, at the total

SUGGESTED ANSWER:

contract price of P10 million. XY Corp. paid

No. A contract to sell is a species of


conditional sale. The contract to sell
does not sell a thing or property; it sells
the right to buy property. A conditional

50% of the total contract price, the balance


to be paid upon completion of the work. The
work stated immediately, but AB Corp. later
experienced work slippage because of labor
unrest

in

his

company.

AB

Corp.'s

sale is a sale subject to the happening or

employees claimed that they are not being

performance of a condition, such as

paid on time; hence, the work slowdown. As

payment of the full purchase price, or

of the 17th month, work was only 45%

the performance of other prestation to

completed. AB Corp. asked for extension of

give, to do or not to do. Compliance with

time, claiming that its labor problems is a

the condition automatically gives the

case of fortuitous event, but this was

right to the vendee to demand the


delivery of the object of the sale. In a
contract to sell, however, the compliance
with

the

condition

does

not

denied by XY Corp. When it became certain


that the contruction could not be finished
on time, XY Corp. sent written notice
cancelling the contract, and requiring AB
Corp. to immediately vacate the premises.

automatically sell the property to the


vendee. It merely gives the vendee the

(A). Can the labor unrest be considered a

right to compel the vendor to execute

fortuitous event? (1%)

the deed of absolute sale.

SUGGESTED ANSWER:
No.

The

labor

unrest

cannot

be

considered a fortuitous event under Art.

1174 of the Civil Code. A fortuitous

event should occur independent of the

Never Let The Odds Keep You From Pursuing What You Know In Your Heart You Were Meant To Do.Leroy Satchel Paige

Page 90 of 180

Civil Law Q&As (2007-2013)

will

of

the

debtor

hectorchristopher@yahoo.com

or

without

his

participation or aggravation (Paras, Civil


Code Annotated, vol. IV, 2000 ed., p
159). As mentioned in the facts, labor
unrest of the employees was caused by
AB Corp.'s failure to pay its employees
on time.
(B).

Can

XY

Corp.

unilaterrally

and

immediately cancel the contract? (2%)


SUGGESTED ANSWER:
No, XY Corp. cannot unilaterally and
immediately cancel the contract. In the
absence of any stipulation for automatic
rescission, rescission must be judicial
(Art. 1191, Civil Code).
(C).

Must

AB

Corp.

dbaratbateladot@gmail.com

the statement is true, or FALSE if the

return

the

50%

downpayment? (2%)
SUGGESTED ANSWER:
AB Corp. need not return the 50% down
payment because 45% of the work was
already completed, otherwise, XY Corp.
would be unjustly enriching itself at the
expense of AB Corp.

Stipulation; Arbitration Clause (2009)


No. XI. TRUE or FALSE. Answer TRUE if

No.XV. Sarah had a deposit in a savings


statement is false. Explain your answer in
not more than two (2) sentences.
(A). A clause in an arbitration contract
granting one of the parties the power to
choose more arbitrators than the other
renders the arbitration contract void. (1%)
SUGGESTED ANSWER:
True. The Civil Code provides that Any
clause giving one of the parties power to
choose more arbitrators than the other
is void and of no effect (Art 2045, NCC).

Obligations

account with Filipino Universal Bank in the


amount

of

five

million

pesos

(P5,000,000.00). To buy a new car, she


obtained a loan from the same bank in the
amount of P1,200,000.00, payable in twelve
monthly installments. Sarah issued in favor
of the bank post-dated checks, each in the
amount of P100,000.00, to cover the twelve
monthly installment payments. On the
third,

fourth

and

fifth

months,

the

corresponding checks bounced.


The

bank

then

declared

the

whole

obligation due, and proceeded to deduct the


amount of one million pesos

Extinguishment; Compensation (2009)

(P1,000,000.00) from Sarahs deposit after


notice to her that this is a form of

Never Let The Odds Keep You From Pursuing What You Know In Your Heart You Were Meant To Do.Leroy Satchel Paige

Page 91 of 180

Civil Law Q&As (2007-2013)

hectorchristopher@yahoo.com

compensation allowed by law. Is the bank

dbaratbateladot@gmail.com

Extinguishment; Compensation (2008)

correct? Explain. (4%)


No. XV. Eduardo was granted a loan by XYZ
SUGGESTED ANSWER:

Bank for the purpose of improving a

No, the bank is not correct. While the

building

Bank is correct about the applicability of

Eduardo, executed the promissory note

compensation, it was not correct as to

("PN") in favor of the bank, with his friend

the amount compensated.

Recardo as co-signatory. In the PN, they


both

which

XYZ

acknowledged

leased

that

from

him.

they

are

A bank deposit is a contract of loan,

"individually and collectively" liable and

where the depositor is the creditor and

waived the need for prior demand. To

the bank the debtor. Since Sarah is also

secure the PN, Recardo executed a real

the debtor of the bank with respect to


the loan, both are mutually principal
debtors and creditors of each other. Both
obligation

are

due,

demandable

and

liquidated but only up to the extent of

estate mortgage on his own property. When


Eduardo defaulted on the PN, XYZ stopped
payment of rentals on the building on the
ground that legal compensation had set in.
Since there was still a balance due on the
PN

after

applying

the

rentals,

XYZ

P300,000.00 (covering the unpaid third,

foreclosed the real estate mortgage over

fourth and fifth monthly installments).

Recardo's property. Recardo opposed the

The entire one million was not yet due

foreclosure on the ground that he is only a

because the loan has no acceleration

co-signatory; that no demand was made

clause in case of default. And since there

upon him for payment, and assuming he is

is

liable, his liability should not go beyond

no

retention

commenced

by

or

third

controversy
person

and

communicated in due time to the debtor,


then

all

the

requisites

of

legal

compensation are present but only up to


the amount of P300,000.00. The bank,
therefore, may deduct P300,000.00 from
Sarahs

bank

deposit

by

way

of

half the balance of the loan. Further,


Recardo said that when the bank invoked
compensation between the reantals and the
amount of the loan, it amounted to a new
contract or novation, and had the effect of
extinguishing the security since he did not
give his consent (as owner of the property
under the real estate mortgage) thereto.

compensation.
(A). Can XYZ Bank validly assert legal

compensation? (2%)

Never Let The Odds Keep You From Pursuing What You Know In Your Heart You Were Meant To Do.Leroy Satchel Paige

Page 92 of 180

Civil Law Q&As (2007-2013)

hectorchristopher@yahoo.com

Eduardo defaulted on the PN, XYZ stopped

SUGGESTED ANSWER:

payment of rentals on the building on the

Yes, XYZ Bank can validly assert legal


compensation. In the present case, all of
the elements of legal compensation are
present: (1) XYZ Bank is the creditor of
Eduardo while Eduardo is the lessor of
XYZ Bank; (2) both debts consist in a
sum of money, or if the things due are
consumable, they be of the same kind,
and also of the same quality if the latter
has been stated; (3) the two debts be
due;

(4)

they

be

liquidated

and

demandable, and (5) over neither of them


there be any retention or controversy,
commenced

by

third

persons

and

communicated in due time to the debtor


(Art. 1279, Civil Code).

Extinguishment; Novation (2008)


No. XV. Eduardo was granted a loan by XYZ
Bank for the purpose of improving a
building

which

XYZ

leased

from

him.

Eduardo, executed the promissory note


("PN") in favor of the bank, with his friend
Recardo as co-signatory. In the PN, they
both

acknowledged

dbaratbateladot@gmail.com

estate mortgage on his own property. When

that

they

are

"individually and collectively" liable and


waived the need for prior demand. To
secure the PN, Recardo executed a real

ground that legal compensation had set in.


Since there was still a balance due on the
PN

after

applying

the

rentals,

(C). Does Recardo have basis under the


Civil Code for claiming that the original
contract was novated? (2%)

XYZ

foreclosed the real estate mortgage over

SUGGESTED ANSWER:

Recardo's property. Recardo opposed the


foreclosure on the ground that he is only a

No. Recardo has no basis for claiming

co-signatory; that no demand was made

novation of the original contract when

upon him for payment, and assuming he is

the bank invoked compensation because

liable, his liability should not go beyond

there was simply partial compensation

half the balance of the loan. Further,

(Art. 1290, Civil Code) and this would

Recardo said that when the bank invoked

not bar the bank from recovering the

compensation between the reantals and the


amount of the loan, it amounted to a new
contract or novation, and had the effect of

remaining balance of the obligation.


ALTERNATIVE ANSWER:

extinguishing the security since he did not


give his consent (as owner of the property

No. In order that an obligation may be

under the real estate mortgage) thereto.

extinguished by another, it is imperative

Never Let The Odds Keep You From Pursuing What You Know In Your Heart You Were Meant To Do.Leroy Satchel Paige

Page 93 of 180

Civil Law Q&As (2007-2013)

hectorchristopher@yahoo.com

may be refused receipt by the oblige

that it be so declared in unequivocal


terms,

or

that

the

old

and

new

obligations be on every point compatible


with

each

other.

Novation

is

never

presumed (Art. 1292, Civil Code).

Extinguishment;

dbaratbateladot@gmail.com

Payment

of

Check

(2013)
No.VI. Lito obtained a loan of P1,000,000
from Ferdie, payable within one year. To
secure payment, Lito executed a chattel
mortgage on a Toyota Avanza and a real
estate mortgage on a 200-square meter
piece of property.
(B) Lito's failure to pay led to the extrajudicial foreclosure of the mortgaged real
property. Within a year from foreclosure,
Lito tendered a manager's check to Ferdie to
redeem the property. Ferdie refused to
accept payment on the ground that he
wanted payment in cash: the check does
not qualify as legal tender and does not
include the interest payment. Is Ferdie's
refusal justified? (4%)
SUGGESTED ANSWER:
A check, whether a managers check or
an ordinary check is not legal tender,
and an offer of a check in payment of a
debt is not a valid tender of payment and

is sufficient to compel redemption but is


or creditors (Philippine Airlines v. CA
and

Amelia

Tan,

G.R.

No.

L-49188,

1990). Mere delivery of checks does not


discharge

the

obligation

under

judgment. A check shall produce the


effect of payment only when they have
been cashed or where through the fault
of the creditor they have been impaired
(Art 1249, Civil Code).

right of redemption be exercised by


delivery of legal tender. A check may be
for

the

exercise

redemptioner from his liability to pay


the redemption price (Biana v. Gimenez,
G.R. No. 132768, Sept 9, 2005, citing
Fortunado v. CA).
Redemption within the period allowed by
law is not a matter of intent but a
question of payment or valid tender of
full redemption prices within the said

However, it is not necessary that the

used

not in itself a payment that relieves the

of

right

of

redemption, the same being a right and


not an obligation. The tender of a check

period. Whether redemption is being


made under Art. 3135 or under the
General Banking Law, the mortgagor or
his

assignee

is

required

to

tender

payment to make said redemption valid


(Heirs of Quisumbing v. PNB and SLDC,
G.R. No. 178242, Jan 20, 2009).

Never Let The Odds Keep You From Pursuing What You Know In Your Heart You Were Meant To Do.Leroy Satchel Paige

Page 94 of 180

Civil Law Q&As (2007-2013)

hectorchristopher@yahoo.com

Moreover, Ferdies refusal was justified


on the ground that the amount tendered
does not include interest. In order to
effect the redemption of the foreclosed
property, the payment to the purchaser
must include the following sums: (a) the
bid price; (b) the interest on the bid
price, computed at one per centum (1%)
per month; and (c) the assessments and
taxes, if any, paid by the purchaser with
the same rate of interest (Sec 28, 1997
Rules of Civil Procedure). Unless there is
an express stipulation to that effect, the
creditor cannot be compelled to receive
partial payment of the prestation (Art.
1248, Civil Code).

Extinguishment;

Payment

of

Check;

Legal Tender (2008)


No.

XVII.

Felipe

borrowed

dbaratbateladot@gmail.com

Felipe to pay him in cash the amount of

$100

from

Gustavo in 1998, when the Phil P - US$


exchange rate was P56 - US$1. On March
1, 2008, Felipe tendered to Gustavo a
cashier's check in the amount of P4,135 in
payment of his US$ 100 debt, based on the
Phil P - US$ exchange rat at that time.
Gustavo accepted the check, but forgot to
deposit it until Sept. 12, 2008. His bank
refused to accepted the check because it
had become stale. Gustavo now wants

P5,600.

Claiming

that

the

previous

payment was not in legal tender, and that

check became stale. He is now estopped


there

has

been

extraordinary

deflation

since 1998, and therefore, Felipe should


pay him the value of the debt at the time it
was incurred. Felipe refused to pay him
again, claiming that Gustavo is estopped
from raising the issue of legal tender,

from raising the issue that a cashier's


check is not legal tender.
(B). Can Felipe validly refuse to pay Gustavo
again? (2%)
SUGGESTED ANSWER:

having accepted the check in March, and


that it was Gustavo's negligence in not

Yes, Felipe can refuse to pay Gustavo,

depositing

who allowed the check to become stale.

the

check

immediately

that

caused the check to become stale.


(A). Can Gustavo now raised the issue that
the cashier's check is not legal tender? (2%)

Although a check is not legal tender


(Belisario v. Natividad. 60 Phil 156),
there

are

instances

when

check

produces the effects of payment, for


SUGGESTED ANSWER:
No. Gustavo previously accepted a check
as payment. It was his fault why the

example: (a) when the creditor is in


estoppel or he had previously promised
he would accept a check (Paras, Civil

Never Let The Odds Keep You From Pursuing What You Know In Your Heart You Were Meant To Do.Leroy Satchel Paige

Page 95 of 180

Civil Law Q&As (2007-2013)

hectorchristopher@yahoo.com

Code Annotated, Vol IV, 2000 ed., p.


394); (b) when the check has lost its
value because of the fault of the creditor
(Art. 1249, 2nd par.),as when he was
unreasonably delayed in presenting the
check for payment (PNB v. Seeto, G.R.
No, L-4388, 13 August 1952).
(C). Can Felipe compel Gustavo to receive
US$100 instead? (1%)
SUGGESTED ANSWER:
Felipe cannot compel Gustavo to receive
US$100 because under RA 529, payment
of loans should be at Philippine currency
at the rate of exchange prevailing at the
time of the stipulated date of payment.
Felipe could only compel Gustavo to
receive US$ 100 if they stipulated that
obligation be paid in foreign currency
(R.A. 4100).

Liability; Solidary Liability (2008)


No. XV. Eduardo was granted a loan by XYZ
Bank for the purpose of improving a
building

which

XYZ

leased

from

him.

Eduardo, executed the promissory note


("PN") in favor of the bank, with his friend
Recardo as co-signatory. In the PN, they
both

acknowledged

that

dbaratbateladot@gmail.com

"individually and collectively" liable and

they

are

waived the need for prior demand. To

contract or novation, and had the effect of


secure the PN, Recardo executed a real
estate mortgage on his own property. When
Eduardo defaulted on the PN, XYZ stopped
payment of rentals on the building on the
ground that legal compensation had set in.

extinguishing the security since he did not


give his consent (as owner of the property
under the real estate mortgage) thereto.
(B). Can Recardo's property be foreclosed to
pay the full balance of the loan? (2%)

Since there was still a balance due on the


PN

after

applying

the

rentals,

XYZ

SUGGESTED ANSWER:

foreclosed the real estate mortgage over


Recardo's property. Recardo opposed the

Yes, Recardo's property can be foreclosed

foreclosure on the ground that he is only a

to pay the full balance of the loan

co-signatory; that no demand was made

because when he signed as co-signatory

upon him for payment, and assuming he is

in the promissory note, he acknowledged

liable, his liability should not go beyond

he is solidarily liable with Eduardo. In

half the balance of the loan. Further,


Recardo said that when the bank invoked
compensation between the reantals and the
amount of the loan, it amounted to a new

solidary obligations, a creditor has the


right to demand full payment of the
obligation from any of

Never Let The Odds Keep You From Pursuing What You Know In Your Heart You Were Meant To Do.Leroy Satchel Paige

Page 96 of 180

Civil Law Q&As (2007-2013)

hectorchristopher@yahoo.com

takes

the solidary debtors (Art. 1207, Civil


Code).

Obligations; Without Agreement (2007)


No.V. What are obligations without an
agreement"?
situations

Give

giving

five
rise

to

examples

of

this

of

type

obligations? (10%)
SUGGESTED ANSWER:
"Obligations without an agreement" are
obligations

that

do

not

arise

from

contract such as those arising from: 1.


delicts;

2.

quasi-delicts;

3.

solutio

indebiti; 4. negotiorum gestio; and 5. all


other obligations arising from law.
ALTERNATIVE ANSWER:
"Obligations without an agreement" refer
to the juridical relation of quasi-contract
which

arise

from

certain

lawful,

voluntary and unilateral acts to the end


that no one shall be unjustly enriched or
benefited at the expense of another. (Art.
2142, NCC)
First Example of an obligation without
an agreement is a case of negotiorum
gestio,

whereby

one

who

voluntarily

charge

dbaratbateladot@gmail.com

of

the

agency

or

management of the business or property


of another without any power from the

knowledge of the person obliged to give


latter, is obliged to continue the same
until the termination of the affair and its
incidents,

or

to

require

the

person

concerned to substitute him, if the


owner is in a position to do so (Art.

support, it is given by a stranger, the


latter shall have a right to claim the
same from the former, unless it appears
that he gave it out of piety and without
intention of being repaid (Art. 2164,
NCC).

2144, NCC).
Second

example,

indebiti

may

also

case
give

of
rise

solutio
to

an

obligation without an agreement. This


refers to the obligation to return which
arises when something is received when
there is no right to demand it, and it was
unduly delivered through mistake (Art.
2154, NCC).

Fourth

example,

is

when

through

accident or other causes a person is


injured or becomes seriously ill, and he
is treated or helped while he is not in a
condition to give consent to a contract,
he shall be liable to pay for the services
of the physician or other person aiding
him,

unless

the

service

has

been

rendered out of pure generosity (Art.


Third example, is when without the

2167, NCC).

Never Let The Odds Keep You From Pursuing What You Know In Your Heart You Were Meant To Do.Leroy Satchel Paige

Page 97 of 180

Civil Law Q&As (2007-2013)

hectorchristopher@yahoo.com

a trust against one who, by fraud, duress

Fifth instance of an obligation without


an agreement is when the person obliged
to support an orphan or an insane or
other indigent person unjustly refuses to
give support to the latter, any third
person may furnish support to the needy
individual, with right of reimbursement
from the person obliged to give support.
The provisions of this article apply when
the father or mother of a child under
eighteen years of age unjustly refuses to
support him (Art. 2166, NCC).

Trust
Trust De Son Tort (2007)
No.III. Explain the following concepts and
doctrines and give an example of each:
(A).

concept

of

trust de

son

tort (constructive trust) (5%)


SUGGESTED ANSWER:
A constructive trust is a trust NOT
created by any word or phrase, either
expressly or impliedly, evincing a direct
intention to create a trust, but is one
that

arises

in

order

dbaratbateladot@gmail.com

to

satisfy

the

demands of justice. It does not come


about by agreement or intention but
mainly operation of law and construed as

or abuse of confidence, obtains or holds


the legal right to property which he

provides: "If an absolute conveyance of


ought

not,

in

equity

and

good

conscience, to hold (Heirs of Lorenzo Yap


v. CA, 371 Phil 523, 1991). The following
are examples of constructive trust: 1.
Art.

1456

NCC

which

provides:

"If

property is acquired through mistake or


fraud, the person obtaining it is, by force
of law considered a trustee of an implied
trust for the benefit of the person for
whom the property comes." 2. Art 1451
NCC which provides: "When land passes
by succession through any person and he
causes the legal title to be put in the
name of another, a trust is established
by implication of law for the benefit of
the true owner." 3. Art 1454 NCC which

property is made in order to secure the


performance of an obligation of the
grantor toward the grantee, a trust by
virtue

of

law

is

established.

If

the

fulfillment of the obligation is offered by


the grantor when it becomes due, he
may demand the reconveyance of the
property to him." 4. Art 1455 NCC which
provides: "When any trustee, guardian or
any

person

holding

fiduciary

relationship uses trust funds for the


purchase

of

property

and

causes

conveyance to be made to him or to


third person, a trust is established by
operation of law in favor of the person to
whom the funds belong."

Never Let The Odds Keep You From Pursuing What You Know In Your Heart You Were Meant To Do.Leroy Satchel Paige

Page 98 of 180

Civil Law Q&As (2007-2013)

hectorchristopher@yahoo.com

In order to secure the financial capital, she

Sales
Condominium

Act;

Partition

of

Condominium (2009)
No.XVIII.

The

Ifugao

Arms

is

condominium project in Baguio City. A


strong earthquake occurred which left huge
cracks in the outer walls of the building. As
a result, a number of condominium units
were rendered unfit for use. May Edwin,
owner of one of the condominium units
affected, legally sue for partition by sale of
the whole project? Explain. (4%)
SUGGESTED ANSWER:
Yes, Edwin may legally sue for partition
by

sale

of

the

whole

condominium

project under the following conditions:


(a) the damage or destruction caused by
the earthquake has rendered one-half
(1/2)

or

more

of

the

untenantable,

and

condominium

owners

(b)

units

therein

that
holding

the
an

aggregate of more than thirty percent


(30%) interests of the common areas are
opposed

to

the

dbaratbateladot@gmail.com

business of buying and selling large cattle.

restoration

of

the

condominium project (Sec 8 [b], Republic


Act No. 472 Condominium Act).

Mortgage; Equitable Mortgage (2012)


No.VI. (b) Eulalia was engaged in the

no
advanced for her employees (biyaheros). She
required them to surrender TCT of their
properties and to execute the corresponding
Deeds

of

Sale

in

her

favor.

Domeng

Bandong was not required to post any


security but when Eulalia discovered that
he incurred shortage in cattle procurement
operation, he was required to execute a

sale

intended

but

only

equitable

mortgage for the purpose of securing the


shortage

incurred

by

Domeng

in

the

amount of P 70, 000.00 while employed as


"biyahero" by Eulalia. Was the Deed of Sale
between Domeng and Eulalia a contract of
sale or an equitable mortgage? Explain.
(5%)
SUGGESTED ANSWER:

Deed of Sale over a parcel of land in favor of


Eulalia. She sold the property to her

The contract between Domeng Bandong

grandneice

thereafter

and Eulalia was an equitable mortgage

instituted an action for ejectment against

rather than a contract of sale. The

the Spouses Bandong.

purported deed of sale was actually

Jocelyn

who

To assert their right, Spouses Bandong filed


an action for annulment of sale against
Eulalia and Jocelyn alleging that there was

intended to merely secure the payment


of the shortage incurred by Domeng in
the

conduct

of

the

cattle-buying

operations. Under Art 1602, Civil Code,

Never Let The Odds Keep You From Pursuing What You Know In Your Heart You Were Meant To Do.Leroy Satchel Paige

Page 99 of 180

Civil Law Q&As (2007-2013)

hectorchristopher@yahoo.com

the contract shall be presumed to be an


equitable mortgage when it may be fairly
inferred that the real intention of the
parties is simply to secure the payment
of a debt or the performance of any
other obligation. The present transaction
was clearly intended to just secure the
shortage incurred by Eulalia because
Bandung remained in possession of the
property inspite of the execution of the
sale.

Option Contract; Liquor & Pulutan as


Consideration (2013)
No.III.Sergio is the registered owner of a
500-square meter land. His friend, Marcelo,
who

has

long

been

interested

in

the

property, succeeded in persuading Sergio to


sell it to him. On June 2, 2012, they agreed
on the purchase price of P600,000 and that
Sergio would give Marcelo up to June30,
2012 within which to raise the amount.
Marcelo, in a light tone usual between
them, said that they should seal their
agreement through a case of Jack Daniels
Black and P5,000 "pulutan" money which
he immediately handed to Sergio and which
the latter accepted. The friends then sat
down and drank the first bottle from the
case of bourbon.

dbaratbateladot@gmail.com

On June 15, 2013, Sergio learned of


another buyer, Roberto, who was

offering P800,000 in ready cash for the


land. When Roberto confirmed that he
could pay in cash as soon as Sergio could
get

the

documentation

ready,

Sergio

decided to withdraw his offer to Marcelo,

Yes. Marcelo has a cause of action


against Sergio.
Under Art. 1324, when the offerer has
allowed the offeree a certain period to

hoping to just explain matters to his friend.

accept, the offer may be withdrawn at

Marcelo,

any

however,

objected

when

the

time

before

acceptance

by

withdrawal was communicated to him,

communicating such withdrawal, except

taking the position that they have a firm

when

and binding agreement that Sergio cannot

consideration,

simply walk away from because he has an

promised.

the

option
as

is

founded

something

upon

paid

or

option to buy that is duly supported by a


duly accepted valuable consideration.

An accepted unilateral promise to buy or


sell a determinate thing for a price

(A) Does Marcelo have a cause of action


against Sergio? (5%)
SUGGESTED ANSWER:

certain is binding upon him if the


promise is supported by a consideration
distinct from the price (Art. 1479).

Never Let The Odds Keep You From Pursuing What You Know In Your Heart You Were Meant To Do.Leroy Satchel Paige

Page 100 of 180

Civil Law Q&As (2007-2013)

hectorchristopher@yahoo.com

Art. 1403 does not apply.

Consideration in an option contract may


be anything of value,, unlike in sale
where it must be the price certain in
money or its equivalent (San Miguel
Properties Inc. v. Spouses Huang, G.R.
No. 137290, July 31, 2000).
Here, the case of Jack Daniels Black and
the P5,000.00 pulutan money was a
consideration to seal their agreement,
an agreement that Marcelo is given until
June 30, 2012 to buy the parcel of land.
There is also no showing that such
consideration will be considered part of
the

purchase

unilateral

price.

withdrawal

dbaratbateladot@gmail.com

Thus,
of

Sergios

the

offer

violated the Option Contract between


him and Marcelo.
(B) Can Sergio claim that whatever they
might have agreed upon cannot be enforced
because any agreement relating to the sale
of real property must be supported by
evidence in writing and they never reduced
their agreement to writing? (3%)
SUGGESTED ANSWER:
No. Sergios claim has no legal basis.
The contract at issue in the present case
is the option contract, not the contract
of sale for the real property. Therefore,

The

Statute

of

Frauds

covers

an

agreement for the sale of real property


or

of

an

interest

therein.

Such

agreement is unenforceable by action,


unless

the

same,

or

some

note

or

memorandum, thereof, be in writing,


(Art. 1403 (e), Civil Code). Here, Marcelo
and

Sergio

Option

merely

Contract,

entered

which

into

refers

an

to

unilateral promise to buy or sell, which

ALTERNATIVE ANSWER:
No. Sergios claim has no legal basis.
The contract of sale has already been
partially executed which takes it outside
the ambit of the Statute of Frauds is
applicable only to executory contracts,
not to contracts that are totally or
partially performed (Carbonnel v. Poncio,
G.R. No. L-11231, May 12, 1958).

need not be in writing to be enforceable


(Sanchez v. Rigos, G.R. No. L-25494,
June 14, 1972, citing Atkins, Kroll and
Co.

Inc.

v.

Cua

Hian

Tek

and

Southwestern Sugar & Molasses Co. v.


Atlantic Gulf & Pacific Co.).

Right of First Refusal; Lessee; Effect


(2008)
No.XVI. Dux leased his house to Iris for a
period of 2 years, at the rate of P25,000.00

Never Let The Odds Keep You From Pursuing What You Know In Your Heart You Were Meant To Do.Leroy Satchel Paige

Page 101 of 180

Civil Law Q&As (2007-2013)

hectorchristopher@yahoo.com

would make her privy to the agreement

monthly, payable annually in advance. The


contract stipulated that it may be renewed
for another 2-year period upon mutual
agreement of the parties. The contract also
granted Iris the right of first refusal to
purchase the property at any time during
the lease, if Dux decides to sell the property
at the same price that the property is
offered for sale to a third party. Twentythree months after execution of the lease
contract, Dux sold breach of her right of
first refusal. Dux said there was no breach
because the property was sold to his
mother who is not a third party. Iris filed an
action to rescind the sale and to compel
Dux to sell the property to her at the same
price. Alternatively, she asked the court to
extend the lease for another 2 years on the
same terms.
(A). Can Iris seek rescission of the sale of
the property to Dux's mother? (3%)
SUGGESTED ANSWER:
Yes, because the right of first refusal is
included in the contract signed by the
parties. Only if the lessee failed to
exercise the right of first refusal could
the

lessor

lawfully

sell

the

subject

property to others, under no less than


the

same

terms

and

dbaratbateladot@gmail.com

conditions

previously offered to the lessee. Granting


that the mother is not a third party, this

of Dux and Iris, aware of the right of first

refusal. This makes the mother a buyer


in bad faith, hence giving more ground
for

rescission

(Equatorial

of

Realty,

the
et

sale
al.

v.

to

her

Mayfair

Theater, G.R. No. 106063, 21 Nov. 1996).

No. The contract stipulated that it may


be renewed for another 2-year period
upon mutual agreement of the parties.
Contracts

are

binding

between

the

parties; validity or compliance cannot be


left to the will of one of the parties (Art.

ALTERNATIVE ANSWER:

1308, Civil Code).

No, Iris cannot seek rescission of the

ALTERNATIVE ANSWER:

sale of the property to Duxs mother


because the sale is not one of those
rescissible contracts under Art. 1381 of
the Civil Code.

It depends. The alternative prayer for


the extension of the lease may prosper if
(a) there is a stipulation in the contract
of sale; (b) Dux's mother is aware of the

(B). Will the alternative prayer for extension

existing contract of lease; or (c) the lease

of the lease prosper? (2%)

is recorded in the Registry of Property

SUGGESTED ANSWER:

(Art. 1676, Civil Code).

Never Let The Odds Keep You From Pursuing What You Know In Your Heart You Were Meant To Do.Leroy Satchel Paige

Page 102 of 180

Civil Law Q&As (2007-2013)

hectorchristopher@yahoo.com

Lease
Builder;

Good

dbaratbateladot@gmail.com

reimbursed the value of the improvements


he introduced. (4%)

Faith;

Useful

Improvements (2013)
No.IV.Anselmo is the registered owner of a
land and a house that his friend Boboy

SUGGESTED ANSWER:
Boboys claim that he is a builder in
good faith has no basis. A builder in good

occupied for a nominal rental and on the

faith

condition that Boboy would vacate the

property in concept of an owner. The

property

provisions

on

demand.

With

Anselmo's

is

someone

on

who

occupies

the

builder-planter-sower

knowledge, Boboy introduced renovations

under the Civil Code cover cases in

consisting of an additional bedroom, a

which the builder, planter and sower

covered veranda, and a concrete block

believe themselves to be owners of the

fence, at his own expense.

land, or at least, to have a claim of title

Subsequently, Anselmo needed the property


as his residence and thus asked Boboy to
vacate and turn it over to him. Boboy,
despite an extension, failed to vacate the

thereto.
As Boboy is a lessee of the property,
even if he was paying nominal rental,

property, forcing Anselmo to send him a

Art. 1678, Civil Code, is applicable.

written demand to vacate.

Under this provision, if the lessee makes,


in

In his own written reply, Boboy signified


that he was ready to leave but Anselmo
must first reimburse him the value of the
improvements

he

introduced

on

the

property as he is a builder in good faith.


Anselmo

refused,

insisting

that

Boboy

cannot ask for reimbursement as he is a

good

faith,

useful

improvements

which are suitable to the use for which


the lease is intended, without altering
the form or substance of the property
leased, the lessor upon the termination
of the lease, shall pay the lessee one-half
of the value of improvements at that

mere lessee. Boboy responded by removing

time.

Should

the

lessor

refuse

to

the improvements and leaving the building

reimburse said amount, the lessee may

in its original state.

remove the improvements, even though


the principal thing may suffer damage

(IVa) Resolve Boboy's claim that as a builder


in good faith, he should be

thereby.

Never Let The Odds Keep You From Pursuing What You Know In Your Heart You Were Meant To Do.Leroy Satchel Paige

Page 103 of 180

Civil Law Q&As (2007-2013)

hectorchristopher@yahoo.com

(IVb) Can Boboy be held liable for damages


for

removing

the

improvements

over

Anselmo's objection? (4%)


SUGGESTED ANSWER:
No. Boboy cannot be held liable for
damages.
The

lessor,

Anselmo,

refused

to

reimburse one-half of the value of the


improvements, so the lessee, Boboy, may
remove
principal
thereby.

the

same,

thing
If

in

improvements

even

may

though

suffer

removing
Boboy

the

damage

the

useful

caused

more

impairment in the property leased than


is necessary he will be liable for damages
(Art. 1678, Civil Code).

Lease; Caveat Emptor (2009)


No.VIII. Jude owned a building which he
had leased to several tenants. Without
informing

his

tenants,

dbaratbateladot@gmail.com

when the term of their lease shall have

Jude

sold

the

building to Ildefonso. Thereafter, the latter


notified all the tenants that he is the new
owner of the building. Ildefonso ordered the
tenants to vacate the premises within thirty
(30) days from notice because he had other
plans for the building. The tenants refused
to vacate, insisting that they will only do so

expired. Is Ildefonso bound to respect the

status of the occupants of their right to


lease contracts between Jude and his

occupy the building before buying it.

tenants? Explain your answer. (3%)

Agency

SUGGESTED ANSWER:
Yes, Ildefonso must respect the lease
contracts between Jude and his tenants.
While it is true that the said lease
contracts

were

No.XVI. X was the owner of an unregistered

annotated on the title to the property,

parcel of land in Cabanatuan City. As she

Ildefonso

innocent

was abroad, she advised her sister Y via

purchaser for value. He ought to know

overseas call to sell the land and sign a

still

registered

an Agent (2010)

and

is

not

Agency; Sale of a Real Property through

not

an

the existence of the lease because the


building was already occupied by the
tenants

at

the

time

he

bought

contract of sale on her behalf.


Y thus sold the land to B1 on March 31,

it.

2001 and executed a deed of absolute sale

Applying the principle of caveat emptor,

on behalf of X. B1 fully paid the purchase

he should have checked and known the

price.

Never Let The Odds Keep You From Pursuing What You Know In Your Heart You Were Meant To Do.Leroy Satchel Paige

Page 104 of 180

Civil Law Q&As (2007-2013)

hectorchristopher@yahoo.com

B2, unaware of the sale of the land to B1,


signified to Y his interest to buy it but
asked Y for her authority from X. Without
informing X that she had sold the land to
B1, Y sought X for a written authority to
sell.
X e-mailed Y an authority to sell the land. Y
thereafter sold the land on May 1, 2001 to
B2 on monthly installment basis for two
years, the first installment to be paid at the
end of May 2001.
Who between B1 and B2 has a better right
over the land? Explain. (5%)
SUGGESTED ANSWER:
B-2 has a better title. This is not a case
of double sale. Since the first sale was
void. The law provides that when a sale
of a piece of land or any interest therein
is through an agent, the authority of the
latter shall be in writing; otherwise, the
sale shall be void (Art 1874, NCC). The
property was sold by Y to B1 wihtout any
written authority from the owner X.
Hence, the sale to B1 was void.
ALTERNATIVE ANSWER:
Under the facts, B-1 has a better right to
the land. Given the fact that the Deed of
Sale in favor of B-1 and B-2 are not
inscribed in the Registry of Deeds, the

dbaratbateladot@gmail.com

case is governed by Art 1544 of the New


Civil Code which provides that in case of

extinguishing right to real property not


double sales of an immovable property,
the

ownership

shall

pertain

to

the

person who is in good faith was first in


possession and in the absence thereof to

registered under Act 496 shall be valid


except as between the parties. Thus, the
Deed of Sale of B-2 has no binding effect
on B-1.

the person who presents the oldest title,

Partnership

provide there is good faith.


In a case, the Supreme Court has held
that in a sale of real estate the execution
of

notarial

document

of

sale

is

Liability; Liability of a Partner (2010)


No.XV.

A,

partnership

B,

and

to

operate

entered
a

into

restaurant

tantamount to delivery of the possession

business. When the restaurant had gone

of the property sold. The ownership of

past break-even stage and started to garner

the land therefore pertains to the first

considerable profits, C died. A and B

buyer. It may also be mentioned that

continued the business without dissolving

under Art 3344 no instruments or deed

the partnership. They in fact opened a

establishing,
acknowledging,

transmitting,
modifying,

or

branch

of

the

restaurant,

incurring

obligations in the process. Creditors started

Never Let The Odds Keep You From Pursuing What You Know In Your Heart You Were Meant To Do.Leroy Satchel Paige

Page 105 of 180

Civil Law Q&As (2007-2013)

demanding

for

the

hectorchristopher@yahoo.com

of Mota v. Serra, 47 Phil

payment

of

their

obligations.
(A). Who are liable for the settlement of the
partnerships obligations? Explain? (3%)
SUGGESTED ANSWER:
The two remaining partners, A and B, are
liable. When any partner dies and the
business

is

continued

without

any

settlement of accounts as between him


or his estate, the surviving partners are
held liable for continuing the business
despite the death of C (Art 1841, 1785,
par 2, and Art 1833 of NCC).
(B).What are the creditors recourse/s?
Explain. (3%)
SUGGESTED ANSWER:
Creditors
actions,

can
for

file

instance,

dbaratbateladot@gmail.com

the
an

appropriate
action

for

collection of sum of money against the


partnership at will and if there are no
sufficient funds, the creditors may go
after the private properties of A and B
(Art 816, NCC). Creditors may also sue
the estate of C. The estate is not excused
from the liabilities of the partnership
even if C is dead already but only up to
the time that he remained a partner (Art
1829, 1835, par 2, NCC; Testate Estate

464 [1925]). However, the liability of Cs

not in writing.
individual properties shall be subject
first to the payment of his separate
debts (Art 1835. NCC).

ALTERNATIVE ANSWER:
TRUE. An oral is a consensual of the
partnership is valid even though not in
writing.

However,

If

it

involves

contribution of an immovable property

Oral Partnership (2009)

or a real right, an oral contract of

No.I. TRUE or FALSE. Answer TRUE if the

partnership is void. In such a case, the

statement

the

contract of partnership to be valid, must

statement is false. Explain your answer in

be in a public instrument ( Art. 1771

not more than two (2) sentences.

,NCC

is

true,

or

FALSE

if

(C). An oral partnership is valid. (1%)

Partnership

and

the

inventory

of

said

property signed by the parties must be


attached to said public instrument (Art.
1773, NCC).

SUGGESTED ANSWER:
TRUE.

),

is

consensual

contract, hence, it is valid even though

ALTERNATIVE ANSWER:

Never Let The Odds Keep You From Pursuing What You Know In Your Heart You Were Meant To Do.Leroy Satchel Paige

Page 106 of 180

Civil Law Q&As (2007-2013)

TRUE.

Partnership

is

hectorchristopher@yahoo.com

consensual

contract, hence, it is valid even though


not in writing. The oral contract of
partnership is also valid even if an
immovable property or real right is
contributed thereto. While the law, in
such a case, requires the partnership to
be in a public document, the law does
not expressly declare the contract void if
not

executed

in

the

required

form

(Article 1409 (7 ,NCC ). And there being


nothing in the law from which it can be
inferred that the said requirement is
prohibitory
NCC),

the

or

mandatory

said

oral

(Article

contract

5,
of

partnership must also be valid. The


interested party may simply require the
contract

to

be

made

into

public

document in order to comply with the


required form (Article 1357, NCC). The
purpose of the law in requiring a public
document is simply to notify the public
about the contribution.

Share; Demand during the Existence of


Partnership (2012)
No.X.b) A partner cannot demand the
return of his share (contribution) during
the existence of a partnership. Do you
agree? Explain your answer. (5%)

dbaratbateladot@gmail.com

SUGGESTED ANSWER:

Such agreement was not reduced to writing.


Yes, he is not entitled to the return of
his contribution to the capital of the
partnership, but only to the net profits

Siga-an demanded interest which was paid


by Villanueva in cash and checks. The total
amount Villanueva paid accumulated to P
1, 200, 000.00. Upon advice of her lawyer,

from the partnership business during the

Villanueva demanded for the return of the

life of the partnership period. If he is a

excess amount of P 660, 000.00 which was

limited partner, however, he may ask for

ignored by Siga-an.

the

return

of

his

contributions

as

provided in Art 1856 and 1857, Civil


Code.

Interest;

Solutio

Indebiti

(2012)
No.VI.a)

(3%)
SUGGESTED ANSWER:

Commodatum & Mutuum


Mutuum;

(1) Is the payment of interest valid? Explain.

No, Art. 1956, Civil Code, provides that


no interest shall be due unless it has
been expressly stipulated in writing.

Siga-an

granted

loan

to

Villanueva in the amount of P 540, 000.00.

(2) Is solution indebiti applicable? Explain.


(2%)

Never Let The Odds Keep You From Pursuing What You Know In Your Heart You Were Meant To Do.Leroy Satchel Paige

Page 107 of 180

Civil Law Q&As (2007-2013)

hectorchristopher@yahoo.com

miscarriage of another, the Statute of


Frauds requires it to be in writing to be

SUGGESTED ANSWER:
Yes,

Solutio

because

Indebiti

Villanueva

P600,000.00

is

applicable

Overpaid

representing

by

interest

payment which is not due. He can,


therefore, demand its return.

Guaranty
Guaranty (2009)
No.I. TRUE or FALSE. Answer TRUE if the
statement

is

true,

or

FALSE

if

the

statement is false. Explain your answer in


not more than two (2) sentences.
(D). An oral promise of guaranty is valid
and binding. (1%)
SUGGESTED ANSWER :

FALSE. An oral contract of guaranty,


being a special promise to answer for the
debt of
another,

is

unenforceable

unless

in

writing (Article 1403 [2] b, NCC ).


ALTERNATIVE ANSWER:
TRUE. An oral promise of guaranty is
valid and binding. While the contract is
valid,

however

,it

is

unenforceable

because it is not writing . Being a special


promise

answer

for

dbaratbateladot@gmail.com

the

debt,

or

secondary; whereas, in suretyship, it is


enforceable ( Article 1403 [2] b, NCC).The
validity

of

the

contract

should

be

distinguished from its enforceability .

primary.
(2) In guranty, the undertaking is to pay if
the

Surety

principal

debtor

cannot

pay;

whereas, in suretyship, the undertaking


is to pay if the principal debtor does not

Surety (2010)

pay .

No.III. Define, Enumerate or Explain. (2%


each)
(A).

(3) In guranty, the guarantor is entitled to


the benefit of excussion; whereas, in

What

is

the

difference

between

"guaranty" and "suretyship"?

(4) Liability in guaranty depends upon


an independent agreement to pay the

SUGGESTED ANSWER:

obligations of the principal if he fails to

Guaranty and Suretyship distinguished


(1)The

obligation

suretyship the surety is not entitled.

in

guaranty

do so; whereas, in suretyship, the surety


is

assumes liability as a regular party.

Never Let The Odds Keep You From Pursuing What You Know In Your Heart You Were Meant To Do.Leroy Satchel Paige

Page 108 of 180

Civil Law Q&As (2007-2013)

hectorchristopher@yahoo.com

case of default by the pledgor.

(5)The Guarantor insures the solvency


of the principal debtor; whereas, the
surety insures the debt.
(6)In

guaranty,

subsidiarlty
suretyship,

the

liable;
the

guarantor

whereas,

surety

binds

in

is
a

himself

solidarity with the principal debtor (Art


2047, Civil Code).

Pledge
Pledge; Pactum Commissorium (2009)
No.XVII.

Rosario

obtained

loan

of

P100,000.00 from Jennifer, and pledged her


diamond ring. The contract signed by the
parties stipulated that if Rosario is unable
to redeem the ring on due date, she will
execute a document in favor of Jennifer
providing that the ring shall automatically
be considered full payment of the loan.
(A). Is the contract valid? Explain. (3%)
SUGGESTED ANSWER:
The contract is valid because Rosario
has to execute a document in favor of
Jennifer to transfer the ownership of the
pledged ring to the latter. The contract
does

not

commissorium

amount
because

dbaratbateladot@gmail.com

to
it

pactum
does

not

provide for the automatic appropriation


by the pledgee of the thing pledged in

them xxx. Jennifer cannot immediately


(B). Will your answer to [a] be the same if

sell by herself the thing pledged. It must

the contract stipulates that upon failure of

be foreclosed by selling it at a public

Rosario to redeem the ring on due date,

auction

Jennifer may immediately sell the ring and

procedure under Art 2112 of the New

appropriate the entire proceeds thereof for

Civil Code.

herself

as

full

payment

of

the

loan?

Reasons. (3%)
SUGGESTED ANSWER:
No, my answer will be different. While

in

accordance

with

the

Torts and Damages


Damages (2012)
No.I. a) Roberto was in Nikko Hotel when he

the contract of pledge is valid, the

bumped into a friend who was then on her

stipulation authorizing the pledgee to

way to a wedding reception being held in

immediately sell the thing pledged is

said hotel. Roberto alleged that he was then

void under Art 2088 of the New Civil

invited by his friend to join her at the

Code, which provides that the creditor

wedding reception and carried the basket

cannot appropriate the things given by


way of pledge or mortgage, or dispose of

full of fruits which she was bringing to the


affair. At the reception, the wedding

Never Let The Odds Keep You From Pursuing What You Know In Your Heart You Were Meant To Do.Leroy Satchel Paige

Page 109 of 180

Civil Law Q&As (2007-2013)

hectorchristopher@yahoo.com

ridicule or shame, his action will

coordinator of the hotel noticed him and


asked him, allegedly in a loud voice, to leave
as he was not in the guest list. He retorted
that he had been invited to the affair by his
friend,
Deeply

who

however

embarrassed

denied
by

doing

the

so.

incident,

Roberto then sued the hotel for damages


under Articles 19 and 21 of the
Civil Code. Will Robertos action prosper?
Explain. (5%)
SUGGESTED ANSWER:
No. Robertos action will not prosper.
From the facts given in the problem, the
wedding coordinator did not abuse her
right when she asked him to leave the
wedding reception because he was not in
the guest list. Hotel Nikko could not be
held liable for damages as its liable
spring from the liability of its employee
(Nikko Hotel Manila Garden v. Reyes,
G.R. No. 154259, Feb 28, 2005).
ALTERNATIVE ANSWER:
It depends. While the hotel has the right
to exclude an uninvited guest from the
wedding reception, that does not give
the

hotel

the

license

dbaratbateladot@gmail.com

to

humiliate

Roberto. If the wedding coordinator of


the hotel acted wrongfully e.g. with the
abuse of right, unfairly, or in a matter
that exposed Roberto to unnecessary

problem makes no statement of that


prosper. Otherwise, Robertos action will
not prosper.

finding. In the contrary, the problem


states that it is a mere allegation.

The hotel is liable for the wrongful acts


of its employees.

Damages; Moral & Exemplary (2009)

COMMENT:
No.XIV. Rodolfo, married to Sharon, had an
The facts of the problem are almost

illicit affair with his secretary, Nanette, a

similar to the facts of Nikko Hotel

19-year old girl, and begot a baby girl,

Manila

No.

Rona. Nanette sued Rodolfo for damages:

154259, Feb 28, 2005. In the said case,

actual, for hospital and other medical

however, there is a categorical finding

expenses

Garden

v.

Reyes,

G.R.

that the hotel employee did not, exposed


the complainant to the ridicule, shame
or

embarrassment;

hence,

did

not

commit any abuse of right. The present

in

delivering

the

child

by

caesarean section; moral, claiming that


Rodolfo

promised

to

marry

her,

representing that he was single when, in


fact, he was not; and exemplary, to teach a
lesson to like-minded Lotharios.

Never Let The Odds Keep You From Pursuing What You Know In Your Heart You Were Meant To Do.Leroy Satchel Paige

Page 110 of 180

Civil Law Q&As (2007-2013)

hectorchristopher@yahoo.com

(A). If you were the judge, would you award


all the claims of Nanette? Explain. (3%)
SUGGESTED ANSWER:
If Rodolfo's marriage could not have
been possibly known to Nanette or there
is no gross negligence on the part of
Nanette, Rodolfo could be held liable for
moral damages.

If there is gross negligence in a suit for


quasi-delict,

exemplary

could

be

awarded.

Damages; Public Officers acting in the


Performance of their Duties (2012)
No.II.a) Liwayway Vinzons-Chato was then
the

Commissioner

of

Internal

Revenue

while Fortune Tobacco Corporation is an


entity

engaged

in

the

manufacture

of

different brands of cigarettes, among which


are

"Champion,"

dbaratbateladot@gmail.com

thereby imposed higher taxes), which the

"Hope,"

and

"More"

cigarettes.
Fortune filed a complaint against VinzonsChato to recover damages for the alleged
violation of its constitutional rights arising
from Vinzons-Chatos issuance of Revenue
Memorandum Circular No. 37-934 (which
re-classified Fortune cigarettes as locally
manufactured with foreign brands and

Supreme Court later declared invalid.

The public officer is not automatically


Vinzons-Chato filed a Motion to Dismiss

considered to have violated the rights or

arguing that she cannot be held liable for

liberties of a person simply because the

damages for acts she performed while in

rule

the

declared

discharge

of

her

duties

as

BIR

Commissioner. Is she correct? Explain. (5%)


SUGGESTED ANSWER:

the

public
invalid

officer
by

the

issued

was

court.

The

complainant must still allege and prove


the particular injury or prejudice he has
suffered

from

the

violation

of

his

Yes. As a general rule, a public officer is

constitutional right by the issuance of

not liable for acts performed in the

the invalidated rule.

discharge of his duties. The exceptions


are when he acted with malice, bad faith,
or gross negligence in the performance
of his duty, or when his act is in
violation of a Constitutional guaranteed
right and liberties of a person under
Art32 of the NCC.

The problem does not state any fact from


which any malice, bad faith or gross
negligence on the part of Vinzons-Chato
may be inferred, or the particular injury
or prejudice the complainant may have
suffered as a result of the violation of his
constitutional right. Hence, she

Never Let The Odds Keep You From Pursuing What You Know In Your Heart You Were Meant To Do.Leroy Satchel Paige

Page 111 of 180

Civil Law Q&As (2007-2013)

cannot

be

held

hectorchristopher@yahoo.com

liable for the damage awarded against

liable.

The

facts

presented are similar to facts of the case


of Vinzons-Chato v. Fortune, G.R. No.
141309, Dec 23, 2008.

Death Indemnity (2009)


No. X. Rommels private car, while being
driven by the regular family driver, Amado,
hits a pedestrian causing the latters death.
Rommel is not in the car when the incident
happened.
(A). Is Rommel liable for damages to the
heirs of the deceased? Explain. (2%)
SUGGESTED ANSWER:
Yes, Rommel may be held liable for
damages if he fails to prove that he
exercised the diligence of a good father
of a family (Art. 2180, par 5, NCC) in
selecting

and

supervising

his

family

driver. The owner is presumed liable


unless

he

proves

the

defense

of

diligence. If the driver was performing


his assigned task when the accident
happened, Rommel shall be solidarily
liable with the driver.

In

case

the

driver

is

dbaratbateladot@gmail.com

convicted

of

reckless imprudence and cannot pay the


civil liability, Rommel is subsidiarily

Yes, my answer will be the same except


the driver and the defense of diligence is
not available.

that in such case the liability of the


owner is not presumed. When the owner
is inside the vehicle, he becomes liable

(B).Would your answer be the same if

only when it is shown that he could have

Rommel was in the car at the time of the

prevented the misfortune by the use of

accident? Explain. (2%)


SUGGESTED ANSWER:
Yes, my answer would be the same.
Rommel, who was in the car, shall be
liable for damages if he could have

due diligence (Art. 2184, NCC). For the


owner to be held liable, the burden of
proving that he could have prevented
the misfortune rests on the shoulder of
the victim.

prevented the misfortune by the use of


due diligence in supervising his driver
but failed to exercise it (Art. 2184, NCC).
In such case, his liability is solidary with
his driver.
ALTERNATIVE ANSWER:

Doctrine of Discovered Peril (Last Clear


Chance) (2007)
No.III. Explain the following concepts and
doctrines and give an example of each:

Never Let The Odds Keep You From Pursuing What You Know In Your Heart You Were Meant To Do.Leroy Satchel Paige

Page 112 of 180

Civil Law Q&As (2007-2013)

hectorchristopher@yahoo.com

(B). doctrine of discovered peril (last clear


chance) (5%)
SUGGESTED ANSWER:
The doctrine of last clear chance states
that where the plaintiff was guilty of
prior or antecedent negligence, but the
defendant,

who

opportunity

to

had
avoid

the
the

ultimate
impending

harm failed to do so, it is the defendant


who is liable for all the consequences of
the accident notwithstanding the prior
negligence of the plaintiff. An example is
where a person was riding a pony on a
bridge and improperly pulled the pony to
the wrong side when he saw a car
coming. The driver of the car did not
stop or change direction, and nearly hit
the horse, and, the frightened animal
jumped to its death. The driver of the
car is guilty of negligence because he
had a fair opportunity to avoid the
accident and failed to avail himself of
that opportunity. He is liable under the
doctrine of last clear chance (Picart v.
Smith, 37 Phil. 809, 1918).

Liability; Owner of a Pet; Fortuitous


Event (2010)

dbaratbateladot@gmail.com

No.XIV. Primo owns a pet iguana which he


keeps in a man-made pond enclosed by a
fence situated in his residential lot. A

lost. This responsibility shall cease only


typhoon knocked down the fence of the

in case the damage should come from

pond and the iguana crawled out of the gate

force majeure or from the fault of the

of Primos residence. N, a neighbor who was

person who has suffered damage (Art

passing by, started throwing stones at the

2183, NCC).

iguana, drawing the iguana to move toward


him. N panicked and ran but tripped on
something and suffered a broken leg.
Is anyone liable for Ns injuries? Explain.

Liability;

Special

Parental

Authority

(2010)

(4%)
No.XII. On May 5, 1989, 16-year old
SUGGESTED ANSWER:
No one is liable. The possessor of an

Rozanno, who was issued a student permit,


drove to school a car, a gift from his
parents. On even date, as his class was

animal or whoever may make use of the

scheduled to go on a field trip, his teacher

same is responsible for the damage it

requested him to accommodate in his car,

may cause, although it may escape or be

as he did, four (4) of his classmates

Never Let The Odds Keep You From Pursuing What You Know In Your Heart You Were Meant To Do.Leroy Satchel Paige

Page 113 of 180

Civil Law Q&As (2007-2013)

hectorchristopher@yahoo.com

because the van rented by the school was


too crowded. On the way to a museum
which the students were scheduled to visit,
Rozanno made a wrong maneuver, causing
a collision with a jeepney. One of his
classmates died. He and the three (3) others
were badly injured.
(A). Who is liable for the death of Rozannos
classmate and the injuries suffered by
Rozanno and his 3 other classmates?
Explain. (2%)
SUGGESTED ANSWER:
At the time the incident occurred in May
1989, Rozanno was still a minor. Being a
minor, Art 218 of the Family Code
applies. Pursuant to Art 218, the school,
its administrators and teachers shall be
liable for the acts of minor Rozanno
because of the special parental authority
and responsibility that they exercise
over him. The authority applies to all
authorized activities, whether inside or
outside the premises of the school,
entity or institution. The field trip on
which occasion Rozanno drove the car,
was an authorized activity, and , thus,
covered by the provision. Furthermore,
the parents of Rozanno are subsidiarily
liable pursuant to Art 219 (FC), and
principally liable under Art 221 (FC), if

they are negligent.

dbaratbateladot@gmail.com

if the incident happened sometime in


(B). How about the damage to the jeepney?
Explain. (2%)

the middle of 1994, Rozanno have been


21 years old at the time. Hence, he was
already of legal age. The law reducing the

SUGGESTED ANSWER:
With respect to the damages caused to
the jeepney, only Rozanno should be
held liable because his negligence or
tortuous act was the sole, proximate and
immediate cause thereof.
(C). Under the same facts, except the date of
occurrence of the incident, this time in

age of majority to 18 years took effect in


December 1989.
Being of legal age, articles 218, 219, and
221 of the Family Code are no longer
applicable. In such case, only Rozanno
will be personally responsible for all the
consequences

of

his

act

unless

his

school or his parents were themselves

mid-1994, what would be your answer?

also

negligent

and

such

negligence

Explain. (2%)

contributed to the happening of the


incident. In that event, the school or his

SUGGESTED ANSWER:
Since Rozanno was 16 years old in 1989,

parents are not liable under Art 218, 218


or 221 of the Family Code, but will be

Never Let The Odds Keep You From Pursuing What You Know In Your Heart You Were Meant To Do.Leroy Satchel Paige

Page 114 of 180

Civil Law Q&As (2007-2013)

hectorchristopher@yahoo.com

dbaratbateladot@gmail.com

liable under general provision on the

court to justify the damages that your client

Civil Code on quasi-delict.

claims? (8%)
SUGGESTED ANSWER:

Quasi-Delict; Claims; Requisites (2013)


No.II. A collision occurred at an intersection

I will the base the claim of my client on


quasi-delict under Art 2176 of the Civil

involving a bicycle and a taxicab. Both the

Code of the Philippines. The requisites

bicycle rider (a businessman then doing his

for a claim under quasi-delict to prosper

morning

are as follows:

exercise)

and

the

taxi

driver

claimed that the other was at fault. Based


on the police report, the bicycle crossed the
intersection first but the taxicab, crossing
at a fast clip from the bicycle's left, could
not brake in time and hit the bicycle's rear
wheel, toppling it and throwing the bicycle
rider into the sidewalk 5 meters away.
The bicycle rider suffered a fractured right
knee, sustained when he fell on his right
side on the concrete side walk. He was
hospitalized

and

was

subsequently

operated on, rendering him immobile for 3


weeks and requiring physical rehabilitation

(1) Act or omission, there being fault or


negligence;
(2) Damage or injury; and
(3) Causal connection between the damage
and the act or omission.
The case clearly involves quasi-delict
where

my

suffered

client,

injury

negligence

of

as
the

the
a

bicycle
result

rider,
of

the

over-speeding

taxi

driver, without fault on my clients part.

for another 3 months. In his complaint for


damages, the rider prayed for the award

To prove actual damages aside from the

ofP1,000,000 actual damages,P200,000

testimony of client, I will present his

moral

hospital and medical bills. Receipts paid

damages, P200,000

damages, P1 00,000 nominal

exemplary
damages

and P50,000 attorney's fees.

on

the

rehabilitation

will

also

be

presented. [The sentence in red should

Assuming the police report to be correct

be replaced with the following sentence

and as the lawyer for the bicycle rider, what

because he is a businessman and not an

evidence

employee. Furthermore, I will present

(documentary

and

testimonial)

and legal arguments will you present in

income tax returns, contracts and other

documents to prove unrealized profits as

a result of this temporary injury.] I will

Never Let The Odds Keep You From Pursuing What You Know In Your Heart You Were Meant To Do.Leroy Satchel Paige

Page 115 of 180

Civil Law Q&As (2007-2013)

hectorchristopher@yahoo.com

the award for moral damages.

also call the attending physician to


testify as to the extent of the injuries
suffered by my client, and to corroborate
the contents of the medical documents.
Based on Art. 2202, in quasi-delicts, the
defendant shall be liable for all damages
which

are

the

natural

and

probable

consequences of the act or omission


complained of. It is not necessary that
the damages have been foreseen or could
have been foreseen by the defendant.
Unlike

actual

damages,

no

proof

of

pecuniary loss is necessary in order that


moral, nominal, temperate liquidated or
exemplary damages may be adjudicated.
The assessment is left to the discretion
of the Court (Art. 2216, Civil Code).
There

must

be

proof

pecuniary

estimation, however.
Moral damages can be recovered by my
client under Articles 2219 and 2200.
Moral damages may be recovered in case
of

quasi-delict

dbaratbateladot@gmail.com

causing

physical

injuries. Additionally, it must be proved


that such damages were the proximate
result of the act complained of. Medical
certificates will be presented, along with
the testimony from my client and other
eyewitness accounts, in order to support

Exemplary damages may be granted if


the defendant acted in wanton,

when exemplary damages are awarded


fraudulent,

reckless,

oppressive,

or

(Art 2208, Civil Code).

malevolent manner. While the amount of


exemplary damages may not be proved,
the

plaintiff

entitled
damages.

to
In

must
moral

show
or

support

of

that

he

is

compensatory
this,

will

Quasi Tort (2010)


No.III. Define, Enumerate or Explain. (2%
each)

present the police report showing the


circumstance under which the accident

(B). Define quasi tort. Who are the persons

took place, taking into account the

liable under quasi torts and what are the

actions of the parties. I will ask the

defenses available to them?

officials who responded to the accident


to testify as to the conduct of the

Note: It is recommended that the examiner


exercise leniency and liberality in grading

parties at the time of the accident in

the answers given to this question. The term

order to determine whether defendant

quasi-tort is not part of legal developments

was guilty of gross negligence.

in civil law. In Philippine legal tradition,


quasi-

Finally, attorneys fees may be recovered


Never Let The Odds Keep You From Pursuing What You Know In Your Heart You Were Meant To Do.Leroy Satchel Paige

Page 116 of 180

Civil Law Q&As (2007-2013)

hectorchristopher@yahoo.com

equivalent of quasi-delict. Hence the

delict has been treated as the closest civil


law equivalent of the common law tort. In
fact,

in

decisions,

number
the

two

of

Supreme

terms

have

Court
been

considered synonymous. In reality, however,


the common law tort is much broader in
scope than the civil law quasi-delict. In
recent developments in common law, the
concept of quasi-torts can be considered as
the closest common law equivalent of the
civil law concept of quasi-delict. This is
because it is argued that the growing
recognition of quasi-torts as a source of
obligation is hinged on the acceptance at
common law of the civil law principles of
quasi-delict.
SUGGESTED ANSWER:
Quasi -tort is a legal concept upholding
the doctrine that some legal duty exists
that cannot be classified strictly as a
personal duty (thus resulting in a tort),
nor as a contractual duty but rather
some other kind of duty recognizable by
the law. Tort or Quasi-tort is an
Anglo

American

or

Common

Law

concept, while Delict or Quasi-Delict


is

Civil

Law

concept

(Wikipedia

encyclopedia).
ALTERNATIVE ANSWER:
Quasi

-tort

is

dbaratbateladot@gmail.com

considered

as

the

rules of the latter pertaining to persons

his negligence is not the proximate


who can be held liable and their defenses
would also apply.
Those liable for quasi-delict include:

cause of the injury ( Article 2179 NCC );


(c) That the plaintiff's own negligence was
the immediate and proximate cause of
his injury ( Article 2179 NCC );

(1) Those tortfeasor or the person causing


damage to another through fault or
negligence ( Article 2176 NCC ); and
(2) Persons vicariously liable under Article
2180 (NCC ).

(d ) That the person vicariously liable


has observed all the diligence of a good
father of a family to prevent damage
( Article 2180 NCC ); and
(e) That the cause of action has

The defenses available include:


(a) That the defendant was not negligent or

prescribed after the lapse s (Article 2179


NCC ).

that he exercised due diligence ( Article

The

2176 NCC );

committed contributory negligence is a

(b) That although the defendant is negligent

fact

that

the

plaintiff

had

partial defense (Art 2179, NCC).

Never Let The Odds Keep You From Pursuing What You Know In Your Heart You Were Meant To Do.Leroy Satchel Paige

Page 117 of 180

Civil Law Q&As (2007-2013)

hectorchristopher@yahoo.com

MULTIPLE CHOICE

per capita, and not by right of


representation (Art 975, Civil Code)

QUESTIONS
2013

dbaratbateladot@gmail.com

I. (2) How much is Dante's share in the net

Taxation Law Exam

estate? (1%)

MCQ (October 13, 2013)

(A) P150,000.
(B) P200,000.

I. Armand died intestate. His full-blood

(C) P300,000.

brothers, Bobby and Conrad, and half-

(D) P400,000.

blood brothers, Danny, Edward and Floro,

(E) None of the above.

all predeceased him. The following are the


surviving relatives:

SUGGESTED ANSWER:

1. Benny and Bonnie, legitimate children of


Bobby;

There is no showing that Danny is an

2. Cesar, legitimate child of Conrad;

illegitimate

3. Dante, illegitimate child of Danny;

value

of

Armand's

estate

brother

of

contrary, the law presumes that the

5. Felix, grandson of Floro.


net

half-blood

Armand. In the absence of proof to the

4. Ernie, adopted child of Edward; and

The

E. None of the above.

is

Pl,200,000.

relationship is legitimate. Thus, Dante,


an illegitimate child of Danny, is barred
from inheriting from Armand pursuant

I. (1) How much do Benny and Bonnie

to

stand to inherit by right of representation?

disqualifies an illegitimate child from

(1%)
(A) P200,000
(B) P300,000
(C) P400,000
(D) P150,000

the

iron

inheriting

ab

curtain

intestao

rule

from

which

the

legitimate children and relatives of his


father or mother, and vice versa (Art
992, Civil Code).

(E) None of the above.


I. (3) How much is Ernie's share in the net
SUGGESTED ANSWER:

estate . (1%)

(E) None of the above.

(A) P 0.

If all the brothers/sisters are disqualified


to inherit, the nephews/nieces inherit

(B) P400,000.
(C) P150,000.
(D) P200,000.

(E) None
above.

of

the

SUGGESTED
ANSWER:

Never Let The Odds Keep You From Pursuing What You Know In Your Heart You Were Meant To Do.Leroy Satchel Paige

Page 118 of 180

Civil Law Q&As (2007-2013)

hectorchristopher@yahoo.com

(A) 0 or (E) None of the above.


The

legal

relationship

dbaratbateladot@gmail.com

Should the share of insolvent debtor C be

created

by

adoption is strictly between the adopter


and the adopted. It does not extend to
the relatives of either party (Sayson v.
CA, G.R. Nos. 89224-25, Jan 23, 1992).
(Note: E. None of the above is another
answer because Ernie has no share at
all in the net estate).

divided

only

between

the

two

other

remaining debtors, A and B? (1%)


(A) Yes. Remission of D's share carries with it
total extinguishment of his obligation to the
benefit of the solidary debtors.
(B) Yes. The Civil Code recognizes remission as
a mode of extinguishing an obligation. This
clearly applies to D.
(C) No. The rule is that gratuitous acts should
be restrictively construed, allowing only the

I. (4) How much is Felix's share in the net


estate? (1%)

least transmission of rights.


(D) No, as the release of the share of one

(A) P400,000.

debtor would then increase the burden of

(B) P150,000.

the other debtors without their consent.

(C) P300,000.
(D) P0.

SUGGESTED ANSWER:

(E) None of the above.

(D). No, as the release of the share of one

SUGGESTED ANSWER:

debtor would then increase the burden of

(D). 0. Or (E) None of the above.

the other debtors without their consent.

In the collateral line, representation is

When one of the solidary debtors cannot,

granted only to children of brother or

because of his insolvency, reimburse his

sisters,

share

Felix

is

grandson

of

to

the

debtor

paying

the

predeceased brother.

obligation, such share shall be borne by

(Note: E. None of the above: is another

all his co-debtors, in proportion to the

answer because Felix has no share at

debt of each (Art 1217, Civil Code).

all in the net estate)

Additionally, D was released only from


his share of P10,000.00 not from the

II. A, B, C and D are the solidary debtors of

solidary tie that binds him to A, B and C.

X for P40,000. X released D from the


payment of his share of PI 0,000. When the

III. Amador obtained a loan of P300,000

obligation became due and demandable, C

from Basilio payable on March25, 2012. As

turned out to be insolvent.

security for the payment of his loan,

residential house and lot in Basilio's favor.

Amador constituted a mortgage on his


Never Let The Odds Keep You From Pursuing What You Know In Your Heart You Were Meant To Do.Leroy Satchel Paige

Page 119 of 180

Civil Law Q&As (2007-2013)

hectorchristopher@yahoo.com

dbaratbateladot@gmail.com

(Note: A is not the correct answer


Cacho, a good friend of Amador, guaranteed
and obligated himself to pay Basilio, in case
Amador fails to pay his loan at maturity.

III. (1) If Amador fails to pay Basilio his loan


on March 25, 2012, can Basilio compel
Cacho to pay? (1%)

(A) No, Basilio cannot compel Cacho to pay


because as guarantor, Cacho can invoke the
principle of excussion, i.e., all the assets of
Basilio must first be exhausted.
(B) No, Basilio cannot compel Cacho to pay
because Basilio has not exhausted the
available remedies against Amador.
(C) Yes, Basilio can compel Cacho to pay
because the nature of Cacho's undertaking
indicates

that

he

has

bound

himself

solidarily with Amador.


(D) Yes, Basilio can compel Cacho who bound
himself to unconditionally pay in case
Amador fails to pay; thus the benefit of
excussion will not apply.
SUGGESTED ANSWER:
(B) No, Basilio cannot compel Cacho to
pay because Basilio has not exhausted
the available remedies against Amador.
The guarantor cannot be compelled to
pay the creditor unless the latter has
exhausted all the property of the debtor
and has resorted to all the legal remedies
against the debtor (Art. 2058, Civil Code)

because it states that all the assets of

ownership on the buyer, Diego, who must


Basilio

(the

creditor)

must

first

be

therefore consent.
(D) No, Basilio cannot foreclose the real estate

exhausted)
III. (2) If Amador sells his residential house
and lot to Diego, can Basilio foreclose the
real estate mortgage? (1%)

(A) Yes, Basilio can foreclose the real estate

mortgage. To deprive the new owner of


ownership and possession is unjust and
inequitable.
SUGGESTED ANSWER:

mortgage because real estate mortgage

(B) Yes, Basilio can foreclose the real

creates a real right that attaches to the

estate mortgage. It is binding upon Diego

property.

as the mortgage is embodied in a public

(B) Yes, Basilio can foreclose the real estate

instrument.

mortgage. It is binding upon Diego as the

Since

mortgage

instrument, there is constructive notice

is

embodied

in

public

to

instrument.
(C) No, Basilio cannot foreclose the real
estate

mortgage.

The

sale

the

Diego,

mortgage

who

is

is

the

in

buyer

public

if

the

mortgaged property.

confers
ALTERNATIVE ANSWER:

Never Let The Odds Keep You From Pursuing What You Know In Your Heart You Were Meant To Do.Leroy Satchel Paige

Page 120 of 180

Civil Law Q&As (2007-2013)

hectorchristopher@yahoo.com

of

(C) No, Basilio cannot foreclose the real


estate

mortgage.

The

sale

confers

ownership on the buyer, Diego, who must


therefore consent.
The mortgage is not registered, thus,
cannot be binding against third persons
(Art. 2125, Civil Code)

IV. Cruz lent Jose his car until Jose


finished his Bar exams. Soon after Cruz
delivered

the

car,

Jose

brought

it

to

Mitsubishi Cubao for maintenance check


up and incurred costs of P8,000. Seeing the
car's peeling and faded paint, Jose also had
the car repainted for P10,000. Answer the
two

questions

below

based

on

these

common facts.

IV. (1) After the bar exams, Cruz asked for


the return of his car. Jose said he would
return it as soon as Cruz has reimbursed
him for the car maintenance and repainting
costs of P 18,000.
Is Jose's refusal justified? (1%)
(A) No, Jose's refusal is not justified. In this
kind of contract, Jose is obliged to pay for
all

the

expenses

incurred

for

the

preservation of the thing loaned.


(B) Yes, Jose's refusal is justified. He is obliged
to

pay

for

extraordinary

all

the

expenses,

ordinary
but

subject

and
to

reimbursement from Cruz.


(C) Yes, Jose's refusal is justified. The principle

unjust

dbaratbateladot@gmail.com

enrichment

warrants

reimbursement of Jose's expenses.

the

The

bailee,

Jose,

has

no

right

of

(D) No, Jose's refusal is not justified. The

retention on the ground that the bailor

expenses he incurred are useful for the

owes him something, even if it may be

preservation of the thing loaned. It is

by reason of expenses. He can only

Jose's obligation to shoulder these useful

retain it if he suffers damages by reason

expenses.

of a flaw or defect in the thing loaned of


which the bailor knows (Art 1951, Civil

SUGGESTED ANSWER:

Code).

(D) No, Jose's refusal is not justified. The


expenses he incurred are useful for the

IV. (2) During the bar exam month, Jose

preservation of the thing loaned. It is

lent the car to his girlfriend, Jolie, who

Jose's obligation to shoulder these useful

parked the car at the Mall of Asia's open

expenses.
In commodatum, the bailee is obliged to
pay for the ordinary expenses for the use
and preservation of the thing loaned (Art

parking lot, with the ignition key inside the


car. Car thieves broke into and took the car.

Is Jose liable to Cruz for the loss of the car


due to Jolie's negligence? (1%)

1941, Civil Code).


Never Let The Odds Keep You From Pursuing What You Know In Your Heart You Were Meant To Do.Leroy Satchel Paige

Page 121 of 180

Civil Law Q&As (2007-2013)

hectorchristopher@yahoo.com

(A) No, Jose is not liable to Cruz as the loss


was not due to his fault or negligence.
(B) No, Jose is not liable to Cruz. In the
absence of any prohibition, Jose could lend
the car to Jolie. Since the loss was due to
force majeure, neither Jose nor Jolie is
liable.
(C) Yes, Jose is liable to Cruz. Since Jose
lent the car to Jolie without Cruz's
consent, Jose must bear the consequent
loss of the car.
(D) Yes, Jose is liable to Cruz. The contract
between them is personal in nature. Jose
can neither lend nor lease the car to a third
person.
SUGGESTED ANSWER:
(C) Yes, Jose is liable to Cruz. Since Jose
lent the car to Jolie without Cruz's
consent, Jose must bear the consequent
loss of the car.
The bailee is liable for the loss of the
thing, even if it should be through a
fortuitous event if he lends or leases the
thing to a third person, who is not a
member of his household (Art 1942, Civil
Code).

V. In 2005, L, M, N, 0 and P formed a


partnership. L, M and N were capitalist
partners who contributed P500,000 each,
while 0, a limited partner, contributed P1 ,

dbaratbateladot@gmail.com

000,000. P joined as an industrial partner,


contributing only his services. The Articles
of Partnership, registered with the

V. (l) Assuming that the just and equitable


Securities

and

Commission,

share of the industrial partner, P, in the

designated L and 0 as managing partners; L

profit in 2006 amounted to P1 00,000, how

was liable only to the extent of his capital

much is the share of 0, a limited partner, in

contribution; and P was not liable for

the P800,000 net profit? (1%)

losses.

(A) P160,000.

In 2006, the partnership earned a net profit

(B) P175,000.

of P800,000. In the same year, P engaged in

(C) P280,000.

a different business with the consent of all

(D) P200,000.

the

(E) None of the above.

partners.

partnership

Exchange

However,

incurred

in

2007,

net

loss

the
of

P500,000. In 2008,the partners dissolved

SUGGESTED ANSWER:

the partnership. The proceeds of the sale of

(C) P280,000.

partnership

First, deduct the share of P from the

assets

were

insufficient

to

settle its obligation. After liquidation, the

profits.

partnership

P700,000. Next, get the share of O by

ofP300,000.

had

an

unpaid

liability

P800,000

less

P100,000

is

following the proportion that the shares


of L, M, N, O is 1:1:1:2, respectively.

Never Let The Odds Keep You From Pursuing What You Know In Your Heart You Were Meant To Do.Leroy Satchel Paige

Page 122 of 180

Civil Law Q&As (2007-2013)

hectorchristopher@yahoo.com

partner.

V. (2) In 2007, how much is the share of 0,


a limited partner, in the net loss of
P500,000? (1%)
(A) P 0.
(B) P1 00,000.
(C) P125,000.
(D) P200,000.
(E) None of the above.
SUGGESTED ANSWER:
(D) P200,000
A limited partner shall not become liable
a s a general partner unless, in addition
to the exercise of his rights and powers
as a limited partner, he takes part in the
control of the business (Art 1948, Civil
Code). In the absence of stipulation as to
profits and losses, the share of each
partner

in

the

losses

shall

be

proportionate to what he may have


contributed (Art 1797).

V. (3) Can the partnership creditors hold L,


0 and Pliable after all the assets of the
partnership are exhausted? (1%)
(A) Yes. The stipulation exempting P from
losses is valid only among the partners. L is
liable because the agreement limiting his
liability to his capital contribution is not
valid insofar as the creditors are concerned.
Having taken part in the management of
the partnership, 0 is liable as capitalist

dbaratbateladot@gmail.com

Partnership embodying such stipulations


(B) No. P is not liable because there is a valid
stipulation exempting him from losses.
Since the other partners allowed him to
engage in an outside business activity, the
stipulation absolving P from liability is
valid. For 0, it is basic that a limited
partner is liable only up to the extent of his

serves

as

constructive

notice

to

the

partnership creditors.(E) None of the above


is completely accurate.
(E) None

of

the

above

is

completely

accurate.
SUGGESTED ANSWER:
(E) None of the above is completely

capital contribution.
(C) Yes. The stipulations exempting P and L

accurate.

from losses are not binding upon the

VI. Gary is a tobacco trader and also a

creditors. 0 is likewise liable because the

lending investor. He sold tobacco leaves to

partnership was not formed in accordance

Homer

with

although the period for delivery was not

the

requirements

of

limited

partnership.
(D) No. The Civil Code allows the partners to
stipulate that a partner shall not be liable
for losses. The registration of the Articles of

for

delivery

within

month,

guaranteed. Despite Gary's efforts to deliver


on

time,

transportation

problems

and

government red tape hindered his efforts


and he could only deliver after 30 days.

Never Let The Odds Keep You From Pursuing What You Know In Your Heart You Were Meant To Do.Leroy Satchel Paige

Page 123 of 180

Civil Law Q&As (2007-2013)

hectorchristopher@yahoo.com

Homer refused to accept the late delivery


and to pay on the ground that the agreed
term had not been complied with.
As

lending

investor,

Gary

granted

Pl,000,000 loan to Isaac to be paid within


two years from execution of the contract. As
security for the loan, Isaac promised to
deliver to Gary his Toyota Innova within
seven (7) days, but Isaac failed to do so.
Gary

was

thus

compelled

to

demand

payment for the loan before the end of the


agreed two-year term.

VI. (l) Was Homer justified in refusing to


accept the tobacco leaves? (1%)
(A) Yes. Homer was justified in refusing to
accept the tobacco leaves. The delivery was
to be made within a month. Gary's promise
of delivery on a "best effort" basis made the
delivery uncertain. The term, therefore, was
ambiguous.
(B) No. Homer was not justified in refusing
to

accept

the

tobacco

leaves.

He

consented to the terms and conditions


of

the

sale

and

must

abide

by

it.

Obligations arising from contract have


the force of law between the contracting
parties.
(C) Yes. Homer was justified in his refusal to
accept

the

delivery.

dbaratbateladot@gmail.com

could not insist that Homer accept the

The

contract

contemplates an obligation with a term.


Since the delivery was made after 30 days,
contrary to the terms agreed upon, Gary

tobacco leaves.

parties.
(D) No. Homer was not justified in refusing

It is clear under the facts that the period

to accept the tobacco leaves. There was no

of delivery of the tobacco leaves was not

term in the contract but a mixed condition.

guaranteed.

The fulfillment of the condition did not

factors which may prevent him from

depend purely on Gary's will but on other

making the delivery within a month.

factors, e.g., the shipping company and the

True enough, transportation problems

government. Homer should comply with his


obligation.

No.

Homer

was

not

justified

in

consented to the terms and conditions


the

sale

other

and government red tape did. Such

Obligations arising from contract have

refusing to accept the tobacco leaves. He

of

anticipated

slight delay was, thus, excusable.

SUGGESTED ANSWER:
(B)

Gary

and

must

abide

by

it.

Obligations arising from contract have


the force of law between the contracting

the force of law between the contracting


parties and should be complied with in
good faith (Art. 1160, Civil Code)

VI. (2) Can Gary compel Isaac to pay his


loan even before the end of the two-year
period? (1%)

Never Let The Odds Keep You From Pursuing What You Know In Your Heart You Were Meant To Do.Leroy Satchel Paige

Page 124 of 180

Civil Law Q&As (2007-2013)

(A) Yes,

Gary

can

immediately

pay

hectorchristopher@yahoo.com

immediately demandable. Isaac lost his

compel
the

Isaac
loan.

to
Non-

compliance with the promised guaranty


or

security

renders

the

obligation

immediately demandable. Isaac lost his


right to make use of the period.
(B) Yes, Gary can compel Isaac to immediately
pay the loan. The delivery of the Toyota
Innova is a condition for the loan. Isaac's
failure to deliver the car violated the
condition upon which the loan was granted.
It is but fair for Gary to demand immediate
payment.
(C) No,

Gary

cannot

compel

Isaac

to

immediately pay the loan. The delivery of


the car as security for the loan is an
accessory contract; the principal contract is
still the P 1,000,000 loan. Thus, Isaac can
still make use of the period.
(D) No,

Gary

cannot

compel

Isaac

to

immediately pay the loan. Equity dictates


that Gary should have granted a reasonable
extension of time for Isaac to deliver his
Toyota Innova. It would be unfair and
burdensome
the P1,000,000

for

Isaac

to

pay

simplybecause

the

promised security was not delivered.


SUGGESTED ANSWER:
(A)

Yes,

Gary

immediately

can
pay

compel
the

Isaac

loan.

to

Non-

compliance with the promised guaranty


or

security

renders

dbaratbateladot@gmail.com

the

obligation

right to make use of the period.

Under Art 1198 (2) of the Civil Code, the


debtor shall lose every right to make use
of the period when he does not furnish
to

the

creditor

the

guaranties

or

securities which he has promised.

In 2008, Lita met and married Jaime. They


now have a child of their own.
While on a tour with her former high school
classmates in a remote province of China in
2010, Lita was surprised to see Lito or
somebody who looked exactly like him, but
she was sure it was Lito because of the

VII. Lito was a commercial pilot who flew for

extreme surprise that registered in his face

Pacific-Micronesian Air. In 1998, he was the

when he also saw her. Shocked,

co-pilot of the airline's Flight MA916 that

immediately fled to her hotel and post haste

mysteriously disappeared two hours after

returned to the country the next day. Lita

take-off from Agana, Guam, presumably

now comes to you for legal advice. She asks

over the Pacific Ocean. No trace of the plane

you the following questions:

she

and its 105 passengers and crew was ever


found despite diligent search; Lito himself

VII. (l) If Lito is alive, what is the status of

was never heard of again. Lito left behind

his marriage to Lita? (1%)

his wife, Lita, and their two children.


Never Let The Odds Keep You From Pursuing What You Know In Your Heart You Were Meant To Do.Leroy Satchel Paige

Page 125 of 180

Civil Law Q&As (2007-2013)

hectorchristopher@yahoo.com

(A) The marriage subsists because the marital


bond has not been terminated by death.

(B) The marriage is valid. After an absence of


more

(B) The marriage was terminated when Lita


married Jaime.

dbaratbateladot@gmail.com

than

10

years,

Lito

is

already

presumed dead for all purposes.


(C) The marriage is void. Lito's mere absence,

(C) The marriage subsists because Lita's

however lengthy, is insufficient to authorize


Lita to contract a subsequent marriage.

marriage to Jaime is void.


(D) The marriage is terminated because Lito is

(D) The marriage is void. If Lito is indeed alive,

presumed dead after his plane has been

his marriage to Lita was never dissolved

missing for more than 4 years.

and they can resume their marital relations

(E) The marriage can be formally declared

at any time.

terminated if Lito would not resurface.


SUGGESTED ANSWER:
SUGGESTED ANSWER:

Any answer is correct.

(C) The marriage subsists because Lita's

Under Art 390 of the Civil Code, after an

marriage to Jaime is void.

absence of seven years, it being unknown

For

the

purpose

of

contracting

the

whether or not the absentee still lives,

subsequent marriage under Art 41 of the

he

Family Code, the spouse present must

purposes, except for those of succession.

institute

as

This provision was not repealed by the

provided in the Family Code for the

present Family Code. Applying this to

declaration of presumptive death of the

the problem, (A) may be correct. (B) may

absentee, without prejudice to the effect

also be correct. (C) and

of

(D) may also be correct under Art 41 of

the

summary

reappearance

proceeding

of

the

absent

spouse.

shall

presumed

dead

doe

all

the Family Code.

VIII.Which
VII. (2) If Lito is alive, what is the status of

be

of

the

following

actions

or

defenses are meritorious: (1%)

Lita's marriage to Jaime? (1%)


(A) An action for recovery of down payment
(A) The marriage is valid because Lita's

paid under a rescinded oral sale of real

marriage to Lito was terminated upon Lito's

property.

disappearance for more than seven years.

(B) A defense in an action for ejectment that

the lessor verbally promised to extend or

renew the lease.

Never Let The Odds Keep You From Pursuing What You Know In Your Heart You Were Meant To Do.Leroy Satchel Paige

Page 126 of 180

Civil Law Q&As (2007-2013)

hectorchristopher@yahoo.com

down payment on the basis of solution

(C) An action for payment of sum of money filed


against one who orally promised to answer
another's debt in case the latter defaults.
(D) A defense in an action for damages that the
debtor has sufficient, but unliquidated
assets to satisfy the credit acquired when it
becomes due.
(E) None of the above.
SUGGESTED ANSWER:
(A) An action for recovery of down
payment paid under a rescinded oral sale
of real property.
An oral sale of real property is an
unenforceable

contract

under

the

Statute of Frauds. Since, in the problem,


the vendee paid down payment, it takes
it out of the ambit of Statute of Frauds.
The rescission here must be in the sense
of resolution of the reciprocal obligation
arising from the contract of sale. If
rescinded (resolved) by the vendee on
account of the vendors failure to deliver
the thing sold, the parties will go back to
their status prior to the contract. If the
vendor

refuses

to

return

dbaratbateladot@gmail.com

the

down

payment, then the vendee can file an


action to recover the down payment.
If, on the other hand, the vendor and the
vendee mutually agree to rescind i.e.
cancel the contract, the vendee likewise
can file an action for the recovery of the

indebiti.

can be enforced in a court action. (Art


1403, Civil Code)
ALTERNATIVE ANSWER:
(E) None of the above.
(a) The recovery of the down payments

(d) The fact that a debtor has unliquidated


assets does not excuse him from paying
his debt.

should be made in the same action for

(e) In the technical meaning of rescission

rescission. Otherwise, it would be a

under Art 1191 of the Civil Code will be

ground for dismissal under Rule 2, Sec 4

adhered to, then there is no absolutely

of Rules of Court.

correct answer. Hence, letter E is also a

(b) Lease of a real property is covered by the

possible answer.

Statute of Frauds. Furthermore, it also


consists of interest in real property.

IX. Betty entrusted to her agent, Aida,

Hence, it must be in writing. (Art 1403,

several pieces of jewelry to be sold on

Civil Code)
(c) A contract of guaranty is a promise to
answer for the debt of another and
hence, it is also covered by the Statute

commission with the express obligation to


turn over to Betty the proceeds of the sale,
or to return the jewelries if not sold in a
month's

time.

Instead

of

selling

the

jewelries, Aida pawned them with the

of Frauds. It must be in writing before it


Never Let The Odds Keep You From Pursuing What You Know In Your Heart You Were Meant To Do.Leroy Satchel Paige

Page 127 of 180

Civil Law Q&As (2007-2013)

Tambunting

Pawnshop,

hectorchristopher@yahoo.com

and

used

dbaratbateladot@gmail.com

the

(C) I will rule in favor of Tambunting. Its good

money for herself. Aida failed to redeem the

faith takes precedence over the right of

pawned jewelries and after a month, Betty

Betty to recover the jewelries.

discovered what Aida had done. Betty

(D) I will rule in favor of Tambunting. Good

brought criminal charges which resulted in

faith is always presumed. Tambunting's

Aida's conviction for estafa.

lawful acquisition in the ordinary course of


business coupled with good faith gives it

Betty thereafter filed an action against

legal right over the jewelries.

Tambunting Pawnshop for the recovery of


the

jewelries.

Tambunting

raised

the

SUGGESTED ANSWER:

defense of ownership, additionally arguing

(A) I will rule in favor of Betty. My ruling

that it is duly licensed to engage in the

is based on the Civil Code provision that

pawnshop and lending business, and that it

one who has lost any movable or has

accepted the mortgage of the jewelry in


good faith and in the regular course of its
business.

If you were the judge, how will you decide

been unlawfully deprived thereof may


recover it from the person in possession
of the same. Tam bunting's claim of good
faith is inconsequential.

the case? (1%)


Although possession of movable property
(A) I will rule in favor of Betty. My ruling is

acquired in good faith is equivalent to a

based on the Civil Code provision that

title, nevertheless, one who has lost any

one who has lost any movable or has

movable or has been unlawfully deprived

been unlawfully deprived thereof may

thereof may recover it from the person

recover it from the person in possession

in possession of the same. Betty has

of the same. Tam bunting's claim of good

been deprived unlawfully of her jewelries

faith is inconsequential.

by the estafa committed by Aida. The

(B) I will rule in favor of Betty. Tambunting's

pledge of the said jewelries by Aida to

claim of good faith pales into insignificance

Tambunting pawnshop is void because

in light of the unlawful deprivation of the

the pledgor is not the owner (Art 2085

jewelries. However, equity dictates that

(2), Civil Code). Tambuntings claim of

Tambunting must be reimbursed for the


pawn value of the jewelries.

good faith is inconsequential, because,


aside from good faith, Tambunting must

prove also that it acquired the jewelries

retain the jewelries until reimbursed by

at a public sale in order to be able to


Never Let The Odds Keep You From Pursuing What You Know In Your Heart You Were Meant To Do.Leroy Satchel Paige

Page 128 of 180

Civil Law Q&As (2007-2013)

hectorchristopher@yahoo.com

Betty the amount of loan including


interest (Art 559, Civil Code).

The only exception the law allows is


when there is acquisition in good faith of
the possessor at a public sale, in which
case, the owner cannot obtain its return
without reimbursing the price (Dizon v.
Suntay, 47 SCRA 160, Sept 29, 1972).

X. Arlene owns a row of apartment houses


in Kamuning, Quezon City. She agreed to
lease Apartment No. 1 to Janet for a period
of 18 months at the rate of P10,000 per
month. The lease was not covered by any
contract. Janet promptly gave Arlene two
(2) months deposit and 18 checks covering
the rental payment for 18 months. This
show of good faith prompted Arlene to
promise Janet that should Arlene decide to
sell the property, she would give Janet the
right of first refusal.
X. (1) Not long after Janet moved in, she
received news that her application for a
Master

of

Laws

scholarship

at

King's

College in London had been approved.


Since her acceptance of the scholarship
entailed a transfer of residence, Janet asked
Arlene

to

payments

return
she

the

made.

dbaratbateladot@gmail.com

the action is based, is unenforceable.

advance
Arlene

rental
refused,

prompting Janet to file an action to recover


the payments. Arlene filed a motion to
dismiss, claiming that the lease on which

the cause of action does not seek to


If you were the judge, would you grant
Arlene's motion? (1%)

(A) Yes, I will grant the motion because the

enforce any right under the contract of


lease.
SUGGESTED ANSWER:

lease contract between Arlene and Janet

(D) No. I will not grant the motion

was not in writing, hence, Janet may not

because the cause of action does not

enforce any right arising from the same

seek to enforce any right under the

contract.

contract of lease.

(B) No, I will not grant the motion because to


allow

Arlene

payments

to

would

retain
amount

the
to

advance
unjust

enrichment.
(C) Yes, I will grant the motion because the
action for recovery is premature; Janet
should first secure a judicial rescission of
the contract of lease.
(D) No. I will not grant the motion because

Janet is not asking for the continued use


of the leased premises. Moreover, the
contract

is

aside

the

ambit

of

the

Statute of Frauds as the same has


already been partially performed.

X. (2)Assume that Janet decided not to


accept

the

scholarship

and

continued

leasing Apartment No. 1. Midway through

Never Let The Odds Keep You From Pursuing What You Know In Your Heart You Were Meant To Do.Leroy Satchel Paige

Page 129 of 180

Civil Law Q&As (2007-2013)

hectorchristopher@yahoo.com

As such, it is covered by the Statute of

the lease period, Arlene decided to sell


Apartment No. 1 to Jun in breach of her
promise to Janet to grant her the right of
first refusal. Thus, Janet filed an action
seeking the recognition of her right of first
refusal, the payment of damages for the
violation of this right, and the rescission of
the sale between Arlene and Jun.
Is Janet's action meritorious? (1%)

(A) Yes, under the Civil Code, a promise to buy


and sell a determinate thing is reciprocally
demandable.
(B) No,

the

promise

to

buy

dbaratbateladot@gmail.com

and

sell

determinate thing was not supported by a


consideration.
(C) Yes, Janet's right of first refusal was clearly
violated when the property was not offered
for sale to her before it was sold to Jun.
(D) No, a right of first refusal involves an
interest over real property that must be
embodied in a written contract to be
enforceable.
(E) None of the above.
SUGGESTED ANSWER:
(D) No, a right of first refusal involves an
interest over real property that must be
embodied in a written contract to be
enforceable.
The right of first refusal involves a
transfer of interest in the real property.

be the subject of legal relations.


Frauds under Art 1403 (2)(e) of the Civil
Code. It must be in writing in order to be
enforceable.

d) Juridical capacity cannot exist


without capacity to act.

2. Which of the following is NOT a restriction


on ones capacity to act?

2012

Taxation Law Exam

MCQ (October 14, 2012)

a) Minority
b) Marriage
c) Deaf-mute
d) Civil Interdiction

1. Which of the following is NOT included in


the attributes of juridical capacity?

SUGGESTED ANSWER:

a) Juridical capacity is inherent in

This question should be disregarded.

every natural person, and therefore

(NOTE: There is no correct answer among

it is not acquired.

the choices given. All choices are restrictions

b) Juridical

capacity

is

lost

only

through death.
c) Juridical capacity is the fitness to

on ones capacity to act. While Marriage is


the only one not mentioned in Articles 38
and 39 of the NCC as a restriction on

Never Let The Odds Keep You From Pursuing What You Know In Your Heart You Were Meant To Do.Leroy Satchel Paige

Page 130 of 180

Civil Law Q&As (2007-2013)

hectorchristopher@yahoo.com

dbaratbateladot@gmail.com

capacity to act, it restricts the capacity of a

b)

married person in cases of adoption.)

Philippine law.
c)

3. This

attribute

or

incident

of

case

determine whether it is a conflict-of-laws


case or one covered by domestic law.

b) Foreign element

No, the governing law is Spanish

will. He could have observed Spanish Law

4. The capacity of an heir to succeed shall be


governed by the:
a) national law of the decedents heirs
country

where

the

decedent was a resident at the time


of his death
c) national law of the person who
died
d) law

d)

observed by the testator in executing his

d) Forum non conveniens

the

act of the testator.

Note: The facts do not state the Law

c) Jurisdiction

of

No, attestation clause is not an

law.

a) Cause of action

b) law

Yes, the will is not valid under

or Philippine Law (see comment of Tolentino


to Art. 815 NCC in 3Tolentino117, 1992). If
he observed Spanish Law, the opposition is
not correct because the will is valid under
Spanish Law, hence choice (d) is the correct
answer. If he observed Philippine Law, the
opposition

is

still

not

correct

because

Philippine Law does not require the testator


to sign the Attestation Clause of his will,

of

the

properties

of

country
the

where

the

decedent

are

located.

5. Atty. BUKO, a Filipino, executed a will while


he was in Spain. The attestation clause of
the said will does not contain Bukos
signature. It is valid under Spanish law. At
its probate in Manila, it is being opposed on
the ground that the attestation clause does
not contain BUKOs signature.
Is the opposition correct? Choose the best
answer..
a) Yes, because it is a fatal defect.

said clause not being his act. In such case,


choice (c) is the correct answer).

6. Ramon, a Filipino, executed a will in


Manila, where he left his house and located
in BP Homes Paraaque in favor of his
Filipino

son,

Ramgen.

Ramons

other

children RJ and Ramona, both Turkish


nationals, are disputing the bequest to
Ramgen. They plotted to kill Ramgen.
Ramon learned of the plot, so he tore his
will in two pieces out of anger. Which
statement is most accurate?
a) The mere act of Ramon Sr. is

immaterial because the will is still

readable.

Never Let The Odds Keep You From Pursuing What You Know In Your Heart You Were Meant To Do.Leroy Satchel Paige

Page 131 of 180

Civil Law Q&As (2007-2013)

hectorchristopher@yahoo.com

b) The mere act of tearing the will


amounts to revocation.
c) The

tearing

of

the

will

may

amount to revocation if coupled


with intent of revoking it.
d) The

act

of

tearing

the

will

is

material.

7. Even if the applicable law is a foreign law, a


count in the Philippines may be constrained
to apply Philippine law under any of the
following instances, except:
a) when the foreign law, judgment or
contract is contrary to a sound and
important

public

policy

of

the

forum;
b) when the property subject of the
case is located outside of the
Philippines;
c) when the foreign law or judgment
is penal in nature;
d) when the foreign law is procedural
in nature.

8. If a will is executed by a testator who was


born

Filipino

citizen

but

became

naturalized Japanese citizen at the time of


his

death,

testamentary

what

law

provisions

will
if

govern
the

will

its
is

executed in China and the property being


disposed is located in Indonesia?
a) Chinese law
b) Philippine law

dbaratbateladot@gmail.com

c) Indonesia law

d) Japanese law

a) American law
b) Philippine law
9. A Japanese national and a Filipino national
entered into a contract for services in

c) Canadian law
d) Japanese law

Thailand. The services will be rendered in


Singapore. In case of breach, what law will

11. A French national revokes his will in Japan


where he is domiciled. He then changed his

govern?

domicile to the Philippines where he died.

a) Thailand law

The revocation of his will in Japan is valid

b) Philippine law

under Japanese law but invalid under

c) Singapore law

Philippine law. The affected heir is a

d) Japanese law

Malaysian
10. Pedro

(Filipino)

and

his

wife

Jane

national

residing

in

the

Philippines. What law will apply?

(American) executed a joint will in Canada,

a) Japanese law

where such joint will is valid. In case the

b) Philippine law

joint will is probated in Japan, what law

c) French law

will govern the formalities of the joint will?

d) Malaysian law

Never Let The Odds Keep You From Pursuing What You Know In Your Heart You Were Meant To Do.Leroy Satchel Paige

Page 132 of 180

Civil Law Q&As (2007-2013)

hectorchristopher@yahoo.com

Pedro.

12. In the absence of contrary stipulation in a


marriage settlement, property relations of
Filipino spouses shall be governed by --a) Philippines laws
b) Law of the place where the spouses
reside
c) Law

of

the

place

where

the

properties are situated


d) Law of the place where they were
married.

13. The will of a Filipino executed in a foreign


country --a) cannot

be

probated

in

the

Philippines;
b) may be probated in the Philippines
provided

that

properties

in

the

estate are located in the Philippines;


c) cannot be probated before the death
of the testator;
d) may

be

probated

Philippines

provided

in

the

it

was

executed in accordance with the


laws of the place where the will
was executed.

14. Pedro (Filipino and Bill (American) entered


into a contract in Australia, whereby it was
agreed that Pedro will build a commercial
building for Bill in the Philippines, and in
payment for the construction, Bill will
transfer and convey his cattle ranch located
in Japan in favor of

dbaratbateladot@gmail.com

In case Pedro performs his

Filipino), by whom she had a daughter,


Regine. In 2009, Regine married James

obligation, but Bill fails or refuses to pay,

(son of Justine with Lea) in California,

what law will govern?

where such marriage is valid.

a) American law
b) Philippine law

15. What is the current status of the

c) Australian law

marriage of Charice and Justine under

d) Japanese law

Philippine laws?

(Facts for item numbers 15-18)


In 1989, Charice (Filipina) and Justine
(American),

were

married

in

the

Philippines. In 1990, they separated and


Justine

went

to

Las

Vegas

where

he

obtained a divorce in the same year. He


then married another Filipina, Lea, in
Canada on January 1, 1992. They had two
(2) sons, James and John (who were both
born in 1992). In 1993, after failing to hear
from Justine, Charice married Bugoy (a

a) Valid
b) Void
c) Voidable
d) Dissolved
(Note: While Art 26 of the FC does not
categorically provide that the first marriage
is dissolved by the divorce obtained by the
foreign spouse abroad, but provides that
such divorce merely gives the Filipino spouse
the capacity to contract a second marriage, it

Never Let The Odds Keep You From Pursuing What You Know In Your Heart You Were Meant To Do.Leroy Satchel Paige

Page 133 of 180

Civil Law Q&As (2007-2013)

hectorchristopher@yahoo.com

dbaratbateladot@gmail.com

is believed that the dissolution of the first

examinations unless he marries Princess.

marriage us the necessary consequence of

As a consequence of the threat, Ricky

the foreign divorce.)

married Princess. Can the marriage be


annulled on the ground of intimidation

16. What id the status of the marriage between


Charice and Bugoy under Philippine laws?

under Article 45 of the Family Code?


Choose the best answer.

a) Valid

a) Yes, because without the threat,

b) Void

Ricky

c) Voidable

Princess.

d) Unenforceable

would

not

have

married

b) Yes, because the threat to enforce


the claim of Princess vitiates the

17. What is the status of the marriage between

consent of Ricky in contracting the

Charice and Bugoy under Philippine laws?

marriage.

a) Valid

c) No, because the threat made by

b) Void

Marforth is just and legal.

c) Voidable

d) No, because Marforth is not a party

d) Unenforceable

to the contract of marriage between


Princess and Ricky.

18. What is the status of the marriage between


Regine and James under Philippine laws?
a) Valid

Malolos City from Franco for P 1Million. A

b) Void

contract was executed between them which

c) Voidable

already vested upon Audrey full ownership

d) Unenforceable

19. Ricky

and

Princess

of
were

sweethearts.

Princess became pregnant. Knowing that


Ricky is preparing for the examinations,
Marforth, a lawyer and cousin of Princess,
threatened

Ricky

20. Audrey, single, bought a parcel of land in

with

the

filing

of a

complaint for immorality in the Supreme


Court, thus preventing him from taking

the

property,

although

payable

in

monthly installments for a period of four (4)


years. One (1) year after the execution of
the contract, Audrey got married to Arnel.
They

executed

marriage

settlement

whereby they agreed that their properties


shall be governed by the regime of conjugal
partnership

of

gains.

Thereafter,

subsequent installments were paid from the


conjugal partnership funds. Is the land

conjugal or paraphernal?
Never Let The Odds Keep You From Pursuing What You Know In Your Heart You Were Meant To Do.Leroy Satchel Paige

Page 134 of 180

Civil Law Q&As (2007-2013)

hectorchristopher@yahoo.com

a) The land is conjugal because the


installments were paid from the
conjugal partnership funds.
b) The land is paraphernal because
ownership thereof was acquired
before the marriage.
c) The

land is

both

conjugal

and

paraphernal funds of installments


were paid from both the personal
funds of Audrey and the conjugal
partnership funds.
d) The land is paraphernal because it
was Audrey who purchased the
same.

21. Ernesto donated a mobile phone worth P


32,000 to Hubert orally and delivered the
unit

to

Hubert

who

accepted.

Which

statement is most accurate?


a) The donation is void and Ernesto
may get mobile phone back.
b) The donation is void but Ernesto
cannot get the mobile phone back.
c) The donation is voidable and may be
anulled.
d) The donation is valid.

22. Agay, a Filipino citizen and Topacio, an


Australian
consular
Australia.

citizen,
office

got

of

According

dbaratbateladot@gmail.com

marriage is celebrated in accordance with

married

the
to

in

Philippines
the

laws

the
in
of

Australia, a marriage solemnized by a


consular official is valid, provided that such

was celebrated.
the laws of such consular official. Under
Philippine law, what is the status of the
marriage of Agay and Topacio? Choose the
best answer.
a)

Void, because the consular official

only

has

authority

to

solemnize

marriages between Filipinos.


b)

(Note: The issues in the problem is whether


or not the fact that one of the parties to the
marriage was an alien constituted absence
of authority or mere irregularity of authority.
The problem only give the choice, letter (a),
in case it is interpreted as absence of
authority. The problem does not give a

Valid, because according to the laws

choice in case it is interpreted as an

of Australia, such consular official has

irregularity thereby making all the other

authority to celebrate the marriage.


c)

Voidable,

because

there

answers wrong).
is

an

irregularity in the authority of the consular


official to solemnize marriages.

23. Separation of property between spouses

d)

during the marriage may take place only:

Valid, because such marriage is

recognized as valid in the place where it

a) by agreement of the spouses.

Never Let The Odds Keep You From Pursuing What You Know In Your Heart You Were Meant To Do.Leroy Satchel Paige

Page 135 of 180

Civil Law Q&As (2007-2013)

hectorchristopher@yahoo.com

dbaratbateladot@gmail.com

b) If one of the spouses has given

b) No trial shall be held without the

ground for legal separation.

6-month cooling off period being

c) Upon order of the court.

observed.

d) If one spouse has abandoned the

c) The spouses will be entitled to live

other.

separately upon the start of the


trial.

24. The husband may impugn the

d) The prosecuting attorney has

legitimacy of his child but not on the

to conduct his own investigation.

ground that:
a) the

wife

is

suspected

of 27. A husband by chance discovered

infidelity.

hidden treasure on the paraphernal

b) the husband had a serious illness

property of his wife. Who owns the

that prevented him from engaging in

discovered treasure?

sexual intercourse.

a) The half pertaining to the

c) they were living apart.

husband (finder) belongs to the

d) he is physically incapable of

conjugal partnership.

sexual intercourse.

b) The half pertaining to the wife (as


owner) belongs to the conjugal

25. A marriage is void if:

partnership.

a) solemnized with a marriage

c) One half shall belong to the

license issued without complying

husband as finder and the other

with the required 10-day posting.

half shall belong to the wife as

b) solemnized by a minister whom

owner of the property.

the parties believe to have the

d) a and b

authority.
c) between parties both 23 years of

28. Which of the following marriages is void

age but without parental advice.

for reasons of public policy?

d) none of the above

a) Between brothers and sisters,


whether of the full or half blood.

26. In legal separation, which is not

b) Between step-parents and step

correct?

children.

a) The aggrieved spouse may file the

c) Between parents-in-law and

action within five (5) years from the

children-in-law.

time of the occurrence of the cause.

d) b and c

Never Let The Odds Keep You From Pursuing What You Know In Your Heart You Were Meant To Do.Leroy Satchel Paige

Page 136 of 180

Civil Law Q&As (2007-2013)

29. The

following

hectorchristopher@yahoo.com

constitute

the

different

dbaratbateladot@gmail.com

a) Children conceived or born outside


a valid marriage.

circumstances or case of fraud which will


serves as ground for the annulment of a

b) Children

marriage, except?

born

under

valid

marriage, which was later declared


previous

void because of the psychological

conviction by final judgment of the

incapacity of either or both of the

other party of a crime involving

spouses.

a) Non-disclosure

of

the

moral turpitude.
b) Concealment

c) Children
of

conceived

and

outside a valid marriage.

sexually-

transmissible disease, regardless of

d) Children

born

under

its nature, existing at the time of the

marriage,

marriage.

obtained a legal separation.

c) Concealment

of

drug

born

but

the

parents

valid
later

addiction,

habitual alcoholism, homosexuality

32. An illegitimate child may use the surname

or lesbianism existing at the time of

of

marriage.

established

d) Concealment by the wife or the


husband of the fact of sexual

his

father
in

when
any

his
of

filiation

the

is

following

instances, except:
a) Filiation has been recognized by the
father through the record of birth

relations prior to the marriage.

appearing in the civil register


30. Which of the following is not a requisite for

in a public document.

a valid donation propter nuptias?


a) The donation must be made before

donation

shall

be

automatically revoked in case of


non-celebration of the marriage.
c) The donation must be made in
consideration of the marriage.
d) The donation must be made in favor
of one or both of the future spouses.
31. Who are illegitimate children?

c) Private handwritten instrument is


made by the father acknowledging

the celebration of the marriage.


b) The

b) Admission of filiation by the father

his filiation.
d) Affidavit by the mother stating
the name of his true father.

33. Under RA 8043, an adopter is required to


be at least ____ years old and ____ years
older than the child to be adopted at the
time of the application unless the adopter is
the parent by nature of the child.

a) 30 and 15

b) 27 and 16

Never Let The Odds Keep You From Pursuing What You Know In Your Heart You Were Meant To Do.Leroy Satchel Paige

Page 137 of 180

Civil Law Q&As (2007-2013)

hectorchristopher@yahoo.com

dbaratbateladot@gmail.com

c) 50 and 10

d) Subjects the child or allows him

d) 18 and 15

to

be

subjected

to

acts

of

lasciviousness.
34. Under RA 8043, a child qualified to be
adopted is any person below _____ years

37. Which of the following statements is

old.

wrong?
a) 18

a) The possessor in bad faith shall

b) 21

reimburse the fruits received and

c) 15

those which the legitimate possessor

d) 16

could have received.

35. Which of the following DOES NOT result in


permanent

termination

of

parental

authority?

reimbursement

expenses

and

for

necessary

those

for

gathering

the
and

preservation of the fruits.

b) Death of the child.


c) Emancipation of the child.
d) Conviction of the parents of a
crime which carries with it the
penalty of civil interdiction.

36. The court, in an action filed for the


purpose, may suspend parental authority if
the parent or the person exercising parental
authority commits any of the following acts,
except:

c) The possessor in bad faith is not


entitled to a refund of ornamental
expenses.
d) The possessor in bad faith is
entitled to a refund of useful
expenses.

38. Which phrase most accurately completes


the statement The expenses incurred in
improvements

for

the

luxury

or

mere

pleasure shall not be refunded to thew


the

child

with

excessive

harshness or cruelty.
b) Gives the child corrupting orders,
counsel or example.
c) Compels the child to take up a
course in college against his/her
will.

of

production,

a) Death of the parents.

a) Treats

b) The possessor in bad faith has right

possessor in bad faith:


a) but he may remove the objects for
which such expenses have been
incurred, provided that the thing
suffers no injury thereby, and that
the

lawful

possessor

prefer to retain them.

does

not

Never Let The Odds Keep You From Pursuing What You Know In Your Heart You Were Meant To Do.Leroy Satchel Paige

Page 138 of 180

Civil Law Q&As (2007-2013)

hectorchristopher@yahoo.com

b) and he may not remove the objects


for which such expenses have been

dbaratbateladot@gmail.com

of the dominion over the property as owner.


What action is being referred to?
a) Accion publiciana

incurred.

b) Accion reinvindicatoria

c) and he may not remove the objects


for which such expenses have been

c) Accion interdictal

incurred, unless he pays the value

d) Quieting of Title

they may have at the time he


42. A summary action to recover physical or

entered into possession.


d) but he may remove the objects for

material possession only and must be

which such expenses have been

brought within one (1) year from the time

incurred.

the cause of action arises. What action is


being referred to?
a) Accion publiciana

39. The following are the limitations on the


right of ownership imposed by the owner

b) Accion reinvindicatoria

himself, except:

c) Accion interdictal
d) Quieting of Title

a) Will/Succession
b) Mortgage

43. The following things are property of public

c) Pledge

dominion, except:

d) Lease

a) ports and bridges constructed by


the State.

40. A plenary action for the recovery of the


possession

of

real

estate,

upon

mere

b) vehicles and weapons of the Armed


Forces of the Philippines.

allegation and proof of a better right

c) rivers.

thereto, and without allegation of proof of


title. This action can only be brought after

d) lands reclaimed by the state from


the sea.

the expiration of one (1) year. What action is


being referred to?
a) Accion publiciana
b) Accion reinvindicatoria
c) Accion interdictal
d) Quieting of Title

SUGGESTED ANSWER:
This

question

should

be

disregarded

because there is no correct answer.


(Note: At first glance, one gets the impression
that vehicles and weapons of the AFP are

41. Action to recover real property based on


ownership. Here, the object is the recovery

not property of the public domain. But they


are actually property of the public dominion

under the second paragraph of Art 420 of


Never Let The Odds Keep You From Pursuing What You Know In Your Heart You Were Meant To Do.Leroy Satchel Paige

Page 139 of 180

Civil Law Q&As (2007-2013)

hectorchristopher@yahoo.com

---

the NCC. Property of the state which are not


for public use but are intended for some
public service are properties of the public
dominion. While the vehicles and weapons
of the AFP are not for public use, they are
used for the defense of the State which is a
public service.)

44. Which of the following statements is


wrong?
a) patrimonial property of the state,
when

no

longer

intended

for

public use or for public service,


shall become property of public
dominion.
b) all property of the State, which is
not

of

public

dominion,

is

patrimonial property.
c) The property of provinces, cities and
municipalities

is

property

public

for

divided
use

into
and

patrimonial property.
d) Property is either of public dominion
or of private ownership.

45. The following cannot ask for the reduction


of inofficious donation, except:
a) Creditors of the deceased
b) Devisees or legatees
c) Compulsory heirs of the donor
d) The surviving spouse of the donee.
46. Donation is perfected from the moment

dbaratbateladot@gmail.com

a) the donee accepts the donation.


b) the donor executes the deed of
donation.
c) the donor knows of the donees
acceptance even if the latter has

48. It is a conduct that may consist of giving,


doing, or not doing something.
a) Obligation
b) Juridical necessity
c) Prestation
d) Contract

not received the copy of the deed

49. It is

a juridical relation arising

of donation.

from lawful,

voluntary and unilateral acts

d) the donee confirms that the donor

based on the principle that no one should

has learned the formers acceptance.

unjustly enrich himself at the expense of


another.

47. The following are the elements of an


obligation, except:

a) Quasi-contract
b) Quasi-delict

a) Juridical/Legal Tie

c) Cotract

b) Active subject

d) Delict

c) Passive subject
d) Consideration

50. The following are the elements of quasidelict, except:

Never Let The Odds Keep You From Pursuing What You Know In Your Heart You Were Meant To Do.Leroy Satchel Paige

Page 140 of 180

Civil Law Q&As (2007-2013)

hectorchristopher@yahoo.com

a) Act or omission

dbaratbateladot@gmail.com

54. It is an international evasion of the faithful

b) Fault/negligence

performance of the obligation.

c) Damage/injury

a) Negligence

d) Pre-existing contract

b) Fraud
c) Delay

51. A debtor is liable for damages in case of

d) Mistake

delay if he is guilty of any of the following,


except:

55. The following are the requisites of fortuitous


event, except:

a) default (mora)
b) mistake

a) Cause is independent of the will of


the debtor.

c) negligence (culpa)
d) breach through contravention of the

b)

The

event

is unforeseeable/unavoidable.

tenor thereof

c) Occurrence
52. This term refers to a delay on the part of

renders

it

absolutely

impossible for the debtor to fulfill his

both the debtor and creditor in reciprocal

obligation

obligations.

impossibility must be absolute not

a) Mora accipiendi

d) Debtor

c) Compensation morae

manner;

contributed

to

the

creditor.

53. The following are the requisites of mora


solvendi, except:

56. A debtor may still be held liable for loss or


damages even if it was caused by a

a) Obligation pertains to the debtor


determinate,

due,

demandable, and liquidated.


b) Obligation was performed on its
maturity date.
c) There is judicial or extrajudicial
demand by the creditor.
d) Failure of the debtor to comply with
such demand.

normal

aggravation of the injury to the

d) Solution indibiti

is

partial, otherwise not force majeure.

b) Mora solvendi

and

in

fortuitous event in any of the following


instances, except:
a) The debtor is guilty of dolo, malice
or bad faith, has promised the same
thing to two or more persons who do
not have the same interest.
b) The debtor contributed to the loss.
c) The

thing

generic.

to

be

delivered

is

Never Let The Odds Keep You From Pursuing What You Know In Your Heart You Were Meant To Do.Leroy Satchel Paige

Page 141 of 180

Civil Law Q&As (2007-2013)

d) The

creditor

negligence

is guilty

or

contravened

hectorchristopher@yahoo.com

delay

the

of fraud,
or

tenor

if

c) No, because a motion to dismiss is a


prohibited pleading.

he

of

dbaratbateladot@gmail.com

the

d) Yes, because Fermin and Toti should

obligation.

also

pay

their

share

of

the

obligation.
57. Buko, Fermin and Toti bound themselves
solidarily to pay Ayee the amount of P
5,000.00.

Suppose

Buko

paid

the

59. Buko, Fermin and Toti are solidarily debtors


of

Ayee.

Twelve

(12)

years

after

the

obligation, what is his right as against his

obligation became due and demandable,

co-debtors?

Buko paid Ayee and later on asked for

a) Buko cas ask for reimbursement

reimbursement

of

Fermins

and

Totis

shares. Is Buko correct? Why?

from Fermin and Toti.


b) Buko can sue Fermin and Toti for

a) No, because the obligation has


already prescribed.

damages.

b) Yes,

c) Buko can sue for rescission.

because

the

obligation

is

solidary.

d) Buko can claim a refund from Ayee.

c) No, because in solidary obligation


any one of the solidary debtors can

58. Buko, Fermin and Toti bound themselves


solidarily

to

pay

Ayee

the

sum

of

pay the entire debt.

10,000.00. When the obligation became due

d) Yes, because Fermin and Toti will be

and demandable, Ayee sued Buko for the

unduly enriched at the expense of

payment of the P 10,000.00. Buko moved to

Buko.

dismiss on the ground that there was


failure to implead Fermin and Toti who are

60. Buko, Fermin and Toti are solidary

indispensable parties. Will the motion to

debtors

dismiss prosper? Why?

300,000.00 which has fallen due. The

a) Yes, because Fermin and Toti should


have

been

impleaded

as

their

because

the

loan

obligation

of

creditor has, however, condoned Fermins


entire share in the debt. Since Toti has
become insolvent, the creditor makes a

obligation is solidary.
b) No,

under

creditor

may

proceed against any one of the


solidary debtors or some or all of
them simultaneously.

demand on Buko to pay the debt. How


much, if any, may Buko be compelled to
pay?
a) P 200.000.00
b) P 300,000.00

c) P 100,000.00
Never Let The Odds Keep You From Pursuing What You Know In Your Heart You Were Meant To Do.Leroy Satchel Paige

Page 142 of 180

Civil Law Q&As (2007-2013)

hectorchristopher@yahoo.com

dbaratbateladot@gmail.com

c) That the two (2) debts are not yet


due.

d) P 150,000.00

61. Dina bought a car from Jai and delivered a


check in payment of the same. Has Dina
paid the obligation? Why?
a) No,

not

yet.

promissory

The

notes

delivery

of

payable

to

order, or bills of exchange or


other mercantile documents shall
produce the effect of payment
only when they have been cashed,
or when through the fault of the
creditor they have been impaired.
b) Yes, because a check is a valid legal
tender of payment.
c) It

depends.

If

the

check

is

managers check or cashiers check


it will produce the effect of payment.
If

its

an

ordinary

check,

no

payment.
d) Yes, because a check is as good as
cash.
62. The following are the requisites of legal
compensation, except:
a) That each of the obligors is bound
principally and that he be the same
time a principal creditor of the
other.
b) That both debts consist in a sum of
money, or if the things due are
consumable, they be the same kind,
and also of the same quality if the
latter has been stated.

d) That they be liquidated and


demandable.

63. Which of the following statements is


correct?
a) All contracts are perfected by mere
consent.
b) All

contracts

are

perfected

by

delivery of the object.


c) All contracts are required to be in
writing.
d) All contracts are required to have
a valid consideration.

64. It is a principle which holds that parties are


bound not only by what has been expressly
provided for in the contract but also to the
natural consequences that flow out of such
agreement.
a) Obligatory force of contracts
b) Mutuality of contracts
c) Autonomy of contracts
d) Relativity of contracts

65. It is a principle which holds that contracts


must be binding to both parties and its
validity and effectivity can never be left to
the will of one of the parties.
a) Obligatory force of contracts
b) Mutuality of contracts
c) Autonomy of contracts
d) Relativity of contracts

66. It refers to
is binding

the rule that a contract


not

only between

Never Let The Odds Keep You From Pursuing What You Know In Your Heart You Were Meant To Do.Leroy Satchel Paige

Page 143 of 180

Civil Law Q&As (2007-2013)

hectorchristopher@yahoo.com

parties but extends to the heirs, successors

b)

in interest, and assignees of the parties,

offeree.

provided

c)

that

the

contract

involved

transmissible rights by their nature, or by


stipulation or by law.

dbaratbateladot@gmail.com

Acceptance of the offer by the

Qualified/conditional acceptance of

the offer, which becomes counter-offer.


d)

Subject

matter

a) Obligatory force of contracts

illegal/impossible

b) Mutuality of contracts

communicated.

before

becomes
acceptance

is

c) Autonomy of contracts
d) Relativity of contracts

70. Which of the following statements is


correct?

67. It is rule which holds that the freedom of


the parties to contract includes the freedom
to stipulate, provided the stipulations are
not contrary to law, morals, good customs,

a)

Offers in interrelated contracts are

perfected upon consent.


b)

Offers

in

interrelated

contracts

require a single acceptance.

public order or public policy.

advertisements

c) Business

a) Obligatory force of contracts

are definite

b) Mutuality of contracts

specific acceptance.

c) Autonomy of contracts

d) Advertisements

d) Relativity of contracts

are only

offers

that

require

for

invitations

Bidders
to

make

proposals and the advertiser is


not

68. The following are the ways by which


innominate

contracts

contracts and delicts

71.

unless it

The

following

are solemn

writing), except:
a) Donations of real estate or of

analogous nominate contracts.

movables

d) By the customs of the place.


69. An offer becomes ineffective on any of the
following grounds, except:
civil

the

contracts (Contracts which must appear in

c) By the rules governing the most

a) Death,

accept

appears otherwise.

regulated, except:

b) By the general principles of quasi-

to

highest/lowest bidder,

are

a) By the stipulation of the parties.

bound

interdiction,

insanity/insolvency of either party


before acceptance is conveyed.

if

the

value

exceeds P 5,000.00.
b)

Stipulation to pay interest in loans.

c)

Sale

of

land

through

(authority must be in writing).

an

agent

Never Let The Odds Keep You From Pursuing What You Know In Your Heart You Were Meant To Do.Leroy Satchel Paige

Page 144 of 180

Civil Law Q&As (2007-2013)

hectorchristopher@yahoo.com

dbaratbateladot@gmail.com

d) Construction contract of a 74. The following are the characteristics of


building.

a voidable contract, except:


a) Effective until set aside.

72.

The

following

are

rescissible

b) May be assailed/attacked only in

contracts, except:

an action for that purpose.

a) Entered into by guardian

c) Can be confirmed or ratified.

whenever ward suffers damage more

d) Can be assailed only by either

than of value of property.

party.

b) Agreed upon in representation of


absentees, if absentee suffers lesion

75.

The

following

by more than of value of property.

contracts, except:

are

c) Contracts where fraud is

a) Pactum commissorium

committed on

b) Pactum de non alienando

creditor (accion

pauliana).

c) Pactum leonina

d) Contracts entered into by

d) Pacto de retro

void

minors.
76. The borrower in a contract of loan or
73. The following are the requisites before a

mutuum must pay interest to the lender.

contract entered into in fraud of creditors

a) If there is an agreement in

may be rescinded, except:

writing to the effect.

a) There must be credited existing

b) As a matter of course.

prior to the celebration of the

c) If the amount borrowed is very

contract.

large.

b) There must be fraud, or at least,

d) If the lender so demands at the

the intent to commit fraud to the

maturity date.

prejudice of the creditor seeking


rescission.

77. The liability of the school, its

c) The creditor cannot in any legal

administrators and teachers, or the

manner collect his credit (subsidiary

individual, entity or institution engaged in

character of rescission)

child care over the minor child or damage

d) The object of the contract must

caused by the acts or omissions of the

be legally in the possession of a

unemancipated minor while under their

rd

person in good faith.

supervision, instruction or custody shall be:


a) Joint and subsidiary
b) Principal and solidary

Never Let The Odds Keep You From Pursuing What You Know In Your Heart You Were Meant To Do.Leroy Satchel Paige

Page 145 of 180

Civil Law Q&As (2007-2013)

hectorchristopher@yahoo.com

a) pure

c) Principal and joint


d) Subsidiary and solidary.

78. The creditor has the right to the fruits of


the thing from the time:
a) the thing is delivered.
b) the

obligation

to

deliver

the

things arises.
c) the contract is perfected.
d) the fruits are delivered.

79. If one of the parties to the contract is


without juridical capacity, the contract is:
a) voidable
b) rescissible
c) void
d) unenforceable

80. When both parties to the contract are


minors, the contract is:
a) voidable
b) rescissible
c) void
d) unenforceable

81. When the consent of one of the parties was


vitiated, the contract is:
a) voidable
b) rescissible
c) void
d) unenforceable

82. An obligation which is based on equity and


natural law is known as:

dbaratbateladot@gmail.com

consent from the owner, Mr. Lacas.


b) quasi-contract
c) civil
d) natural

b) valid because all of the essential


requisites of a contract are present.
c) unenforceable

because

Michael

Fermin had no authority but he


83. Consent was given by one in representation
of another but without authority. The
contract is:
a) voidable

sold the car in the name of Mr.


Lacas, the owner.
d) rescissible

because

the

contract

caused lesion to Atty. Buko.

b) rescissible
c) void
d) unenforceable
84. Michael Fermin, without the authority of

85. Which of the following contracts is void?

a) An oral sale of a parcel of land.


b) A sale of land by an agent in a

Pascual Lacas, owner of a car, sold the

public

same car in the name of Mr. Lacas to Atty.

authority from the principal is

Buko. The contract between Atty. Buko and


Mr. Lacas is --a) void because of the absence of

instrument

where

his

oral.
c) A donation of a wrist watch worth P
4,500.00.

Never Let The Odds Keep You From Pursuing What You Know In Your Heart You Were Meant To Do.Leroy Satchel Paige

Page 146 of 180

Civil Law Q&As (2007-2013)

hectorchristopher@yahoo.com

d) A relatively simulated contract

dbaratbateladot@gmail.com

for fulfillment of the obligation even


if he has not tendered payment of

86. Which of the following expresses a correct


principle of law? Choose the best answer.

the purchase price.


c)

a) Failure to disclose facts when there

The contract between the parties is

rescissible.

is a duty to reveal them, does not

d)

constitute fraud.

is subject to ratification by the parties.

The contract between the parties

b) Violence or intimidation does not


render

contract

annullable

if

employed not by a contracting party


but by a third person.

88. Which of the following statements is


wrong?
a) Creditors are protected in cases of

c) A threat to enforce ones claim


through competent authority, if the
claim is legal or just, does not vitiate
consent.

contracts intended to defraud them.


b) Contracts take effect only between
the parties, their assign and heirs,
except in case where the rights and

d) Absolute simulation of a contract


always results in a void contract.

obligations arising from the contract


are

not

transmissible

by

their

nature, or by stipulation or by
87. Aligada orally offered to sell his two-hectare
rice land to Balane for P 10Million. The

provision of law.
c) If a contract should contain some

offer was orally accepted. By agreement, the

stipulation

land was to be delivered (through execution

person,

of a notarized Deed of Sale) and the price

fulfillment

was to be paid exactly one-month from

communicated his acceptance to the

their oral agreement. Which statement is

obligor before its revocation.

most accurate?

on the agreed date despite payment


Balane,

he

favor
may

of

third

demand

its

provided

he

d) In contracts creating real rights,

a) If Aligada refuses to deliver the land

by

in

the

latter

may

not

successfully sue Aligada because


the contract is oral.
b) If Aligada refused to deliver the land,
Balane may successfully sue

third

persons

who

come

into

possession of the object of the


contract are not bound thereby.

89. Which phrase most accurately completes


the statement Any third person who
induces another to violate his contract:

Never Let The Odds Keep You From Pursuing What You Know In Your Heart You Were Meant To Do.Leroy Satchel Paige

Page 147 of 180

Civil Law Q&As (2007-2013)

hectorchristopher@yahoo.com

dbaratbateladot@gmail.com

a) shall be liable for damages only if he


is a party to the same contract.
b) shall be liable for damages to the

93. The

attestation

clause

contains

the

following, except:
a) the number of pages used;

other contracting party.

b) that the testator signed or caused

c) shall not be liable for damages to

another to sign the will and every

the other contracting party.

page thereof in the presence of the

d) shall not be liable for damages if the

instrumental witnesses;

parties are in pari delicto.

c) notary public;
d) the

90. The requisites of succession are as follows,


except:

instrumental

witnesses

witnessed and signed the will and

a) Death of decedent

all the pages thereof in the presence

b) Transmissible estate

of the testator and one another.

c) Existence and capacity of successor,


designated by decedent or law
d) Payment of Taxes

94. The following are the formalities required in


the execution of holographic will, except:
a) Entirely written;
b) Dated;

91. The characteristics of succession are as

c) Signed by testator himself

follows, except:

d) Notarized by a notary public.

a) It is a legal contract.
b) Only property, rights and obligations
to the extent of the value of the
inheritance are transmitted.
c) The transmission takes place only at
the time of death.
d) The transmission takes place either
by will or by operation of law.

95. The

following

are

the

grounds

for

disallowance of wills, except:


a) The formalities required by law have
not been complied with.
b) The testator was insane or mentally
incapable of making will.
c) The will was executed through force

92. The following rights are extinguished by


death, except:
a) Legal support
b) Parental authority
c) Right to inherit
d) Agency

or under duress, or influence of fear


or threats.
d) The will contains an attestation
clause.

Never Let The Odds Keep You From Pursuing What You Know In Your Heart You Were Meant To Do.Leroy Satchel Paige

Page 148 of 180

Civil Law Q&As (2007-2013)

hectorchristopher@yahoo.com

96. It is the omission in the testators will of

dbaratbateladot@gmail.com

consideration, which gives him the right to

one, some or all of the compulsory heirs in

buy

direct line, whether living at the time of

property, from another person, at anytime

execution of the will or born after the death

within the agreed period, at a fixed price.

of the testator. What principle is being

What contract is being referred to?

referred to?

certain

merchandise

or

specified

a) Option Contract

a) reserva troncal

b) Contract to Sell

b) preterition

c) Contract of Sale

c) fideicommissary

d) Lease

d) disposicion captatoria
97. Any disposition made upon the condition
that the heir shall make some provision in
his will in favor of the testator or of any

100.

Which of the following contracts of

sale is void?
a) Sale of EGMs car by KRP, EGMs

other person shall be void. Here, both the

agent,

condition and the disposition are void.

reduced into writing.

What principle is being referred to?

whose

is

not

b) Sale of EGMs piece of land by

a) reserva troncal

KRP,

b) preterition

authority

c) fideicommissary

writing.

d) disposicion captatoria

authority

EGMs
is

agent,
not

whose

reduced

into

c) Sale of EGMs car by KRP, a person


stranger to EGM, without

98. Which phrase most accurately completes


the statement If at the time the contract
of sale is perfected, the thing which is the
object of the contract has been entirely lost:

EGMs consent or authority.


d) Sale of EGMs piece of land by
KRP, a person stranger to EGM,
without EGMs consent or authority.

a) the buyer bears the risk of loss.


b) the contract shall be without any
effect.
c) the seller bears the risk of loss.
d) the buyer may withdraw from the
contract.
99. A contract granting a privilege to a
person,

for which he has paid a

2011 Taxation Law Exam


MCQ (November 13, 2011)
(1)When does a declaration of absence of a
missing person take effect?
(A) Immediately from the issuance of

the declaration of absence.

Never Let The Odds Keep You From Pursuing What You Know In Your Heart You Were Meant To Do.Leroy Satchel Paige

Page 149 of 180

Civil Law Q&As (2007-2013)

hectorchristopher@yahoo.com

(B) 3 months after the publication of


the declaration of absence.

dbaratbateladot@gmail.com

accepted the substitution. Later, however,


the new debtor became insolvent and

(C) 6 months after the publication of

defaulted in his obligation. What is the


effect of the new debtors default upon the

the declaration of absence.


(D) 15 days from the issuance of the
declaration of absence.

original debtor?
(A) The original debtor is freed of
liability since novation took place

(2) The authority that school administrators

and

exercise over school children under their


supervision,

instruction,

or

custody

is

called

his

(B) The original debtor shall pay or

(C) The original debtor remains liable


since he gave no consent to the

(D) special parental authority.

substitution.

(3) Can future inheritance be the subject of a

(D) The original debtor shall pay or


perform 50% of the obligation to

contract of sale?

avoid unjust enrichment on his

(A) No, since it will put the predecessor

part.

at the risk of harm from a tempted


buyer, contrary to public policy.
(B) Yes, since the death of the decedent
is certain to occur.
seller

of

to the new debtor.

(C) ordinary parental authority.

the

him

perform the obligation with recourse

(B) substitute parental authority.

since

relieved

obligation.

(A) legal parental authority.

(C) No,

this

owns

no

inheritance while his predecessor


lives.

(5) Lennie bought a business class ticket from


Alta Airlines. As she checked in, the
manager downgraded her to economy on
the ground that a Congressman had to be
accommodated
Lennie

in

suffered

the
the

business

class.

discomfort

and

(D) Yes, but on the condition that the

embarrassment of the downgrade. She sued

amount of the inheritance can only

the airlines for quasi-delict but Alta Airlines

be ascertained after the obligations

countered

of the estate have been paid.

governed by a contract between them, no

(4) Upon the proposal of a third person, a new


debtor

substituted

the

original

debtor

without the latters consent. The creditor

that,

since

quasi-delict could arise.

her

travel

Is the

was

airline

correct?
(A) No, the breach of contract may in

fact be tortious as when it is

tainted as in this case with

Never Let The Odds Keep You From Pursuing What You Know In Your Heart You Were Meant To Do.Leroy Satchel Paige

Page 150 of 180

Civil Law Q&As (2007-2013)

hectorchristopher@yahoo.com

arbitrariness, gross bad faith, and


malice.
(B) No, denying Lennie the comfort and
amenities of the business class as
provided in the ticket is a tortious
act.
(C) Yes, since the facts show a breach of
contract, not a quasi-delict.
(D) Yes, since quasi-delict presupposes
the

absence

contractual

of

relation

pre-existing
between

the

parties.

(6) Which of the following is an indispensable


requirement in an action for "quieting of
title" involving real property? The plaintiff
must
(A) be

in

actual

possession

of

the

of

the

property.
(B) be

the

registered

owner

property.
(C) have legal or equitable title to the
property.
(D) be

the

beneficial

owner

of

the

property.

(7) X and Y were to marry in 3 months.


Meantime,

to

express

dbaratbateladot@gmail.com

attack. Can Ys heirs get the property?

his

affection,

donated a house and lot to Y, which


donation X wrote in a letter to Y. Y wrote
back, accepting the donation and took
possession of the property. Before the
wedding, however, Y suddenly died of heart

the company of handsome boys. What legal


(A) No, since the marriage did not take

(A) She

place.
(B) Yes, since all the requisites of a
donation

remedy does Lily have?

of

an

immovable

are

present.
(C) No, since the donation and its
acceptance are not in a public
instrument.
(D) Yes, since X freely donated the
property to Y who became its owner.

can

file

an

action

for

annulment of marriage on ground


of fraud.
(B) She can seek a declaration of nullity
of the marriage based on
Renes psychological incapacity.
(C) She can go abroad and file for
divorce in a country that can grant
it.
(D) She has none since she had the

(8) Rene and Lily got married after a brief


courtship. After one month, Lily discovered

opportunity to examine the goods


and freely entered into the marriage.

that while Rene presented himself as a


macho man he was actually gay. He would
not go to bed with her. He kept obscene
magazines of nude men and always sought

(9) Lucio executed a simple deed of donation of


P50 million on time deposit with a bank in
favor of A, B, C, D, and E,

Never Let The Odds Keep You From Pursuing What You Know In Your Heart You Were Meant To Do.Leroy Satchel Paige

Page 151 of 180

Civil Law Q&As (2007-2013)

hectorchristopher@yahoo.com

conformity.

without indicating the share of each donee.


All the donees accepted the donation in
writing. A, one of the donees, died. Will B,
C, D, and E get As share in the money?
(A) Yes,

accretion

will

automatically

apply to the joint-donees in equal


shares.
(B) Yes, since the donors intention is to
give the whole of P50 million to the
jointdonees in equal shares.
(C) No, A"s share will revert to the donor
because accretion applies only if the
joint-donees are spouses.
(D) No, As share goes to his heirs
since

the

donation

did

not

provide for reversion to donor.

(10)

Raul, Ester, and Rufus inherited a 10-

hectare land from their father. Before the


land could be partitioned, however, Raul
sold his hereditary right to Raffy, a stranger
to the family, for P5 million. Do Ester and
Rufus have a remedy for keeping the land
within their family?
(A) Yes, they may be subrogated to
Raffys right by reimbursing to
him

within

the

required

time

what he paid Raul.


(B) Yes, they may be subrogated to
Raffys right provided they buy him
out before he registers the sale.
(C) No, they can be subrogated to
Raffys

right

only

with

his

dbaratbateladot@gmail.com

(A) Its cause comes from the guilt of


(D) No, since there was no impediment

a spouse in a legal separation

to Raul selling his inheritance to a

case, the innocent-spouse having

stranger.

died.
(B) The truth of its cause is denied and

(11)

When one exercises a right recognized

not sufficiently proved by evidence.

by law, knowing that he thereby causes an

(C) Its cause is not authorized by the

injustice to another, the latter is entitled to

law.

recover damages. This is known as the

(D) Its cause is not specified.

principle of
(A) res ipsa loquitur.

(13)

Manuel came to Manila and married

(B) damnum absque injuria.

Marianne. Unknown to Marianne, Manuel

(C) vicarious liability.

had been previously convicted in Palawan of

(D) abuse of rights.

theft and served time for it. After Marianne


learned of his previous conviction, she

(12)

Which of the following is NOT a basis

for rendering a disinheritance defective or

stopped living with him. Can Marianne seek


the annulment of the

imperfect?
Never Let The Odds Keep You From Pursuing What You Know In Your Heart You Were Meant To Do.Leroy Satchel Paige

Page 152 of 180

Civil Law Q&As (2007-2013)

hectorchristopher@yahoo.com

marriage based on Manuels nondisclosure


of his previous crime?
(A) No, since the assumption is that
marriage forgives all past wrongs.
(B) Yes, since the non-disclosure of
that crime is the equivalent of
fraud,

which

is

ground

for

annulment.
(C) No, in case of doubt, the law must
be

construed

to

preserve

the

institution of marriage.
(D) No, since Manuel already served the
penalty for his crime.

(14)

Arthur and Helen, both Filipinos, got

married and had 2 children. Arthur later


worked in Rome where he acquired Italian
citizenship. He got a divorce from Helen in
Rome but, on returning to the Philippines,
he realized his mistake, asked forgiveness of
his wife, and resumed living with her. They
had 2 more children. What is the status of
their 4 children?
(A) The

children

dbaratbateladot@gmail.com

the divorce are all legitimate

born

before

the

divorce are legitimate but those


born after it are not since Arthur
got the divorce when he had
ceased to be a Filipino.
(B) The divorce rendered illegitimate the
children born before it since the
marriage that begot them had been
nullified.
(C) The children born before and after

(16)
since Philippine law does not

industrial

recognize divorce.

contracts entered into in its name and for

(D) All the children are legitimate since


they were born of the same father

partners

for

partnership

its account, when all partnership assets


have been exhausted is

and mother.
(15)

The liability of the partners, including

(A) Pro-rata.
(B) Joint.

Who can make a donation?

(C) Solidary.

(A) All persons who can enter into

(D) Voluntary.

contracts and dispose of their


property.
(B) All persons who are of legal age and
suffer from no civil interdiction.
(C) All persons who can make a last will
and testament.
(D) All persons, whether natural or
artificial, who own property.

(17)

When can a missing person who left

someone to administer his property be


declared an absentee by the court? When
he has been missing for
(A) 2 years from the receipt of the last
news about him.
(B) 7 years from the receipt of the last
news about him.

Never Let The Odds Keep You From Pursuing What You Know In Your Heart You Were Meant To Do.Leroy Satchel Paige

Page 153 of 180

Civil Law Q&As (2007-2013)

hectorchristopher@yahoo.com

(C) 10 years from the receipt of the last


news about him.
(D) 5 years from the receipt of the
last news about him.

(18)

Which of the following claims against

the debtor enjoys preference over the others


with respect to his specific immovable
property and real rights?
(A) Unpaid price of real property sold,
upon the immovable property.
(B) Mortgage credits recorded in the
registry

of

property,

upon

the

mortgaged real estate.


(C) Taxes

due,

upon

the

land

or

building.
(D) Expenses for the preservation and
improvement of property, when the
law

authorizes

reimbursement,

upon the preserved or improved


immovable.

(19)

When bilateral contracts are vitiated

with vices of consent, they are rendered


(A) rescissible.
(B) void.
(C) unenforceable.
(D) voidable.

(20)

dbaratbateladot@gmail.com

deed of absolute sale on behalf of his

An agent, authorized by a special

power of attorney to sell a land belonging to


the principal succeeded in selling the same
to a buyer according to the instructions
given the agent. The agent executed the

claimed that it was exempt from execution,


principal two days after the principal died,

being a family home. Is this claim correct?


(A) Yes, because while Bs parents own

an event that neither the agent nor the

the land, they agreed to have their

buyer knew at the time of the sale. What is

daughter build her family home on

the standing of the sale?

it.

(A) Voidable.

(B) No, because there is no judicial

(B) Valid.

declaration that it is a family home.

(C) Void.

(C) No, since the land does not belong

(D) Unenforceable.

to A and B, it cannot qualify as a


(21)

family home.

Spouses A and B leased a piece of land

belonging to B's parents for 25 years. The

(D) Yes, because the A and Bs family

spouses built their house on it worth

actually lives in that house.

P300,000.00. Subsequently, in a case that


C filed against A and B, the court found the

(22)

Solomon sold his coconut plantation to

latter liable to C for P200,000.00. When the

Aragon, Inc. for P100 million, payable in

sheriff was attaching their house for the

installments of P10 million per month with

satisfaction of the judgment, A and B

6% interest per annum. Solomon married

Never Let The Odds Keep You From Pursuing What You Know In Your Heart You Were Meant To Do.Leroy Satchel Paige

Page 154 of 180

Civil Law Q&As (2007-2013)

hectorchristopher@yahoo.com

Lorna after 5 months and they chose

(24)

dbaratbateladot@gmail.com

When A and B married, they chose

conjugal partnership of gains to govern

conjugal partnership of gains to govern

their

their property relations. After 3 years, B

property

relations.

When

they

married, Aragon had an unpaid balance of

succeeded in getting her marriage to A

P50 million plus interest in Solomons favor.

annulled

To whom will Aragons monthly payments

psychological incapacity. What liquidation

go after the marriage?

procedure will they follow in disposing of

(A) The

principal

conjugal

shall

go

partnership

to

but

the
the

since

the

they

are

of

the

latters

(A) They will follow the rule governing


liquidation

of

conjugal

partnership of gains where the party

(B) Both principal and interests shall go


Solomon

ground

their assets?

interests to Solomon.

to

on

his

who acted in bad faith forfeits his

exclusive properties.

share in the net profits.

(C) Both principal and interests shall go

(B) Since

the

marriage

has

been

to the conjugal partnership since

declared

these

liquidation of absolute community of

become

due

after

the

marriage.

void,

the

rule

for

property shall be followed.

(D) The principal shall go to Solomon

(C) The liquidation of a co-ownership

but the interests to the conjugal

applies

partnership.

brought their property relation

since

the

annulment

under the chapter on property


(23)

X and Y, although not suffering from

regimes without marriage.

any impediment, cohabited as husband and


wife

without

the

benefit

of

Following the birth of their child, the couple


got married. A year after, however, the court
annulled the marriage and issued a decree
of annulment. What is the present status of
the child?
(A) Legitimated.
(B) Illegitimate.
(C) Natural child.
(D) Legitimate.

(D) The

marriage.

law

on

liquidation

of

partnerships applies.
(25)

X and Y agreed verbally before their

marriage

(a)

on

the

paternity

of

the

illegitimate child of Y and (b) on the


economic regime that will govern X and Ys
property relations. Is the verbal agreement
valid?
(A) No, because a marriage settlement
to be valid should be in writing.

Never Let The Odds Keep You From Pursuing What You Know In Your Heart You Were Meant To Do.Leroy Satchel Paige

Page 155 of 180

Civil Law Q&As (2007-2013)

hectorchristopher@yahoo.com

(B) Yes, since ante-nuptial agreements

(B) Illegitimate, because by the color of

need not be in writing.

its skin, the child could not possibly


be that of Fidel.

(C) No, because a marriage settlement


cannot include an agreement on the

(C) Legitimate, because the child was

paternity of an illegitimate child.

(26)

dbaratbateladot@gmail.com

born within a valid marriage.

(D) Yes, since even if it is not a valid

(D) Legitimate, because Fidel agreed to

marriage settlement, it is a valid

treat the child as his own after

verbal contract.

Gloria told him who the father was.

Spouses X and Y have a minor

(28)

The husbands acts of forcibly ejecting

daughter, Z, who needs support for her

his wife without just cause from the

education.

conjugal dwelling and refusing to take her

Both

and

Y,

who

are

financially distressed, could not give the

back constitutes

needed support to Z. As it happens, Zs

(A) desertion.

other relatives are financially capable of

(B) recrimination.

giving that support. From whom may Z first

(C) constructive abandonment.

rightfully demand support? From her

(D) de facto separation.

(A) grandfather.
(29)

(B) brother.

In his will, the testator designated X as

(C) uncle.

a legatee to receive P2 million for the

(D) first cousin.

purpose of buying an ambulance that the


residents of his Barangay can use. What

(27)

Fidel, a Filipino with fair complexion,

kind of institution is this?

married Gloria. Before the marriage, Gloria

(A) a fideicomissary institution.

confessed to Fidel that she was two-month

(B) a modal institution.

pregnant with the child of a black African

(C) a conditional institution.

who had left the country for good. When

(D) a collective institution.

the child was born, Fidel could not accept it


being too black in complexion. What is the

X insured himself for P5 million,

designating

status of the child?


(A) Illegitimate,

(30)

because

Gloria

beneficiary.

Y,

his
The

wife,

as

his

designation

sole
was

confessed that the child is not

irrevocable. A few years later, X had their

Fidels.

marriage annulled in court on the ground


that Y had an existing prior marriage. X

Never Let The Odds Keep You From Pursuing What You Know In Your Heart You Were Meant To Do.Leroy Satchel Paige

Page 156 of 180

Civil Law Q&As (2007-2013)

hectorchristopher@yahoo.com

subsequently died, Is Y entitled to the


insurance benefits?
(A) Yes, since the insurance was not
dependent on the marriage.
(B) Yes,

since

her

designation

as

beneficiary was irrevocable.


(C) No,

Xs

designation

of

is

revoked by operation of law upon


the annulment of their marriage
based on Ys fault.
(D) Yes,

since

revocation,

Xs

without

judicial

designation

of

remains valid and binding.


(31)

May a spouse freely donate communal

or conjugal property without the consent of


the other?
(A) Absolutely not, since the spouses
co-own such property.
(B) Yes, for properties that the family
may spare, regardless of value.
(C) Yes,

provided

the

donation

is

moderate and intended for charity


or family rejoicing.
(D) Yes, in a donation mortis causa that
the donor may still revoke in his
lifetime.

(32)

The decedent died intestate leaving an

estate of P10 million. He left the following


heirs: a) Marlon, a legitimate child and b)
Cecilia, the legal spouse. Divide the estate.
(A) Marlon gets 1/4 and Cecilia gets
3/4.

dbaratbateladot@gmail.com

(B) Marlon gets 2/3 and Cecilia 1/3.

(C) Marlon gets 1/2 and Cecilia gets


1/2.
(D) Marlon gets 3/4 and Cecilia 1/4.

(33)

Contracts take effect only between the

parties or their assigns and heirs, except


where the rights and obligations arising
from the contract are not transmissible by
their nature, by stipulation, or by provision
of law. In the latter case, the assigns or the
heirs are not bound by the contracts. This
is known as the principle of
(A) Relativity of contracts.
(B) Freedom to stipulate.
(C) Mutuality of contracts.
(D) Obligatory force of contracts.

(34)

A buyer ordered 5,000 apples from the

seller at P20 per apple. The seller delivered


6,000 apples. What are the rights and
obligations of the buyer?
(A) He can accept all 6,000 apples
and pay the seller at P20 per
apple.
(B) He can accept all 6,000 apples and
pay a lesser price for the 1,000
excess apples.
(C) He

can

keep

the

6,000

apples

without paying for the 1,000 excess


since

the

seller

delivered

them

anyway.
(D) He can cancel the whole transaction
since the seller violated the terms of
their agreement.

Never Let The Odds Keep You From Pursuing What You Know In Your Heart You Were Meant To Do.Leroy Satchel Paige

Page 157 of 180

Civil Law Q&As (2007-2013)

(35)

hectorchristopher@yahoo.com

Lino entered into a contract to sell with

Ramon, undertaking to convey to the latter


one of the five lots he owns, without
specifying which lot it was, for the price of
P1 million. Later, the parties could not
agree which of five lots he owned Lino
undertook to sell to Ramon. What is the
standing of the contract?
(A) Unenforceable.
(B) Voidable.
(C) Rescissible.
(D) Void.

(36)

Knowing that the car had a hidden

crack in the engine, X sold it to Y without


informing the latter about it. In any event,
the deed of sale expressly stipulated that X
was not liable for hidden defects. Does Y
have the right to demand from

X a

reimbursement of what he spent to repair


the engine plus damages?
(A) Yes. X is liable whether or not he
was aware of the hidden defect.
(B) Yes, since the defect was not
hidden; X knew of it but he acted
in bad faith in not disclosing the
fact to Y.
(C) No, because Y is in estoppel, having
changed

engine

without

prior

demand.
(D) No, because Y waived the warranty
against hidden defects.
(37)

dbaratbateladot@gmail.com

cans known as "Sards." Mylene bought a

Acme Cannery produced sardines in

can of Sards from a store, ate it, and

and Mylene has not overcome such


suffered

from

poisoning

caused

by

noxious substance found in the sardines.


Mylene filed a case for damages against
Acme. Which of the following defenses will
hold?
(A) The expiry date of the "Sards" was
clearly printed on its can, still the
store sold and Mylene bought it.
(B) Mylene

must

have

detected

the

noxious substance in the sardines


by smell, yet she still ate it.
(C) Acme

had

no

transaction

presumption.

a
(38)

Fernando executed a will, prohibiting

his wife Marina from remarrying after his


death, at the pain of the legacy of P100
Million in her favor becoming a nullity. But
a year after Fernandos death, Marina was
so overwhelmed with love that she married
another man. Is she entitled to the legacy,
the amount of which is well within the
capacity of the disposable free portion of
Fernandos estate?

with

(A) Yes, since the prohibition against

Mylene; she bought the "Sards" from

remarrying is absolute, it is deemed

a store, not directly from Acme.

not written.

(D) Acme enjoys the presumption of


safeness of its canning procedure

(B) Yes,

because

the

prohibition

is

inhuman and oppressive and

Never Let The Odds Keep You From Pursuing What You Know In Your Heart You Were Meant To Do.Leroy Satchel Paige

Page 158 of 180

Civil Law Q&As (2007-2013)

hectorchristopher@yahoo.com

violates Marinas rights as a free

(A) It is an ordinary donation since it

woman.
(C) No,

was not given to the bride or

because

the

nullity

of

the

groom.

prohibition also nullifies the legacy.


(D) No,

dbaratbateladot@gmail.com

since

such

prohibition

(B) It is donation propter nuptias since

is

it was given with the marriage in

authorized by law and is not

mind.

repressive; she could remarry but

(C) It is an indirect donation propter

must give up the money.

nuptias

since

the

bride

would

eventually inherit the property from


(39)

her parents.

X, the owner, constituted a 10-year

(D) It is a remunatory donation.

usufruct on his land as well as on the


building standing on it in Ys favor. After
flood totally destroyed the building 5 years

(41)

X and Y, both Filipinos, were married

later, X told Y that an act of God terminated

and resided in Spain although they intend

the usufruct and that he should vacate the

to return to the Philippines at some future

land. Is X, the owner of the land, correct?

time. They have not executed any marriage

(A) No, since the building was destroyed

settlements.

law

governs

their

property relations?

through no fault of Y.

(A) They may choose between Spanish

(B) No, since Y still has the right to

law and Philippine law.

use the land and the materials

(B) Philippine law since they are both

left on it.

Filipinos.

(C) Yes, since Y cannot use the land

(C) No regime of property relations will

without the building.

apply to them.

(D) Yes, since the destruction of the


building

What

without

the

Xs

(D) Spanish law since they live in Spain.

fault

terminated the usufruct.


(42)

Birth determines personality. Death

In gratitude, the grooms parents made

extinguishes it. Under what circumstances

a donation of a property in writing to the

may the personality of a deceased person

brides

continue to exist?

(40)

parents

shortly

childrens

wedding.

The

accepted.

What

the

donation?

is

before

their

donation

was

nature

of

the

(A) In

case

of

re-appearance

of

missing person presumed dead.

Never Let The Odds Keep You From Pursuing What You Know In Your Heart You Were Meant To Do.Leroy Satchel Paige

Page 159 of 180

Civil Law Q&As (2007-2013)

(B) In

protecting

hectorchristopher@yahoo.com

the

works

of

deceased under intellectual property


laws.
(C) In case of declaration of presumptive
death of a missing spouse.
(D) In the settlement of the estate of
a deceased person.

(43)

Six tenants sued X, the landowner, for

willfully denying them water for their farms,


which water happened to flow from land
under Xs control, his intention being to
force them to leave his properties. Is X
liable for his act and why?
(A) No, because the tenants must be
content with waiting for rainfall for
their farms.
(B) No, since X owns both the land and
the water.
(C) Yes, because the tenants farms have
the natural right of access to water
wherever it is located.
(D) Yes,

since

willfully

caused

injury to his tenants contrary to


morals, good customs or public
policy.

(44)

Illegitimate

dbaratbateladot@gmail.com

recognized by their father.

brothers

and

sisters,

whether of full or half-blood, are bound to


support each other, EXCEPT when
(A) the brother or sister who needs
support lives in another place.
(B) such brothers and sisters are not

"attractive nuisance". Is Virgilio liable for


(C) the brother or sister in need stops
schooling without valid reason.
(D) the need for support of a brother
or sister, already of age, is due to
the latter's fault.

the death of MB?


(A) No, the child was 7 years old and
knew the dangers that the pool
offered.
(B) Yes, being an attractive nuisance,
Virgilio had the duty to prevent
children from coming near it.

(45)

Virgilio owned a bare and simple

swimming pool in his garden. MB, a 7-year


old child, surreptitiously entered the garden
and merrily romped around the ledges of

(C) No, since the pool was bare and


had

no

enticing

or

alluring

gadgets, floats, or devices in it

the pool. He accidentally tripped, fell into

that would attract a 7-year old

the pool, and drowned. MBs parents sued

child.

Virgilio for damages arising from their


childs death, premised on the principle of

(D) Yes, since Virgilio did not cover the


swimming pool while not in use to
prevent children from falling into it.

Never Let The Odds Keep You From Pursuing What You Know In Your Heart You Were Meant To Do.Leroy Satchel Paige

Page 160 of 180

Civil Law Q&As (2007-2013)

(59)

hectorchristopher@yahoo.com

dbaratbateladot@gmail.com

The term of a 5-year lease contract

(D) Yes, the donation is not deemed

between X the lessor and Y the lessee,

made until the suspensive condition

where rents were paid from month to

has been fulfilled.

month, came to an end. Still, Y continued


using the property with Xs consent. In

(48)

Illegitimate

children,

those

not

such a case, it is understood that they

recognized by their biological fathers, shall

impliedly renewed the lease

use the surname of their


(A) biological

(A) from month to month under the

the

same

terms

(C) mother.

(C) under the same terms except the

(D) biological father unless he judicially

rent which they or the court must

opposes it.

fix.

control law.

to the municipality on the condition that it


will build a public school on such lot within 2
years from its acceptance of the donation. The
municipality properly accepted the donation
but did not yet build the public school after 2

Asiong borrowed P1 million from a

bank, secured by a mortgage on his land.

(A) Yes, since the donation is subject


to a resolutory condition which

(B) No, but Rex is entitled to recover the


land

from

the

municipality.
(C) No, the transfer of ownership has
been completed.

the payment, can Boyong compel the bank


to subrogate him in its right as mortgagee
of Asiong's land?
(A) No, but the bank can foreclose and
pay Boyong back.

Asiongs

loan

without

his

approval.
(C) Yes, since a change of creditor took

was not fulfilled.

the

the whole loan. Since Asiong benefited from

(B) No, since Boyong paid for

years. Can Rex revoke the donation?

of

(49)

Without his consent, his friend Boyong paid

(60)Rex, a philanthropist, donated a valuable lot

value

no

mothers discretion.

conditions as before.

by 10% pursuant to the rental

to

(B) mother or biological father, at the

and

(D) for only a year, with the rent raised

subject

condition.

same conditions as to the rest.


(B) under

father

place by novation with the banks


consent.
(D) Yes, since it is but right that Boyong
be able to get back his money and, if
not, to foreclose the mortgage in the
manner of the bank.

Never Let The Odds Keep You From Pursuing What You Know In Your Heart You Were Meant To Do.Leroy Satchel Paige

Page 161 of 180

Civil Law Q&As (2007-2013)

(50)

hectorchristopher@yahoo.com

Congress passed a law imposing taxes

on income earned out of a particular


activity that was not previously taxed. The
law, however, taxed incomes already earned
within the fiscal year when the law took
effect. Is the law valid?
(A) No, because laws are intended to be
prospective, not retroactive.
(B) No, the law is arbitrary in that it
taxes income that has already been
spent.
(C) Yes, since tax laws are the lifeblood
of the nation.
(D) Yes, tax laws are an exception;
they

can

be

given

retroactive

effect.

(51)

Rudolf

Rodrigo

borrowed

and

Fernando

P1

million

who

from

acted

as

solidary creditors. When the loan matured,


Rodrigo wrote a letter to Rudolf, demanding
payment of the loan directly to him. Before
Rudolf could comply, Fernando went to see
him personally to collect and he paid him.
Did Rudolf make a valid payment?
(A) No, since Rudolf should have split
the payment between Rodrigo and
Fernando.
(B) No,

since

dbaratbateladot@gmail.com

whole obligation.

Rodrigo,

the

other

solidary creditor, already made a


prior demand for payment from
Rudolf.
(C) Yes, since the payment covers the

its regimes shall now be governed by


the regime of absolute community of

(D) Yes, since Fernando was a solidary


creditor,

payment

to

property.

him

(D) They are superseded by the Family

extinguished the obligation.

Code which has retroactive effect.


(52)

What happens to the property regimes

(53)

The testator executed a will following

that were subsisting under the New Civil

the formalities required by the law on

Code when the Family Code took effect?

succession without designating any heir.

(A) The original property regimes are


immutable and remain effective.
(B) Those

enjoying

specific

regimes

under the New Civil Code may adopt


the regime of absolute community of
property under the Family Code.
(C) Those that married under the New
Civil Code but did not choose any of

The only testamentary disposition in the


will is the recognition of the testator's
illegitimate child with a popular actress. Is
the will valid?
(A) Yes,

since

in

recognizing

his

illegitimate child, the testator has


made him his heir.
(B) No, because the non-designation of
heirs defeats the purpose of a will.

Never Let The Odds Keep You From Pursuing What You Know In Your Heart You Were Meant To Do.Leroy Satchel Paige

Page 162 of 180

Civil Law Q&As (2007-2013)

hectorchristopher@yahoo.com

(C) No, the will comes to life only when

(C) Voidable, because the Judge acted

the proper heirs are instituted.


(D) Yes,

the

recognition

illegitimate

heir

is

an

of

dbaratbateladot@gmail.com

beyond his territorial jurisdiction


and is administratively liable for the

an

same.

ample

(D) Void, because the Judge did not

reason for a will.

solemnize the marriage within the


(54)

premises of his court.

A left B, his wife, in the Philippines to

work in Egypt but died in that country after


a years continuous stay. Two months after

(56)

X and Y, Filipinos, got married in Los

As death, B gave birth to a child, claiming

Angeles, USA, using a marriage license

it is As child. Who can assail the legitimacy

issued by the Philippine consul in Los

of the child?

Angeles, acting as Civil Registrar. X and Y

(A) As other heirs apart from B.


(B) The State which has interest in the
welfare of overseas contract workers.
(C) Any one who is outraged by Bs
claim.

because their mothers, who were sisters,


were

separated

when

they

were

quite

young. Since X did not want to continue


with the relation when he heard of it, he left
Y, came to the Philippines and married Z.

(D) No one since A died.

(55)

did not know that they were first cousins

Can X be held liable for bigamy?

QR and TS who had a marriage license

requested a newly appointed Judge in


Manila to marry them on the beach of

(A) No since Xs marriage to Y is void ab


initio or did not exist.
(B) No since X acted in good faith,

maintained

conscious that public policy did not

Boracay as his residence, he agreed. The

approve of marriage between first

Boracay.

Since

the

Judge

sponsors were all public officials. What is


the status of the marriage.
(A) Valid, since the improper venue is

cousins.
(C) Yes since he married Z without
first

securing

merely an irregularity; all the

declaration

elements of a valid marriage are

marriage to Y.

present.
(B) Void, because the couple did not get
local permit for a beach wedding.

of

a
nullity

judicial
of

his

(D) Yes since his first marriage to Y in


Los Angeles is valid.
(57) Allan
through

bought

Billys

property

Carlos, an agent empowered with a special

power of attorney (SPA) to sell the same.

Never Let The Odds Keep You From Pursuing What You Know In Your Heart You Were Meant To Do.Leroy Satchel Paige

Page 163 of 180

Civil Law Q&As (2007-2013)

hectorchristopher@yahoo.com

deteriorate.

When Allan was ready to pay as scheduled,


Billy called, directing Allan to pay directly to
him. On learning of this, Carlos, Billy's
agent, told Allan to pay through him as his
SPA

provided

and

to

protect

his

commission. Faced with two claimants,


Allan consigned the payment in court. Billy
protested, contending that the consignation
is ineffective since no tender of payment
was made to him. Is he correct?
(A) No, since consignation without
tender of payment is allowed in
the face of the conflicting claims
on the plaintiff.
(B) Yes, as owner of the property sold,
Billy can demand payment directly
to himself.
(C) Yes,

since

Allan

made

no

announcement of the tender.


(D) Yes, a tender of payment is required
for a valid consignation.

(58)

X sold Y 100 sacks of rice that Y was to

pick up from Xs rice mill on a particular


date. Y did not, however, appear on the
agreed date to take delivery of the rice.
After one week, X automatically rescinded
the sale without notarial notice to Y. Is the
rescission valid?
(A) Yes,

automatic

allowed
character

rescission

since,
of

consumables,

is

having

the

movables

and

rice

can

easily

dbaratbateladot@gmail.com

precedent. Is the dismissal proper?


(B) No,

the

buyer is

entitled to

(A) No, efforts at a compromise will only

deepen the wifes anguish.

customary 30-day extension of his

(B) No, since legal separation like

obligation to take delivery of the

validity of marriage is not subject

goods.

to

(C) No, since there was no express


agreement

regarding

agreement

for

purposes of filing.

automatic

rescission.

(C) Yes, to avoid a family feud that is


hurtful to everyone.

(D) No, the seller should first determine


that Y was not justified in failing to

(D) Yes, since the dispute could have

appear.
(59)

compromise

been

settled

with

the

parties

agreeing to legal separation.

The wife filed a case of legal separation

against her husband on the ground of


sexual infidelity

(60)

An Australian living in the Philippines

without previously exerting earnest efforts

acquired shares of stock worth P10 million

to come to a compromise with him. The

in food manufacturing companies. He died

judge dismissed the case for having been

in Manila, leaving a legal wife and a child in

filed without complying with a condition

Australia and a live-in partner with whom

Never Let The Odds Keep You From Pursuing What You Know In Your Heart You Were Meant To Do.Leroy Satchel Paige

Page 164 of 180

Civil Law Q&As (2007-2013)

hectorchristopher@yahoo.com

he had two children in Manila. He also left


a will, done according to Philippine laws,
leaving all his properties to his live-in
partner and their children. What law will
govern the validity of the disposition in the
will?
(A) Australia law since his legal wife and
legitimate child are Australians and
domiciled in Australia.
(B) Australian law since the intrinsic
validity of the provisions of a will
is governed by the decedents
national law.
(C) Philippine law since the decedent
died in Manila and he executed his
will according to such law.
(D) Philippine law since the decedents
properties are in the
Philippines.

(61)X bought a land from Y, paying him cash.


Since they were friends, they did not execute
any document of sale. After 7 years, the heirs
of X asked Y to execute a deed of absolute sale
to formalize the verbal sale to their father.
Unwilling to do so, Xs heirs filed an action for
specific performance against Y. Will their
action prosper?

(A) No, after more than 6 years, the


action

to

enforce

dbaratbateladot@gmail.com

(C) Yes, since X bought the land and

the

verbal

agreement has already elapsed.


(B) No, since the sale cannot under the
Statute of Frauds be enforced.

paid Y for it.

over the intestate proceedings to


a new administrator whom it will

(D) Yes, after full payment, the action

appoint.

became imprescriptible.

(C) Cicero
(62)

automatically

administrator

A court declared Ricardo, an old

of

becomes

Ricardos

estate

until judicially relieved.

bachelor, an absentee and appointed Cicero


administrator of his property. After a year, it

(D) Ciceros

alienations

of

Ricardo's

property will be set aside.

was discovered that Ricardo had died


abroad. What is the effect of the fact of his
death

on

the

administration

of

his

property?

(63)

Baldo, a rejected suitor, intimidated

Judy into marrying him. While she wanted

(A) With Ricardo no longer an absentee

to question the validity of their marriage

but a deceased person, Cicero will

two years after the intimidation ceased,

cease to be administrator of his

Judy decided in the meantime to freely

properties.

cohabit with Baldo. After more than 5 years

(B) The administration shall be given


by the court having jurisdiction

following their wedding, Judy wants to file a


case for annulment of marriage against

Never Let The Odds Keep You From Pursuing What You Know In Your Heart You Were Meant To Do.Leroy Satchel Paige

Page 165 of 180

Civil Law Q&As (2007-2013)

hectorchristopher@yahoo.com

Baldo on ground of lack of consent. Will her


action prosper?
(A) Yes, the action for annulment is
imprescriptible.
(B) No, since the marriage was merely
voidable and Judy ratified it by
freely cohabiting with Baldo after
the force and intimidation had
ceased.
(C) No, since the action prescribed 5
years

from

the

date

of

the

marriage

was

celebration of the marriage.


(D) Yes,

because

the

celebrated without Judy's consent


freely given.

(64)

Is the wife who leaves her husband

without just cause entitled to support?


(A) No, because the wife must always be
submissive and respectful to the
husband.
(B) Yes. The marriage not having been
dissolved, the husband continues to
have an obligation to support his
wife.
(C) No,

because

in

leaving

the

conjugal home without just cause,


she forfeits her right to support.
(D) Yes,

since

support

is

the
not

right

to

subject

receive
to

any

condition.

(65)

dbaratbateladot@gmail.com

where the decedent is legitimate, who is the

In the order of intestate succession

last intestate heirs or heir who will inherit if

liability of Roy and Carlos joint or solidary?


(A) Neither solidary nor joint since they

all heirs in the higher level are disqualified

cannot

or unable to inherit?

waive

the

defense

of

(A) Nephews and nieces.

fortuitous event to which they are

(B) Brothers and sisters.

entitled.
(B) Solidary or joint upon the discretion

(C) State.

of Sam.

(D) Other collateral relatives up to the

(C) Solidary since Roy and Carlos failed

5th degree of consanguinity.

to perform their obligation to deliver


(66)

the motor boat.

Roy and Carlos both undertook a

(D) Joint

contract to deliver to Sam in Manila a boat

since

the

conversion

of

docked in Subic. Before they could deliver

their liability to one of indemnity

it, however, the boat sank in a storm. The

for damages made it joint.

contract provides that fortuitous event shall


not exempt Roy and Carlos from their

(67)

Joanne married James, a person with

obligation. Owing to the loss of the motor

no known relatives. Through James' hard

boat, such obligation is deemed converted

work, he and his wife Joane prospered.

into one of indemnity for damages. Is the

When

James died, his estate alone

Never Let The Odds Keep You From Pursuing What You Know In Your Heart You Were Meant To Do.Leroy Satchel Paige

Page 166 of 180

Civil Law Q&As (2007-2013)

hectorchristopher@yahoo.com

amounted to P100 million. If, in his will,

(70)

dbaratbateladot@gmail.com

Ric and Josie, Filipinos, have been

James designates Joanne as his only heir,

sweethearts for 5 years. While working in a

what will be the free portion of his estate.

European country where the execution of

(A) Joanne gets all; estate has no free


portion left.

joint wills are allowed, the two of them


executed a joint holographic will where they

(B) Joanne gets 1/2; the other half is


free portion.

named each other as sole heir of the other


in case either of them dies. Unfortunately,

(C) Joanne gets 1/3; the remaining 2/3

Ric died a year later. Can Josie have the


joint will successfully probated in the

is free portion.
(D) Joanne gets 1/4; the remaining 3/4

Philippines?

is free portion.

(A) Yes, in the highest interest of comity


of nations and to honor the wishes

(68)

of the deceased.

A warranty inherent in a contract of

sale, whether or not mentioned in it, is

(B) No, since Philippine law prohibits

known as the

the execution of joint wills and

(A) warranty on quality.

such law is binding on Ric and

(B) warranty against hidden defects.

Josie even abroad.

(C) warranty against eviction.

(C) Yes, since they executed their joint

(D) warranty in merchantability.

(69)

will out of mutual love and care,


values that the generally accepted

The doctrine of stare decisis prescribes

adherence

to

precedents

in

order

principles

to

where it was executed, applying the

(A) When adherence to it would result

principle of "lex loci celebrationis."

in the Governments loss of its case.

foreign national.
(C) When necessary to promote the
passage of a new law.
(D) When the precedent has ceased to
be beneficial and useful.

law

(D) Yes, since it is valid in the country

doctrine can be abandoned

would cause great prejudice to a

international

accepts.

promote the stability of the law. But the

(B) When the application of the doctrine

of

(71)

ML inherited from his father P5 million

in legitime but he waived it in a public


instrument in favor of his sister QY who
accepted the waiver in writing. But as it
happened, ML borrowed P6 million from PF
before the waiver. PF objected to the waiver
and filed an action for its rescission on the

ground that he had the right to MLs P5


Never Let The Odds Keep You From Pursuing What You Know In Your Heart You Were Meant To Do.Leroy Satchel Paige

Page 167 of 180

Civil Law Q&As (2007-2013)

hectorchristopher@yahoo.com

million legitime as partial settlement of

(B) Yes,

dbaratbateladot@gmail.com

insofar

as

what ML owed him since ML has proved to

acknowledged

be insolvent. Does PF, as creditor, have the

illegitimate child.

right to rescind the waiver?

his

take place.

his sister QY amounts to a donation

(D) Yes, if they acquired properties while

and she already accepted it.


because

as

(C) None, since the marriage did not

(A) No, because the waiver in favor of

(B) Yes,

Mary

Arnold

the

living together as husband and wife.

waiver

is

(73)

Joseph, a 17-year old Filipino, married

prejudicial to the interest of a

Jenny, a 21-year old American in Illinois,

third person whose interest is

USA, where the marriage was valid. Their

recognized by law.

parents gave full consent to the marriage of

(C) No, PF must wait for ML to become

their children. After three years, Joseph

solvent and, thereafter, sue him for

filed a petition in the USA to promptly

the unpaid loan.

divorce Jenny and this was granted. When

(D) Yes, because a legitime cannot be

Joseph turned 25 years, he returned to the

waived in favor of a specific heir; it

Philippines and married Leonora. What is

must be divided among all the other

the status of this second marriage?


(A) Void, because he did not cause

heirs.

the
(72)

While engaged to be married, Arnold

and

Josephine

agreed

in

public

instrument to adopt out the economic


regime of absolute community of property.
Arnold

acknowledged

in

the

same

instrument that Josephines daughter


Mary,

is

his

illegitimate

child.

But

take place. Does the marriage settlement


have any significance?
since

the

instrument

containing the marriage settlement


is essentially void for containing an
unrelated matter.

issuance

of

declaration of the nullity of his


first marriage to Jenny before
marrying Leonora.
(B) Valid, because Joseph's marriage to
Jenny is void, he being only 17
years of age when he married her.

Josephine died before the marriage could

(A) None,

judicial

(C) Valid,

because

his

marriage

to

Leonora has all the elements of a


valid marriage.
(D) Void,

because

Joseph

is

still

considered married to Jenny since


the Philippines does not recognize
divorce.

Never Let The Odds Keep You From Pursuing What You Know In Your Heart You Were Meant To Do.Leroy Satchel Paige

Page 168 of 180

Civil Law Q&As (2007-2013)

(74)

hectorchristopher@yahoo.com

T died intestate, leaving an estate of

(76)

X,

dbaratbateladot@gmail.com

who was

abroad,

phoned

his

P9,000,000. He left as heirs three legitimate

brother, Y, authorizing him to sell Xs parcel

children, namely, A, B, and C. A has two

of land in Pasay. X sent the title to Y by

children, D and E. Before he died, A

courier service. Acting for his brother, Y

irrevocably repudiated his inheritance from

executed a notarized deed of absolute sale

T in a public instrument filed with the

of the land to Z after receiving payment.

court. How much, if any, will D and E, as

What is the status of the sale?

As children, get from Ts estate?

(A) Valid, since a notarized deed of

(A) Each of D and E will get P1,500,000

absolute

covered

the

by right of representation since their

transaction and full payment was

father repudiated his inheritance.

made.

(B) Each of D and E will get P2,225,000

(75)

sale

(B) Void,

since

should

have

because they will inherit from the

authorized agent Y in writing to

estate equally with B and C.

sell the land.

(C) D and E will get none because of

(C) Valid, since Y was truly his brother

the repudiation; "B" and "C" will

Xs agent and entrusted with the

get As share by right of accretion.

title needed to effect the sale.

(D) Each of D and E will get P2,000,000

(D) Valid, since the buyer could file an

because the law gives them some

action to compel X to execute a deed

advantage due to the demise of "A".

of sale.

No decree of legal separation can be

(77)

In a true pacto de retro sale, the title

and ownership of the property sold are

issued
(A) unless

the

childrens

welfare

is

subject only to the resolutory condition of

attended to first.
(B) without

prior

efforts

at

reconciliation shown to be futile.


(C) unless

the

court

immediately vested in the vendee a retro

first

repurchase by the vendor a retro within the


stipulated period. This is known as
(A) equitable mortgage.

directs

(B) conventional redemption.

mediation of the parties.


(D) without

prior

(C) legal redemption.

investigation

(D) equity of redemption.

conducted by a public prosecutor.


(78)

A natural obligation under the New

Civil Code of the Philippines is one which

Never Let The Odds Keep You From Pursuing What You Know In Your Heart You Were Meant To Do.Leroy Satchel Paige

Page 169 of 180

Civil Law Q&As (2007-2013)

hectorchristopher@yahoo.com

courts authorization.

(A) the obligor has a moral obligation to


do, otherwise entitling the obligee to
damages.
(B) refers to an obligation in writing to
do or not to do.
(C) the obligee may enforce through the
court if violated by the obligor.
(D) cannot be judicially enforced but
authorizes the obligee to retain
the

obligors

payment

or

performance.

(79)

The

husband

administration

of

the

dbaratbateladot@gmail.com

assumed
familys

sole
mango

plantation since his wife worked abroad.


Subsequently, without his wifes knowledge,
the husband entered into an antichretic
transaction

with

company,

possession

and

management

giving
of

it
the

plantation with power to harvest and sell


the fruits and to apply the proceeds to the
payment of a loan he got. What is the
standing of the contract?
(A) It is void in the absence of the wifes
consent.
(B) It is void absent an authorization
from the court.
(C) The transaction is void and can
neither be ratified by the wife nor
authorized by the court.
(D) It is considered a continuing offer
by the parties, perfected only
upon the wifes acceptance or the

(81)
(80)

When

donations

October 1, 2011 but failed to pay her on

without reserving sufficient funds for his

due date. Bessy sent a demand letter to

support

Anne giving her 5 days from receipt within

or

the

donor

for

the

gives

Anne owed Bessy P1 million due on

support

of

his

dependents, his donations are


(A) Rescissible,

since

it

which to pay. Two days after receipt of the


results

in

letter, Anne personally offered to pay Bessy

economic lesion of more than 25%

in manager's check but the latter refused to

of the value of his properties.

accept the same. The 5 days lapsed. May

(B) Voidable, since his consent to the


donation is vitiated by mindless
kindness.

expenditure beyond his means.

donations impaired the support

dependents.

be

considered

extinguished?

himself

and

managers check, which is presumed


funded, amounts to a satisfaction of

(D) Reducible to the extent that the

to

obligation

(A) Yes, since Bessys refusal of the

(C) Void, since it amounts to wanton

due

Annes

his

the obligation.
(B) No, since tender of payment even
in

cash,

if

refused,

will

not

discharge the obligation without


proper consignation in court.

Never Let The Odds Keep You From Pursuing What You Know In Your Heart You Were Meant To Do.Leroy Satchel Paige

Page 170 of 180

Civil Law Q&As (2007-2013)

hectorchristopher@yahoo.com

(C) Yes, since Anne tendered payment of


the full amount due.

legal

The owner of a thing cannot use it in a

way that will injure the right of a third

(D) No, since a managers check is not


considered

(83)

dbaratbateladot@gmail.com

tender

in

the

Philippines.

person. Thus, every building or land is


subject to the easement which prohibits its
proprietor or possessor from committing
nuisance like noise, jarring, offensive odor,

(82)

The residents of a subdivision have

and smoke. This principle is known as

been using an open strip of land as passage

(A) Jus vindicandi.

to the highway for over 30 years. The owner

(B) Sic utere tuo ut alienum non

of that land decided, however, to close it in

laedas.

preparation for building his house on it.

(C) Jus dispondendi.

The residents protested, claiming that they

(D) Jus abutendi.

became

owners

of

the

land

through

acquisitive prescription, having been in

(84)

Janice and Jennifer are sisters. Janice

possession of the same in the concept of

sued

owners,

and

business partner for recovery of property

continuously for more than 30 years. Is this

with damages. The complaint did not allege

claim correct?

that Janice exerted earnest efforts to come

publicly,

peacefully,

(A) No, the residents have not been in

Jennifer

and

Laura,

Jennifers

to a compromise with the defendants and

continuous possession of the land

that

since they merely passed through

dismissed the complaint outright for failure

it in going to the highway.

to comply with a condition precedent. Is the

(B) No, the owner did not abandon his

such

efforts

failed.

The

judge

dismissal in order?

right to the property; he merely

(A) No, since Laura is a stranger to

tolerated his neighbors use of it for

the sisters, Janice has no moral

passage.

obligation to settle with her.

(C) Yes, residents of the subdivision

(B) Yes, since court should promote

have become owners by acquisitive

amicable

prescription.

relatives.

settlement

among

by

(C) Yes, since members of the same

prescription prevails over private

family, as parties to the suit, are

claims.

required to exert earnest efforts to

(D) Yes,

community

ownership

settle their disputes before coming

to court.

Never Let The Odds Keep You From Pursuing What You Know In Your Heart You Were Meant To Do.Leroy Satchel Paige

Page 171 of 180

Civil Law Q&As (2007-2013)

hectorchristopher@yahoo.com

(D) No, the family council, which would


ordinarily mediate the dispute, has

(87)

dbaratbateladot@gmail.com

When does the regime of conjugal

partnership of gains begin to exist?

been eliminated under the Family

(A) At the moment the parties take

Code.

and

declare

husband
(85)

X borrowed money from a bank,

wife

as

before

(B) At the time the spouses acquire

close friend. When the loan matured, Y

properties through joint efforts.

offered to pay the bank but it refused since

(C) On the date the future spouses

Y was not the borrower. Is the banks action

executed their marriage settlements

correct?

because this is the starting point of

(A) Yes, since X, the true borrower, did

their marital relationship.

not give his consent to Ys offer to

(D) On the date agreed upon by the

pay.

future spouses in their marriage

(B) No, since anybody can discharge

settlements since their agreement is

Xs obligation to his benefit.

the law between them.

(C) No, since Y, the owner of the

payment of the obligation.

and

other

officiating officer.

secured by a mortgage on the land of Y, his

collateral, has an interest in the

each

(88)

Josie, 18, married Dante, 25, without

her parents knowledge and consent, and

(D) Yes, since it was X who has an


obligation to the bank.

lived with him. After a year, Josie returned


to her parents home, complained of the
unbearable battering she was getting from

(86)

The right of a mortgagor in a judicial

foreclosure
property

to
after

performance
mortgage

of

but

redeem

the

mortgaged

his

default

the

conditions

before

the

sale

in

the

of

the

of

the

Dante, and expressed a desire to have her


marriage with him annulled. Who may
bring the action?
(A) Dante.
(B) Her parents.

mortgaged property or confirmation of the

(C) Josie herself.

sale by the court, is known as

(D) The State.

(A) accion publiciana.


(B) equity of redemption.

(89)

X, a married man, cohabited with Y, an

(C) pacto de retro.

unmarried woman. Their relation bore them

(D) right of redemption.

BB, a baby boy. Subsequently, after X

became a widower, he married Y. Was BB

legitimated by that marriage?

Never Let The Odds Keep You From Pursuing What You Know In Your Heart You Were Meant To Do.Leroy Satchel Paige

Page 172 of 180

Civil Law Q&As (2007-2013)

hectorchristopher@yahoo.com

(A) Yes, since his parents are now

dbaratbateladot@gmail.com

(D) Yes, as long as they leave sufficient

lawfully married.

property for themselves and for their


dependents.

(B) Yes, since he is an innocent party


and the marriage rectified the wrong
done him.

(92)

X owed Y P1.5 million. In his will, X

(C) No, since once illegitimate, a child

gave Y legacy of P1 million but the will

shall always remain illegitimate.

provided that this legacy is to be set off

(D) No, since his parents were not

against the P1.5 million X owed Y. After the

qualified

to

marry

each

other

when he was conceived.

set off, X still owed Y P500,000. Can Y still


collect this amount?
(A) Yes, because the designation of Y

(90)

as legatee created a new and

The presence of a vice of consent

vitiates the consent of a party in a contract

separate

and this renders the contract

between them, that of testator-

(A) Rescissible.

juridical

relationship

legatee.

(B) Unenforceable.

(B) It depends upon the discretion of

(C) Voidable.

the probate court if a claim is filed

(D) Void.

in the testate proceedings.


(C) No, because the intention of the

(91)

Can

properties

common-law

spouses

donate

testator in giving the legacy is to

of substantial value to one

abrogate his entire obligation to Y.

another?

(D) No, because X had no instruction in

(A) No, they are only allowed to give


moderate

gifts

to

each

his will to deliver more than the

other

legacy of P1 million to Y.

during family rejoicing.


(B) No, they cannot give anything of
value

to

each

other

to

(93)

Josie owned a lot worth P5 million

prevent

prior to her marriage to Rey. Subsequently,

placing their legitimate relatives at a

their conjugal partnership spent P3 million

disadvantage.

for the construction of a house on the lot.


legally

The construction resulted in an increase in

married spouses, such donations

the value of the house and lot to P9 million.

are not prohibited.

Who owns the house and the lot?

(C) Yes,

unlike

the

case

of

Never Let The Odds Keep You From Pursuing What You Know In Your Heart You Were Meant To Do.Leroy Satchel Paige

Page 173 of 180

Civil Law Q&As (2007-2013)

hectorchristopher@yahoo.com

(A) Josie and the conjugal partnership

(C) When

dbaratbateladot@gmail.com

fortuitous

circumstances

of gains will own both on a 50-50

prevented the plaintiff from filing

basis.

the case sooner.

(B) Josie will own both since the


value

of

the

house

and

(D) When the plaintiff is in possession


of the property.

the

increase in the propertys value is


less than her lots value; but she
is

to

reimburse

conjugal

partnership expenses.
(C) Josie still owns the lot, it being her
exclusive property, but the house
belongs to the conjugal partnership.
(D) The house and lot shall both belong to
the conjugal partnership, with Josie
entitled to reimbursement for

the

(95)

applied for a marriage license, making it


appear that they were over 25. They
married without their parents knowledge
before an unsuspecting judge. After the
couple has been in cohabitation for 6 years,
Lindas parents filed an action to annul the
marriage on ground of lack of parental
consent. Will the case prosper?
(A) No, since only the couple can

value of the lot.

(94)

An

action

for

Conrad and Linda, both 20 years old,

question
reconveyance

of

the

validity

of

their

marriage after they became 21 of

registered piece of land may be brought

age;

their

cohabitation

against the owner appearing on the title

convalidated the marriage.

also

based on a claim that the latter merely

(B) No, since Lindas parents made no

holds such title in trust for the plaintiff.

allegations that earnest efforts have

The action prescribes, however, within 10

been made to come to a compromise

years from the registration of the deed or

with Conrad and Linda and which

the date of the issuance of the certificate of

efforts failed.

title of the property as long as the trust had

(C) Yes, since the marriage is voidable,

not been repudiated. What is the exception

the couple being below 21 years of

to this 10-year prescriptive period?

age when they married.

(A) When the plaintiff had no notice of

(D) Yes, since Lindas parents never gave


their consent to the marriage.

the deed or the issuance of the


certificate of title.
(B) When the title holder concealed the
matter from the plaintiff.

(96)

Pepito executed a will that he and 3

attesting witnesses signed following the

formalities of law, except that the Notary

Public failed to come. Two days later, the

Never Let The Odds Keep You From Pursuing What You Know In Your Heart You Were Meant To Do.Leroy Satchel Paige

Page 174 of 180

Civil Law Q&As (2007-2013)

hectorchristopher@yahoo.com

Notary Public notarized the will in his law

dbaratbateladot@gmail.com

(B) Separate

since

their

property

office where all signatories to the will

relations with their legal spouses

acknowledged that the testator signed the

are still subsisting.

will in the presence of the witnesses and

(C) Co-ownership since they agreed to

that the latter themselves signed the will in

work for their mutual benefit.

the presence of the testator and of one

(D) Communal since they earned the

another. Was the will validly notarized?

same as common-law spouses.

(A) No, since it was not notarized on the


occasion

when

the

signatories

affixed their signatures on the will.


(B) Yes, since the Notary Public has
to

be

present

only

when

(98)

an action for revocation of a donation based


on acts of ingratitude of the donee?
(A) 5 years from the perfection of the

the

donation.

signatories acknowledged the acts

(B) 1 year from the perfection of the

required of them in relation to

donation.

the will.

(C) 4 years from the perfection of the

(C) Yes, but the defect in the mere

donation.

notarization of the will is not fatal to

(D) Such action does not prescribe.

its execution.
(D) No, since the notary public did not
require the signatories to sign their
respective attestations again.

(97)

spouses since both have been married to


other persons from whom they had been
in

Hardworking

fact
and

for

several

years.

bright,

each

earned

incomes from their respective professions


and enterprises. What is the nature of their

since

they

earned

the

same while living as husband and


wife.

Before

Karen

married

Karl,

she

inherited P5 million from her deceased

marriage. She later used part of the money


to buy a new Mercedes Benz in her name,
which Karen and her husband used as a
family car. Is the car a conjugal or Karens
exclusive property?
(A) It is conjugal property since the
spouses use it as a family car.
(B) It is Karens exclusive property since

incomes?
(A) Conjugal

(99)

mother which amount she brought into the

Venecio and Ester lived as common-law

separated

What is the prescriptive period for filing

it is in her name.
(C) It is conjugal property having been
bought during the marriage.

Never Let The Odds Keep You From Pursuing What You Know In Your Heart You Were Meant To Do.Leroy Satchel Paige

Page 175 of 180

Civil Law Q&As (2007-2013)

hectorchristopher@yahoo.com

(D) It is Karens exclusive property


since she bought it with her own
money.

(100)

Because of Xs gross negligence, Y

suffered

injuries

that

resulted

in

the

abortion of the foetus she carried. Y sued X


for, among other damages, P1 million for
the death of a family member. Is Y entitled
to indemnity for the death of the foetus she
carried?
(A) Yes, since the foetus is already
regarded as a child from conception,
though unborn.
(B) No, since Xs would not have known
that the accident would result in Ys
abortion.
(C) No,

since

dbaratbateladot@gmail.com

resulting to injuries to some lessees. Who

birth

determines

personality, the accident did not


result in the death of a person.
(D) Yes, since the mother believed in her
heart that she lost a child.

2010 Civil Law Exam MCQ


(September 12, 2010)
No.II. Multiple choice.
(A). A had a 4-storey building which was
constructed by Engineer B. After five years,
the building developed cracks and its
stairway eventually gave way and collapsed,

1659 NCC, the proprietor of a building or


should the lessees sue for damages? (1%)

structure is responsible for the damages

(1). A, the owner

resulting

from

its

total

or

partial

collapse, if it is due to lack of necessary

(2). B, the engineer

repairs.

(3). both A & B

Under Article 1723, NCC, the engineer


or architect who drew up the plans and

SUGGESTED ANSWER:

specifications for a building is liable for

3. Both A & B.

damage if 15 years from the completion

The lessee may proceed against A for

of

breach of contract, and against B for tort

collapse by a reason of a defect by those

or statutory liability. Under Article 1654

plans and specifications, or due to the

(2) of the New Civil Code, the lessor is

defects in the ground. This liability

obliged to make all the necessary repairs

maybe enforced against the architect or

in order to keep the leased property

engineer even by a third party who has

suitable for the use to which it has been

no privity of contract with the architect

devoted.

or engineer under Article 2192, NCC.

Consequently,

under

Article

the

structure

the

same

should

Never Let The Odds Keep You From Pursuing What You Know In Your Heart You Were Meant To Do.Leroy Satchel Paige

Page 176 of 180

Civil Law Q&As (2007-2013)

hectorchristopher@yahoo.com

defects in the construction plans or


specifications.

ALTERNATIVE ANSWER:
No.1. A , the owner .
The lessee can sue only the lessor for
breach of contract under Article 1659 in
relation to Article 1654, NCC. The lessee
cannot sue the architect or the engineer
because there was no privity of contracts
between them. When sued, however, the
lessor may file a third party claim
against the architect or the engineer.
ANOTHER ALTERNATIVE ANSWER:
No. 2. B, the Engineer .
Under

Article

1723

the

engineer

or

architect who drew up the plans and


specifications for a building is liable for
damages if within 15 years from the
completion of the structure, the same
should collapse by reason of a defect in
those plans and specifications, or due to
the defects in the ground. Under Article
2192 (NCC), however, if the damages
should be the result of any of the defects
in the construction mentioned in Art
1723, NCC, the third person suffering
damages may proceed only against the
engineer

or

architect

dbaratbateladot@gmail.com

or

contractor

within the period fixed therein. The


damages suffered by the lessee in the
problem are clearly those resulting from

No. 4. None of the above.


(B) O, owner of Lot A, learning that
Japanese soldiers may have buried gold
and other treasures at the adjoining vacant
Lot B belonging to spouses X & Y, excavated

The general rule is that the treasure


shall belong to the spouses X and Y, the
owner of Lot B. Under Article 438 (NCC),

in Lot B where she succeeded in unearthing

the exception is that when the discovery

gold and precious stones. How will the

of a hidden treasure is made on the

treasures found by O be divided? (1%)

property of another and by chance, one-

(1). 100% to O as finder

half thereof shall belong to the owner of


the

land

and

the

other

one-half

is

(2). 50% to O and 50% to the spouses X

allowed to the finder. In the problem, the

and Y

finding

(3). 50% to O and 50% to the state


(4). None of the above.

of the

treasure was

not

by

chance because O knew that the treasure


was in Lot B. While a trespasser is also
not entitled to any share, and there is no
indication in the problem whether or not

SUGGESTED ANSWER:

O was a trespasser, O is not entitled to a

Never Let The Odds Keep You From Pursuing What You Know In Your Heart You Were Meant To Do.Leroy Satchel Paige

Page 177 of 180

Civil Law Q&As (2007-2013)

hectorchristopher@yahoo.com

acceptance by the donees father alone

share because the finding was not by


chance.

(C) A executed a Deed of Donation in favor


of B, a bachelor, covering a parcel of land
valued at P1 million. B was, however, out of
the country at the time. For the donation to
be valid, (1%)
(1). B may e-mail A accepting the donation.
(2). The donation may be accepted by Bs
father with whom he lives.
(3). B can accept the donation anytime
convenient to him.
(4). Bs mother who has a general power of
attorney may accept the donation for him.
(5). None of the above is sufficient to make
Bs acceptance valid
SUGGESTED ANSWER:
No. 5 None of the above is sufficient to
make B's acceptance valid .
Since

the

donation

covered

an

immovable property, the donation and


the

acceptance

must

dbaratbateladot@gmail.com

be

in

public

document and e-mail is not a public


document. Hence, No.1 is false.
No. 2 and No.4 are both false. The

(D) A executed a 5-page notarial will before a


or

mother

alone,

even

in

public

document, is not sufficient because the

notary public and three witnesses. All of them


signed each and every page of the will.

father and mother did not have a special

One of the witnesses was B, the father of

power of attorney for the purpose. Under

one of the legatees to the will. What is the

Article 745 (NCC), the donee must accept

effect of B being a witness to the will? (1%)

the donation personally, or through an


authorized person with a special power of
attorney for the purpose; otherwise, the
donation shall be void.

No.3 is also false. B cannot accept the


donation anytime at his convenience.

(1). The will is invalidated


(2). The will is valid and effective
(3). The legacy given to Bs child is not valid
SUGGESTED ANSWER:

Under Article 749 NCC, the donee may

No. 3. The legacy given to B's child is not

accept the donation only during the

valid.

lifetime of the donor.

The validity of the will is not affected by


the legacy in favor of the son of an

Never Let The Odds Keep You From Pursuing What You Know In Your Heart You Were Meant To Do.Leroy Satchel Paige

Page 178 of 180

Civil Law Q&As (2007-2013)

hectorchristopher@yahoo.com

attesting witness to the will. However,


the said legacy is void under Article 823
NCC.

dbaratbateladot@gmail.com

ALTERNATIVE ANSWER:
(D). all the above
(2). A deposit made in compliance with a

ALTERNATIVE ANSWER:

legal obligation is:

No. 2 .The will is valid and effective.


(A). an extrajudicial deposit;
Under Article 823 ( NCC ),the legacy
given

in

favor

of

the

son

of

an

instrumental witness to a will has no


effect on the validity of the will. Hence,
the will is valid and effective.

(B). a voluntary deposit;


(C). a necessary deposit;
(D). a deposit with a warehouseman;
(E). letters a and b

2007 Civil Law Exam MCQ

SUGGESTED ANSWER:

(September 09, 2007)

(C). a necessary deposit

No.IX. Multiple choice: Choose the right


answer. (2% each)
(1). The parties to a bailment are the:
(A). bailor;

(3). A contract of antichresis is always:


(A). a written contract;
(B). a contract, with a stipulation that the
debt will be paid through receipt of the
fruits of an immovable;

(B). bailee;
(C). Involves the payment of interests, if
(C) comodatario;

owing;

(D). all the above;

(D). All of the above;

(E). letters a and b

(E). Letters a and b

SUGGESTED ANSWER:

SUGGESTED ANSWER:

(E). letters a and b

Never Let The Odds Keep You From Pursuing What You Know In Your Heart You Were Meant To Do.Leroy Satchel Paige

Page 179 of 180

Civil Law Q&As (2007-2013)

hectorchristopher@yahoo.com

(D). All of the above;

(D).

1/3

of

dbaratbateladot@gmail.com

the

total

debts

must

be

represented by the approving creditors;


(4). An, assignee in a proceeding under the
Insolvency Law does not have the duty of:

(E). Letters a and b

(A). suing to recover the properties of the

SUGGESTED ANSWER:

state of the insolvent debtor;


(C). 3/5 of the number of creditors
(B). selling property of the insolvent debtor;

should agree to the settlement;

(C). ensuring that a debtor corporation

[Note: Items 4&5 on Insolvency Law are

operate

and

not included within the coverage of Civil

effectively while the proceedings are

Law but Commercial Law. It is therefore

pending;

suggested that the examinees be given

the

business

efficiently

(D). collecting and discharging debts owed


to the insolvent debtor.

full credit for the two items regardless of


their answers.]

SUGGESTED ANSWER:
(C). ensuring that a debtor corporation
operate

the

business

efficiently

and

effectively while the proceedings are

References:

(2007, 2009, 2010)

pending;
(5). In order to obtain approval of the

Answers
to
Bar
Examination
Questions by the UP LAW COMPLEX

UP LAW REVIEW

proposed settlement of the debtor in an


insolvency proceeding.
(A). the court must initiate the proposal
(B). 2/3 of the number of creditors should
agree to the settlement;
(C). 3/5 of the number of creditors
should agree to the settlement;

PHILIPPINE ASSOCIATION OF LAW


SCHOOLS (2008)
lawphil.net

Never Let The Odds Keep You From Pursuing What You Know In Your Heart You Were Meant To Do.Leroy Satchel Paige

Page 180 of 180

You might also like